Sei sulla pagina 1di 233

CONTENTS

s.no TOPIC PAGES


INDEX 1
1 Ratio & Proportion 2-15
2 Percentage 16-21
3 Profit & Loss 22-26
4 Blood Relations 27-35
5 Calendar 36-40
6 Clocks 41-46
7 Time & work 47-55
8 Time & Distance 56-70
9 Cube 71-75
10 Directions 76-82
11 Permutations & Combinations 83-91
12 Probability 92-104
13 Average 105-113
14 Alligation & Mixture 114-118
15 Number System 119-127
16 Number series 128-133
17 Partnership 134-141
18 Analogy 142-150
19 Syllogism 151-160
20 puzzle test 161-169
21 Races 170-173
22 Ranking 174-178
23 Ages 179-183
24 Simple Interest & Compound Interest 184-190
25 simple equations 191-200
26 Coding & Decoding 201-213
27 Data interpretation 198-228
28 LCM & HCF 229-232

1
RATIO AND PROPORTION
Ratio: Ratio is a fraction. Which shows the comparison of two (or) more than two quantities.
If a and b two similar quantities, the ratio between a and b is expressed as a : b (or)
a/b (b≠0).
a is to b
↙ ↘
Antecedent Consequent
Ratio is always should be represented in most simplified form.
20 : 60 → 1 : 3
1/2 : 1/3 → 3 : 2
1/2 : 1/3 : 1/4 → 6 : 4 : 3
In a Ratio all quantities should be belongs to same group (or) units.
Ex: What is the Ratio of 18 inches to 3 feet?
3 feet = 3 × 12 inches = 36 inches
18 : 36 = 1 : 2
The multiplication (or) division of each term of a Ratio by the same non-zero number
does not affect the Ratio.
3 : 5 is same as 6 : 10 (or) 9 : 15 (or) 12 : 20 etc.
It a : b is a Ratio
1. Duplicate Ratio a² : b²
2. Triplicate Ratio a3 : b3
3. Sub duplicate Ratio a1/2 : b1/2
4. Sub triplicate Ratio a1/3 : B1/3 (Exists only when each Ratio has two quantities.)

Inverse Ratio: If a : b is Ratio, then the inverse ratio of a : b is 1/a : 1/b = b : a


• Inverse ratio of a : b : c = bc : ac : ab
• Inverse ratio of 1 : 2 : 3 : 4 = 24 : 12 : 8 : 6
= 12 : 6 : 4 : 3
• 1/x : 1/y : 1/z = 2 : 3 : 5, then x : y : z = 15 : 10 : 6
• The speeds of 3 cars are in the ratio 3 : 4 : 5. The ratio between the times
taken by them to travel a fixed distance, is.
S ∞ 1/T
S 3:4:5
T 20 : 15 : 12
Compound Ratio: The Ratio between the products of all antecedents to the product
of all consequents is called compound ratio.
• Compound ratio of a : b, c : d, e : f = ace : bdf.
• Compound ratio of a : b : c, d : e : f does not exist (compound ratio mean it
contains antecedent and consequent)
• A : b = 2 : 3, b : c = 4 : 5, c : d = 6 : 7, then a : d

2
2 × 4 × 6 : 3 × 5 × 7 → 16 : 35
If any No.of fractions are equal then the ratio of numerator is equal to ratio of
denominators in the same order.
• a/11 = b/12 = c/13 a : b : c = 11 : 12 : 13
• 2A = 3B = 4C A:B:C
A/(1/2) = B/(1/3) = C/(1/4) 1/2 : 1/3 : 1/4 = 6 :4 : 3
• If a/2 = b/3 = c/5, then find the value of (a + b + c)/a
= 10/2
=5
a/b = c/d = e/f - - - - - - - = (a + c + e - - - - )/(b + d + f - - - -)
i.e Sum of numerators/sum of denominators
• a/(a + b + c) = b/(a + b + c) = c/(a + b + c) = k k=?
(a + b + c)/3(a + b + c) = 1/3
→ Proportion: (:: is as to)

• The equality of two ratios is called proportion.


• If a/b = c/d , then a, b, c, d are said to be in proportional
a = 1st proportional
b = 2nd proportional
c = 3rd proportional
d = 4th proportional
• If any four quantities are in proportion, then the product of middle two terms
is equal product of extreme terms.
Ad = bc
• 2, x, 3, 12 are in proportion then x = 8.
• P, 8, 4, 2 are in proportion then p = 16.
• What is the 3rd proportion of 7, 8, 24.
7, 8, x, 24
7 × 24 = 8 × x x = 21
Continued Proportion: If a/b = b/c , then a, b, c are said to be in continued
proportion
a = 1st proportion
b = 2nd proportion
c = 3rd proportion
What is mean proportion between 16 and 25 → 20
What is mean proportion between 4/9 and 16/25
What is mean proportion of p, q, r
Does not exist

Note: If p,q,r are in proportional then mean proportional is ‘q’.


1. What number must be subtracted from 11, 15, 21 such that to make the resultant
numbers are in continued proportion.
(11 – x) × (21 – x) = (15 – x)2

3
231 – 11x – 21x + x2 = 225 – 30x + x2
2x = 6
X=3
2. What should be added to each of four numbers 6, 14, 18, 38 to make them
proportional?
(6 + x) (38 + x) = (14 + x) (18 + x)
228 + 44x x2 = 252 + 32x + x2
12x = 24
X=2
Componendo
If a/b = c/d
a/b + 1 = c/d + 1
(a + b)/b = (c + d)/d
Dividendo
If a/b = c/d
a/b – 1 = c/d – a → (a – b)/b = (c – d)d
Componendo-dividendo (c – d)
(a + b)/(a – b) = (c + d)/(c – d)
→ If a/b = c/d → (a + b)/(a – b) = (c + d)/(c – d)
(or)
(a + b)/(a – b) = c/d → a/b = (c + d)/(c – d)
(or)
a/b = (c + d)/(c – d) → (a + b)/(a – b) = c/d
3. If (x + y) : (x – y) = 4 : 1, then(x2 + y2) : (x2 - y2) = ?
(x + y)/(x – y) = 4/1
x/y = 5/3 → x2/y2 = 25/9 → (x2 + y2)/(x2 - y2) = 34/16 = 17/8
4. X = (4ab)/(a + b), then find
(x + 2a)/(x – 2a) + (x + 2b)/(x – 2b) = ?
x/2a = 2b/(a + b) x/2b = 2a/(a + B)
(x + 2a)/(x – 2a) = (3b + a)/(b – a) (x + 2b)/(x – 2b) = (3a +b)/(a – b)
(3b + a)/(b – a) + (3a + b)/(a – b)
(-3b + a + 3a + b)/(a – b) = (2a – 2b)/(a – b) = (2(a – b))/(a - b) = 2
5. (3a + 5b)/(3a – 5b) = 5, then a : b = ?
3a/5b =6/4
a/b = 30/12 = 10/4 = 5/2
a:b=5:2

4
5. {√(x + 1) + √(x – 1)}/{√(x + 1) – √(x -1)} = 2, then find the value of x
[√(x + 1)]/√(x – 1) = (2 + 1)/(2 – 1)
√{(x + 1)/(x – 1)} = 3/1
(x + 1)/(x – 1) = 9/1
→ x/1 = 10/8
→ x = 5/4
6. If a : b = 2 : 3
b:c=3:4 a:b:c=2:3:4
7. If a : b = 2 : 3
b:c=4:5 a : b : c = 8 : 12 : 15
8. If a : b = 1 : 2
b:c=3:1 a : b : c : d= 3 : 6 : 2 : 2
1 2 2 2
3 3 1 1
1 1 1 1
3 :6:2:2
9. 6400Rs. are divided among three workers in the ratio 3/5 : 2 : 5/3
→ 3/5 × 15 : 2 × 15 : 5/3 ×15
→ 9 : 30 : 25
64 p = 6400
30 p = 3000/-
10. The ratio of present ages of A and B is 2: 3 and the ratio of present ages of B and C is
4 : 5. If ‘C’ is 21 years older than A. Then find ‘B’s present age?
A:B:C
2:3:3 7p = 21 years
4:4:5 12p = 36 years
8 : 12:15
11. A certain amount is to be distributed among 3 persons A, B and C such that the share
of A is twice of the share of B and the share of B is thrice of the share C. If ‘A’
received 2000/- more than ‘C’ then find the total amount?
A:B:C A = 2B A:B=2:1
2:1:1 B = BC B:C=3:1
3:3:1 5P= 2000
6:3:1 10P = 4000/-
12. Instead of dividing a certain amount among P, Q, R in the Ratio 1/2 : 1/3 : 1/4 , by
mistake it was divided in the ratio of 2 : 3 : 4 as a result ‘R’ got 25/- more then find
total amount?
P:Q:R P Q R
6:4 :3 13P × 9 54 36 27
2:3 :1 9P × 13 26 39 52
25P = 25/-
117P = 117/-

5
13. A certain amount is divided among 3 persons A, B and C such that the share of ‘A’ is
half of the combined shares of B and C. The share of B is 1/3 of the combined shares
of A and C. If B received 200/- less than C received then then find the total amount?
A = 1/2 (B + C) →A/(B + C) = 1/2 3P × 4 = 4/8
B = 1/3 (A + C) →B/(A + C) = 1/3 4P × 3 = 3/9
A B C 2P =200
4 : 3 :5 12P =1200/-
14. A bag contains two-rupee, one-rupee and 50-p coins in the Ratio 4 : 2 : 1. If the total
money in the bag is Rs 210. Find the number of one-rupee coins?
2/- 1/- 50p 10.5 → 7 coins (total)
4 : 2 : 1 10.5 → 2 coins (one rupee)
210 →? 40 coins
20
15. The sum of the squares of three numbers which are in the ratio 2 : 3 : 4 is 725. What
is smallest number?
(2x)2+ (3x)2 + (4x)2 = 725
29x2 = 725
X2 = 25
X=5 least number = 2 × 5 = 10
16. Two numbers are in the ratio of 3 : 5. If each number is increased by 10, the ratio
becomes 5 : 7,then find the numbers.
3:5 2p = 10
10| |10 3p = 15
5:7 5p = 25
17. Two numbers are in the ratio of 5 : 11. If each number is increased by 8, then the
ratio becomes 1 : 2. Find the sum of the two numbers.
5 : 11 1p = 8
+ 8| |+ 8 16p = 128
1:2
6 : 12 × 6

6
18. The present ages of father and his son are in the ratio 5 : 3. 9 years ago their ages are
in the ratio 23 : 12. Find the present age of father.
F S
5 : 3 9p = 9 years
55 : 33 55p = 55 years
-9| |-9
23 : 12
46 : 24
19. In a mixture the ratio of milk and water is 2 : 1. If 30 lit of water is added to mixture
then the ratio becomes 1 : 2. Find quantity of milk in the mixture?
M:W
2 : 1
1 : 2×2 | 3p = 30 lt
2 : 4 2p = 20 lt
20. Two equal glasses are respectively 1/3 and 1/4 filled up with water and the contents
are mixed in a tumbler. The ratio of milk and water in the tumbler is?
M W M W
(3p) 1 2 1 3 (4p)
4 : 8 3 : 9
7 : 17
21. The incomes of A & B in the ratio of 6 : 5 and their expenditures in the ratio 3 : 2. If
saves are 3000/- and 4000/- respectively. Then find the income of A?
A B
I 6x 5x (6x – 3000)/(5x – 4000) = 3/2
E 3 2 x = 2000
S 3000 4000 6x = 12000/-
22. The incomes of A & B are in the ratio of 3 : 2 and their expenses are in the ratio of 2
: 3. Find the ratio of their savings, If ‘B’ spends 3/4th of his income.
A :B
(I) (6) 3 : 2 (4) × 2 B (E )/B(I) = 3/4
(E) 2 : 3
4: 1

7
23. In a class of 30 students, which of the following cannot be ratio of boys and girls?
(a) 1 : 29 (b) 2 : 3 (c) 1 : 3 (d) cannot be determined
1+3=4p but 30 is not divisible by 4
So answer is 1:3
24. If a carton containing a dozen mirrors is dropped, which of the following can be the
ratio of broken mirrors to unbroken mirrors?
(a) 1 : 4 (b) 2 : 3 (c) 1 : 6 (d) 1 : 2
1+2=3p, 12 is exactly divisible by 3
So answer is 1:2
25. Two candles of the same height are lighted at the same time. The first is consumed
in 8 hours and the second in 6 hours. Assuming that each candle burns at constant
rate, in how many hours after being lighted the ratio between the first and second
candles becomes.
2:1 6 8 (24)
(24 – 4x)/(24 – 3x) = 2/1 x = 4 hours 48 min
26. There are two candles of equal lengths and of different thickness. The thicker one
last of six hours the thinner 2 hours less than thicker one. Ramesh lights two candles
of the same time. When he went to bed he saw the thicker one is twice the length of
the thinness one. How long ago did hours Ramesh light the two candles?
6 hr 4 hr (12 lcm)
(12 – 2x)/(12 – 3x) = 2/1 → 12 – 2x = 24 – 6x
→ 4x = 12
X = 3 hr
27. A dog pursues a hare and takes 5 leaps for every 12 leaps of the hare, but 1 leap of
the dog is equal to 3 leaps of the hare compare the rates of the dog and the hare?
D H
5L 12L
1D = 3 H
15 H 12 H
5:3
28. A hound pursues a hare and takes 6 leaps for every 9 leaps of the hare, but 3 leaps of
the hound are equal to 5 leaps of the hare compare the rates of the hound and hare.
Hound Hare 3 Hu = 5 H →1 Ho = 5/3 H
6L 9L 6 × 5/3 = 10 H
10 H : 9H → 10 : 9
29. An amount of Rs. 2430 is divided among A, B and C such that if their shares be
reduced by Rs. 5, Rs. 10 and Rs. 15 respectively, the remainders shall be in the ratio
of 3 : 4 : 5 then ‘B’ share was?
2430 – 30 = 2400 → 3 : 4 : 5 B got 810/-

8
30. The contents of two vessels containing water and milk are in the ratio 1 : 2 and 2 : 3
are mixed in the ratio of 2 : 1 find the ratio after mixing the two mixture.
W : M W : M W : M W : M
(3) 1 : 2 2 : 3 (3p) 1 : 2 2 : 3 (5p)
10 lt 20 lt 6 lt 9 lt 5 : 10 6 : 9
2 : 1 (15 l) 2 : 1 (15 l)
(30 l) 16 lt : 29 lt →16 : 29 (30 l) 10 + 6 = 16 20 + 9 = 29 →16 : 29
31. Vessels A and B contain mixtures of milk and water in the ratio 4 : 5 and 5 : 1
respectively. In what ratio should quantities of mixtures be taken from A to B to form
a mixture in which milk to water in the ratio 5 : 4.
M : W M : W
(9p) 4 : 5 5 : 1 (6p)
8 : 10 15 : 3
5 : 4
(8x + 15y)/(10x + 3y) = 5/4
32x + 60y = 50x + 15y
x/y = 45/18 = 5/2
x:y=5:2
32. Two vessels A and B contain milk and water mixed in the ratio 5 : 3 and 2 : 3 when
these mixtures are mixed to form a new mixture containing half milk and half water,
they must be taken in the ratio?
A B
(8p) 5 : 3 2 : 3 (5p)
M:W M:W
25 : 15 16 : 24
1:1
(25x + 16y)/(15x + 24y) = 1/1
25x + 16y = 15x + 24y
X:y=4:5
33. A diamond falls down and breaks into 3 pieces whose weights are in the ratio of 1 : 3
: 6. The value of a diamond is proportional to square of it’s weight. It the original
value was Rs. 30,000, then what is the loss in value due to breakage?
Weight of diamonds ratio 1 : 3 : 6
Weight of diamonds x, 3x, 6x
Total weight of diamond = x + 3x + 6x + 10x
Value of diamonds → x2, 9x2, 36x2
Total value of diamonds = x2, 9x2, 36x2 = 46x2
Total value of diamond = (10 x2)2 = 100x2

9
100x2 =30,000/- 100x2= 30,000
10x = 100x2- 46x2 x2 = 300
= 54x2 54x2= 54 × 300
= 16,200

TOP MOST RATIOS PLACEMENT QUESTIONS


1. Diamond’s value is proportional to its weight square. A piece of diamond broke and
the weights of the broken pieces were in the ratio 1:2:3:4:5. The loss in value is Rs
85000/-. What is the value of the diamond twice the Weight of the original
diamond?(TCS, WIPRO, ZENTECH, DRDO GATE, CAT)
SOLUTION: - Weight of diamond pieces = 1: 2: 3: 4: 5
Weight of diamond pieces = x, 2x, 3x, 4x, 5x
Total weight of diamond = x, 2x, 3x, 4x, 5x = 15x
Value of total diamond = (15x) 2 = 2252
Value of diamond pieces = x2 + (2x) 2 + (3x) 2 + (4x) 2 + (5x) 2
= x2 + 4x2 + 9x2 + 16x2 + 25x2
= 55x2
Original value = 225x2
Loss value = 225x2 - 55x2 = 170x2
= 170x2 =85000
= x2 =500
Twice the weight of original diamond = 15x × 2 =30x
Value of twice of the original diamond = (30x) 2 = 900x2
= 900 × 500
= 450000

2. In a mixture R is two parts and S is one part. In order to have S as 25% of the mixture
how much R is to be added?(TCS, GRE, BIRLA SOFT)
SOLUTION: - R S
2 1
75% 25%
3 1
So that R should be add one part.

3. IBM and SGI quote for a tender. On the tender opening day, IBM realizes that their
quotation are in the ratio 7:4 and decreases its price to make it Rs 1 lakh lower than
the SGI’s quoted price. SGI later found out that the final quotes of the two were in
the ratio of 3:4. What was the price at which IBM won the bid?(TCS, IBM, CAPGEMINI,
GENPACT)
SOLUTION: - 1BM 5GI
3 : 4

10
Difference one part = 1 lakh
IBM = 3 parts = 3 lakhs

4. Concentrations of three wines A, B and C are 10, 20 and 30 per cent respectively.
They are mixed in the ratio 2: 3: x resulting in a 23% concentration solution. Find
x(WIPRO)
SOLUTION: - A B C
10% 20% 30%
2 : 3 : x
2 × 10/100 + 3 × 20/100 + x × 30/100 = (2 + 3 + x) 23/100
20 + 60 + 30x = (5 + x) 23
30x + 80 = 115 + 23x
7x = 35
X=5

5. 3 glasses containing mixture of water and alcohol in ratio 2:3 , 3:4 and 5:9 When all 3
are mixed what is the new ratio?(MINDTREE, TCS, ORACLE, HUAWEL)
SOLUTION:- 2 : 3 3:4 5:9
5P 7P 14P
So that 1 cm of 5p, 7p, 14p = 70p
5p × 14, 7p × 10 14p × 5
28: 42 30: 40 25: 45
Resultant ratio = (28 + 30 + 25): (42 + 40 + 45)
= 83: 127

6. There are 200 questions on a 3 hr examination. Among these questions are 50


mathematics problems. It is suggested that twice as much time be spent on each
maths problem as for each other question. How many minutes should be spent on
mathematics problems (L&T,3I INFOTECH, SASIEN, INFOTECH, INFOVIEW, INFO SOFT)
SOLUTION:- Time taken of 1 math problem = 2 general problems
M = 2G
50M + 150G = 180min
50M + 150/2M = 180min
125M = 180
M = 180/125
Total time taken for 50M problems = 180/125 × 50 = 72 min.

7. In a class composed of x girls and y boys what part of the class is composed of girls
a) y/(x + y) b) x/xy c) x/(x + y) d) y/xy (ACCENTURE, IBM)
SOLUTION:- Number of girls = x
Part of the class is composed of girls = girls/total students
= x/(x + y)

8. The ratio of white balls and black balls is 1:2. If 9 grey balls is added it becomes 2:4:3.
Then what is number of black balls? (GRE, INFOTECH, INFOVIEW, INFOSOFT)

11
SOLUTION:- W B G
1 : 2
2 : 4 : 3
3P = 9 balls, black balls = 4p = 12 balls.

9. After gathering 770 chestnuts, three girls divided them up, so that the amounts were
in the same proportion of their ages. As often as Mary took four chestnuts, Nelli took
three and for every six that Mary took, Susie took seven. How many Chestnuts did
each girl get?(INFOSIS, ORACLE)
SOLUTION:- M N M S
4 : 3 , 6 : 7
N: M = 3: 4
M: S = 6 : 7
N: M: S = 3: 4: 4
6: 6: 7
18: 24: 28
Total chestnuts = 70p = 770
1 part = 11
M = 24 × 11 = 264
N = 18 × 11 = 198
S = 28 × 11 = 308

10. Two solutions have milk & water in the ratio 7:5 and 6:11.Find the proportion in
which these two solutions should be mixed so that the resulting solution has 1 part
milk and 2 parts water? (CTS,VEDANTA)
SOLUTION:- 7/12 6/17 (milk)
1/3
(6/17 – 1/3): (1/3 – 7/12)
Negative value not possible.

11. There are two categories of persons in ratio A: B = 2:3. A type earns 2.5 dollars and B
type 1 dollar. Total money earned by all is 24 dollars. What is the total number of
persons?(TCS, SYNTEL)
SOLUTION:- A B
2 : 3
2x, 3x persons in A, B.
2x × 2.5 + 3x × 1 = 24
5x + 3x = 24
8x = 24
x=3

12. 8 litres are drawn from a cask full of wine and is then filled with water this operation
is performed 3 more times the ratio of the quantity of wine now left in cask to that of
water is 16:65 how much wine did the cask hold originally(Genpact ,WIPRO, SSC, CGL,
SASLEN)
SOLUTION:- Initial x lt wine in a cask.
Total 4 times water replaced

12
{x ×[(x – 8)/x]4 /x} = 16/18
[(x – 8)4/x2 = 16/18
(x – 8)/x = 2/3
x = 24 lt

13. The ratio of the ages of the father and the son is 5:3. After 10 years it will be in the
ratio 3:2. What will be their ages?(CTS, DE SHAW)
SOLUTION: - F S
5 : 3 (8P)
After 10 years
3 : 2 (5P)
F : S
5 : 3
After 10 years
6 : 4
One part = 10 years
Father = 50 years
Son = 30 years

14. A container contains wine to its full capacity. 4 liters of wine is taken out and replaced
by water. 4 liters of the mixture is taken out and 4 liters of water is poured. The ratio
of wine to the total is now 36:49.Find the capacity of the container?
SOLUTION:- Initially x lt of wine in a containers taken water replaced
{x × [(x × 4)/x]2/x} = 36/49
(x × 4)2/x2 = 36/49
(x × 4)/x = 6/7
X = 28 lt

15. The milk and water in two vessels are in the ratio 5:4 and 3:4 respectively. In what
ratio the liquid in both the vessels are mixed to obtain a new mixture in vessel C
containing equal amount of water and milk(TCS, IBPS, SSC, CGL, DE SHAW)
SOLUTION:- A B C
5:4 3:4 1:1
5/9 x + 3/7 x = 1/2 (x + y)
10x + 54 y = 63x + 63y
7x =9y
x/y = 9/7
So option 45 = 35 is right option.

16. Karan and Arjun run a 100 meter race, where Arjun beats Karan by 10 meters. To do a
favour to Karan, Arjun starts 10 meters behind the starting line in a second 100 meter
race. They both run at their earlier speeds. Which of the following is true in
connection with the second race(TCS, CAT, GMAT, HCL, CAT)
a. Karan and Arjun reach the finishing line simultaneously
b. Arjun beats Karn by 1 meter
c. Arjun beats Karan by 11 meters
d. Karan beats Arjun by 1 meter

13
19. A mixture of 40 litres of salt and water contains 70% of salt how much water must be
added to decrease the salt percentage to 40%? (COCUBES)
SOLUTION:- Salt 70% of 40 = 28 lt
28/(40 + x) = 40/100
X = 30 lt
20. A chocolate drink contains 11% pure chocolate by volume. If 13 liters of pure milk is
added to 29 liters of this drink, the percentage of pure chocolate in the new mixture
is?(TCS)
SOLUTION:- 11% OF 29 lt = 11/100 × 29 = 3.19 lt
Percentage of chocolate = 3.19/42 × 100
= 7.56%

21. A wheel named ‘A’ with 16 cogs is meshed with another wheel B with 24 cogs which
in turn meshed with another wheel ‘C’ with 8 cogs. If wheel ‘A’ makes 100 revolutions
how many revolutions will wheel ‘C’ make?
SOLUTION: - The number of revolutions made by the gears are inversely proportional
to the number of cogs.
A B C
Cogs 16 : 24 : 8
2 : 3 : 1
Inverse 3 : 2 : 6
A: C = 3: 6 = 1: 2
For 1 revolution of A, gear C makes 2.
For 100 revolution of A, gear C makes 200.

15
PERCENTAGE
Percentage: Any value that can be compared to hundred is called percentage. Per cent
mean divided by hundred.

• To express x% as a fraction: We have, x% = x/100.


Thus, 20% = 20/100 = 1/5 ; 48% = 48/100 = 12/25, etc.
• To express a/b as a per cent: We have, a/b = [a/b × 100]%.
Thus, 1/4 = [1/4 × 100]% = 25% ; 0.6 = 6/10 = 3/5 = [3/5 × 100]% = 60%

100% = 1 20% = 1/5 10% = 1/10 33 1/3 % =1/3


(33.33%)

50% = 1/2 40% = 2/5 30% = 3/10 66 2/3 % = 2/3


(66.66%)

25% = 1/4 60% = 3/5 70% = 7/10 11 1/9 % = 1/9


(11.11%)

12.5% = 1/8 80% = 4/5 90% = 9/10 9 1/11 % = 1/11


(9.09%)

12.5% = 1/8 1 = 100% 1/6 = 16.66%

37.5% = 3/8 1/2 = 50% 1/7 = 14.28%

62.5% = 5/8 1/3 = 33 1/3 % 1/8 = 12.5%

87.5% = 7/8 1/4 = 25% 1/9 = 11.11%

1/5 = 20% 1/10 = 10%

1. ‘A’ is what percentage of B or what per cent is A of B what percentage of A is B.


Is/of × 100

2. What percentage is 120 of 90?


120/90 100 = 133 1/3 %

3. ? % of 64 = 8
8/64 × 100 = 12.5%

16
4. 30% of ? = 180 180/x × 100 = 30x = 600
5. What percentage of 2/7 is 1/35
6. What is 25% of 25%
a. 1 b. 625 c.0.0625
7. What percentage is 3% of 5% = [3/100]/[5/100] ×100
= [3/100] × [100/5] × 100
= 60%
8. 60 = ? % of 400
60/400 × 100 = 15%
9. Increase / decrease percentage:
Increase percentage = difference/least value × 100
Decrease percentage = difference/highest value × 100
If increase % is given, as R%, then

Decrease % = R/(100 + R) × 100

Increase % = R/(100 – R) × 100

10. The price of bananas in 1995 was 24 Rs/- dozen and in 1996 it was 28 Rs/- per dozen.
The find increase percentage?
60/80
60 is how much percentage less than 80
80 is how much percentage more than 60
11. If A’s salary is 25% less than that of B, then how many percentage is B’s salary more
than that of A?
Increase % = 25/(100 – 25) × 100
= 25/75 × 100
= 33 1/3 %
12. If the price of tea is increased by 20% by how much percentage must the
consumption of tea be decreased so as not to increase the expenditure?
Dec % = 20/(100 + 20) × 100
= 20/120 × 100
= 1/6 × 100
= 16.66%
13. In this chapter any unknown value is taken as 100
14.

20% -10% R2
Number R1
100 120 108
Net change = 8% effect = 8% increases
15. Number + 20% R1 -10% R2 +30%140.4
32.4

17
100 120 108
Net change = 40.4%
Effect = 40.4% increases
Initial × 120/100 × 90/100 × 130/100 = final value.

16. A number is increased by 40%, then the number becomes 84. Find the original
number?
Number + 40% R1 140 = 84
40 100 = ?
100 140 60

17. A number is decreased by 25%, then the number becomes 300. Find the 40% of the
number?
100 (-25%/25) 75 = 300 40 = ? 160

18. The difference between 72% of a number and 54% of a number is 270. Find 16% of
the number?
72 ~ 54 18 = 240
16 = ?

19. The length of rectangle is increased by 10% and breadth decreased by 10%. Then,
the area of new rectangle is.
Area + 10% 110 -10% 99
100 10 - 11
1% decreased 10 × 10 = 100 11 × 9 = 99

20. If the radius of sphere is decreased by 10% then find percentage change in the
volume of the sphere.
100 -10% 90 -10% 81 -10% 72.9 27.1%
Dec%
-10 -9 - 8.1

21. A man salary is increased by 30% and then decreased by 20% and again increased by
40%. If it’s latest salary is 2912 rupees then find the original salary.

100 + 30% 130 -20% 104 +40%145.6


-26 41.6

145.6 = 2912 100 = ? 2000/-

18
22. The population of a town is 368000. It increases annually at the rate of 30% p.a.
what was it’s population 2 years ago, what will be its population after 2 years?

100 + 30% 130 -30% 169 169 = 3,38,000


2 years ago present 100 = 2,00,000
100 + 30% 130 30% 169 100 = 3,38,000
169 = ?
5,71,220
Present

23. A man spends 10% of his income on rent, 20% of the remaining on food and 25% of
the remaining on education. If he saves Rs 10800 then find his expenditure on food?
100 + 10% 90 20% 72 +25% 54
10 18 18

54 = 10800
18 = ? 3600

24. Rajesh salary is 10% more than Prakash salary, Prakash salary is 10% less than 10
Lokesh salary. If sum of their salaries is 8670. Then find 10 lokesh salary?
R P L 289 = 8670
99 90 100 100 = ?
3000

25. In an election among 3 persons A, B and C, A got 30% of the total votes, B got 60% of
the remaining votes. If ‘C’ got 14,000 votes by how many votes majority has the
winner won election?
A B C 28 = 14000
30 42 28 12 = ? 6000

26. 605 Sweets were distributed equally among children in such a way that the number
of sweets received by each child is 20% of the total number of children. How many
sweets did each child receive?
100x (100 x )×( 20x) = 605
2000 x2 = 600
x = 605/2000 = 605/(100 × 20) = 11/20
= 100 = 100 × 11/20 = 55
20% of 55 = 1/5 × 55 = 11

27. In an exam a student secure 60% of the total marks gets 40 marks more than pass
marks another student secures 40% of the total marks and pass marks of the exam.
20 = 60 60% of 300 = 180
100 = 300 m pass marks = 180 – 40 = 140 m

19
28. A man spends 60% of his income. His income is increased by 10% and he increased
his expenditure by 18%. By what percentage his saving increased (or) decreased?
I E S
100 60 40
10% 18% 0.8/40 × 100 2%

110 70.8 39.2

29. The expenses on rice, fish and oil of a family are in the ratio 8 : 5 : 3. The prices of
these articles are increased by 25%, 20%, 33 1/3% respectively. The total expenses of
the family on these articles are increased by?
R F O
8 5 3 (16p) 4/16 × 100 = 25%
25% 20% 33 1/3%

10 : 6: 4 (20p)

30. There are 3 numbers 5/7 of the first number is equal to 48% of the second number.
The second number is 1/9th of the 3rd number. If the 3rd number is 1125. Find the
25% of the first number?
x y z y = 1/9 z
5/7 × x = 48% of y , y = 1125/9 = 125
5/7 × = 48/100 × 125
x = 84 25% of 84 = 21

31. Suresh and Ramesh saves 20% and 40% of their respective incomes. If the
expenditures are equal, then what is the ratio of Ramesh salary to Suresh salary?
S R
I 100x 100y 80x = 60y
S 20x 40y y/x = 4/3 4:3
E 80x = 60y

32. Fresh fruit contains 68% water and dry fruit contains 20% water. How much dry fruit
can be obtained from 100 kg of fresh fruits?
W P 80% = 32 k.g
F 68 32 (100k.g) 100 = ? 40 kg
P 20% 80%

33. Fresh grapes contain 80% water while dry grapes contain 10% water. If the weight of
dry grapes is 250 kg, what was it’s total weight when it was fresh? (Pulp)
W G 100% = 250 k.g
F 80% 20% 90% = 225%
D 10% 90% (225) 20% = 225
100y = ? 1125 k.g

20
17

25 35

23%

36. The price of wheat is decreased by 25% by what percentage must as consumption be
increased. If the expenditure on its increased by 5%?
P C
10 × 10 = 100 (e)

7.5 × (?) = 105


X = 105/75 = 105/75 × 10 = 14 40%

37. Of the 1000 inhabitants of town, 60% are males of whom 20% are literate. If, of all
the inhabitants, 25% are literate, then what percent of the females of the town are
literate?

1000(T)

60%

(M) 600 400(f) 130/400 × 100 =


= 37.5%

480(il) 120(l) 130(l)

250(l)
PROFIT AND LOSS
COST PRICE:- The price at which an article is purchased, is called it’s cost price.

SELLING PRICE:- The price at which an article is sold, is called it’s selling price.

PROFIT & GAIN:- If selling price is more than cost price, the we can say it is profit.
SP > CP
Profit = SP > CP
Profit % = (SP > CP)/CP × 100 = P/CP × 100

P% = P/CP × 100

LOSS %:- If cost price is more than selling price, then we get a loss.
CP > SP
Loss = CP – SP
Loss % = (CP – SP)/CP × 100

Loss % = L/CP × 100

Profit or Loss is always calculated on CP. So that in P & L problems CP is taken 100.
Example:- 20% profit 25% loss
CP = 100 CP = 100
P = 20 L = 25
SP = 120 SP = 75
If cost price = Selling price (No profit – no loss) it is term is called Breakeven.

1. A person purchased a table for Rs. 1600 find his profit (or) loss percentage, he sold
the article.
a. 1680/- 80/1600 × 100 = 5% profit.
b. 1440/- 160/1440 × 100 = 10% loss.

2. A person purchased an article for 480/- at what price should he sold the article?
a. To gain 12.5% profit 540/-
b. To get a loss of 16 2/3% loss 480 – 80 = 400/-

3. A person sold an article for 750/- find cost price of the article?
a. If he gain 25% 600/-
b. If he loss 25% 1000/-

4. By selling an article for 360/- a man get a loss of 10% if to gain 10% profit at what
price should he sold this articles?
90 = 360
110 = ? 440/-

22
5. A man sold a book at 15% loss. If he had sold it for 75/- more than he gains a profit
of 10%. Then find cost price of book?
85 - 110
25 = 75
100 = ? 300/-

6. A vendor sells lemons at 5 for a rupee gaining 40%, how many did he buy for a
rupee?
5 L - 100 p
1 L - 20 p
140 = 20p
100 = ? (100 × 20)/140 =100/7
100/7 p 1
100p 7L

7. By selling toffees at 20 for a rupee, a man loses 4%. In order to gain 20% how many
for a rupee must he sell?
20 - 100p 96 = 5p
1 - 5p 120 = ? (120 × 5)/5 = 50/8
50/8 p – 1
100 p - ? (100 × 8)/50 = 16

8. Mr. Peter purchased a machine for Rs 80,000 and spent Rs. 5000 on repair and
5000/- for transportation. At what price should he sell it in order to earn a profit of
18%?
CP = 80,000 + 5000 + 5000 90,000
100 = 90,000
118 = ? 1,06,200/-

9. The profit earned on selling an article 1680 is twice the loss incurred on selling the
article for Rs. 1200. What is the cost price of the article?
3p = 480 p = 2L P/L= 2/1
1p = 160
1200 + 160 = 1360/-

10. If the selling price of 10 articles is the same as the cost price of 12 articles, find the
gain percentage?
SP of 10A = CP of 12A
SP/CP = 12/10 = 6/5 1/5 × 100 = 20%

11. By selling 100 pens, a shop keeper gains the selling price of 20 pens. Find his gain
percentage.
SP P CP
SP of 100p SP of 20p CP of 100p
P = SP – CP SP of 20P = SP of 100p – CP of 100p
CP of 100p = SP of 80p
23
CP/SP = 80/100 = 4/5 1/4 × 100 =25%
12. Bananas are bought at 11 for Rs 10 and sold at 10 for Rs.11. Find gain percentage?
CP 11 10 100
SP 10 11 121 21%

13. The price of sugar is reduced by 20% a person enables to get 5 k.g more for 200/-.
Then find new price and original price of sugar?
X 0.8x
200/0.8x – 200/x = 5 2000/8x – 200/x 5 2000 – 1600 = 40x
40x = 400 x = 10/-
Consumption is 25%
25 = 5 kg 25kg - 200 8/-
125 = ? 25kg 20kg - 200 10/-

14. A dishonest dealer to sell his good at cost price but he is uses a weight of 800grams
instead of 1kg. Find his profit percentage & error percentage?
CP SP
800 1000 200/800 × 100 = 25%
CP SP
1000 800 200/1000 × 100 = 20%

15. A grocer sells rice at a profit of 10% and uses weight which are 20% less than the
market weight. The total gain earned by him will be?
800 1000 profit = 25% and 10%
10%
100 --------- 110 135.5% overall = 37.5%
+8
16. A fair price shopkeeper takes 10% profit on his goods. He last 20% goods during
theft. His loss percentage?
100 -20% 80 10% 88 12% loss
+8
17. A cloth merchant sold half of his cloth at 20% profit, half of the remaining at 20% loss
and rest was sold at the cost price, in the total transaction, his gain or loss is?
100
50 20% +10
25 20% -5
25 0 0/+5 5% profit

24
18. A trader by mistake calculated his profit on SP and it is 20% then find his original
profit.
SP = 100 CP = 80 20/80 × 100 = 25%
19. A trader by mistake calculated his loss on SP and it is 25%. Then find his actual loss
on percentage?
SP = 100 CP = 125 25/125 × 100 = 20%

20. A sells a bicycle to B at a profit of 20% B sells it to C at a profit of 25%. If C pays Rs.
528 for it. The cost price of the bicycle for A is?
A B C 132 = 528
100 120 132 100 = ?
400/-
CP’s are equal (2) x% P, x% L (no profit no loss)
X% P, y% L (x – y)/2
SP’s are equal (2) x% P, x% L loss = x2/100
X% P, y% L {100(x – y) – 2xy}/{(100 + x) + (100 Y)}
21. A man purchased two articles and sold them Rs. 2000 each on one he gains 5% and
on the other, he losses 5%. His gain (or) loss percentage in the whole transaction is?
52/100 = 0.25% loss

22. A man sells two commodities for Rs. 4000 each, heather losing nor gaining in the
deal. Of he sold one commodity at a gain of 25%, find the loss percentage of other
commodity?
{100 (x – Y) – 2xy}/{(100 + x) + (100 – y)} = 0 100(x – y) = 2xy
100 (25 – y) 2 × 25 × y = 50 – 2y = y 3y = 50
Y = 16 2/3 %

DISCOUNT:-
Marked Price / Label Price / Listed Price:- The price at which the article is marked is
called marked price.
Discount:- It is a reduced amount from the marked price before selling.
Discount = MP – SP
d% = (MP – SP)/MP × 100 = d/MP × 100
20%
CP/100 MP/120

SP/108

23. A shopkeeper marks his goods 30% above on cost price and allows a discount of 20%
on marked price. Then find his profit (or) loss percentage.
CP MP SP
100 30% 130 20% 104 4% profit

25
24. A trader marks his goods 20% above on the cost price and offers a certain discount
on marked price and net gets a profit of 2%. Then find discount percentage?
CP MP SP
100 120 102 18/120 × 100 = 15%

25. A shopkeeper marks his goods a certain percentage on cost price and offers a
discount 20% on marked price and get a profit of 8%. Find his mark-up percentage?
M × D = SP
? × 8 = 108 mark up %= 108/8 = 13.5 = 35%

26. A shopkeeper sold a TV set for Rs. 24,000 with a discount of 20% and earned a profit
of 20%. What would have been the percentage of profit earned if no discount was
offered?
No discount means profit % = mark up %
M × D = SP
? × 8 = 120 mark up = 15 = 50%
27. The price of a VCR is marked at Rs. 12,000. If successive discounts of 15%, 10% and
5% be allowed. Then at what price does a customer buy it?
8.5 × 9 × 9.5 = 726.75
1000 = 12,000
726.75 = ? 8721
28. After allowing two successive discount a marked price of shirt Rs. 105. If the first
discount is 20%. Then find second discount percentage?
150
-30 20% second discount = 15/120 × 100 = 12.5%
120

105

26
BLOOD RELATION
• Parent’s father – Grand father
• Parent’s mother – Grand mother
• Parent’s brother – uncle
• Parent’s sister – Aunt
• Parent’s sibling’s child – Cousin
• Sibling’s son – Nephew
• Sibling’s daughter -- Niece
• Spouse’s father – Father – in - law
• Spouse’s mother – Mother – in - law
• Spouse’s brother – Brother – in - law
• Spouse’s sister – Sister – in - law
• Sister’s husband – Brother – in –law
• Brother’s wife -- Sister – in - law
• Daughter’s husband – Son – in –law
• Son’s wife – Daughter – in –law
NOTE
• Father side – Paternal
• Mother side – Maternal
• Siblings – Children of same parents
FAMILY TREE – SYMBOLS

↑ -- Above generation

↓ -- Below generation
--- -- Same generation

⇌ -- Couples (Spouse)
_ Male person
Female person

27
• A is Father of B

A

B
• A is brother of B
B --- A

• A is wife of B
B
⇌ A
• A is son of B
B

• A is grandfather of B
A

Sample problem

P C ⇌ F

M A
⇌ B
K U

N Z
P S T

28
1. A S either son or daughter
2. N M Aunt
3. F N Grandson
4. C S either grandson or grand daughter
5. K A Brother – in – law
6. B M sister – in – law
7. A K either brother – in – law or sister – in – law
8. A F Father – in – law
9. A O Niece
10. N P Grand aunt (Maternal)

PROBLEMS
1. In a family of six persons A, B, C, D, E and F they are two
married couples. D is grandmother of A and mother of B. C is
wife of B and mother of F. F is granddaughter of E
a. How is A related to F
b. How is C related to D D ⇌ E

c. A is E’s
Answers

a. Sister or brother B
⇌ C

b. Daughter – in – law
c. Grandson or granddaughter A
F

2. A is mother of B. B is sister of C. D is son of C. E is brother of


D. F is mother of E. G is daughter of B. H has only two children
B and C
a. How F related to H
b. How C related to E
c. How B related to F
29
Answers

a. Daughter - in – law A ⇌ H

b. Father
c. Sister – in - law

B C
⇌ F

G D E

3. A is father of C but C is not his son. E is the daughter of C. F is


the spouse of A. B is brother of C. D is son of B. G is spouse of
B.H is father of G. A ⇌ F H

a. Who is grandmother of D
b. Who is son of F

Answers C B ⇌ G

a. F
b. B
E D

4. In a family there are six members A, B, C, D, E and F. A and B


are married couples, A being a male member D is the only son
of C, who is the brother of A. E is sister of D. B is daughter – in-
law of F, whose husband has died. How many male members
are there?

Answer : 3 F ⇌ X

C A
⇌ B

D 30 E
5. There are 7 family members P, Q, R, S, T, U and V. In which
they two married couples. P is sister of Q, who is maternal
grandson of T. The father of S has two maternal grandchildren.
R is Sister – in –law of V, who is father of P. U is a female. Who
is husband of U?
Answer : T U ⇌ T

V ⇌ S R

P Q

6. Salesman, engineer and accountant. There are two married


couples in a family. D, the salesman married to lady teacher.
The doctor is married to the lawyer. F, the accountant is the son
of B and brother of E. C, the lawyer is daughter – in –law of A. E
is unmarried engineer. A is the grandmother of F. what is
profession of B?
Answer D
⇌ A lady teacher
Doctor sales man

B ⇌ C
Doctor lawyer

F E

Accountant (Engineer)

31
Equation based questions
P Q
7. A * B A is sister of B
A + B A is mother of B
A / B A is father of B
A - B A is brother of B R
S

a. P * Q /R –S +T then P is T’s ?

Short cut 2 T

P × Q/R–S+T

0+ 1+ 0+ 1=2
b. Which of the following P is aunt of T?
1. P+Q-R*S/T × 1+0+0+1=2
1 0 01
2. P-Q+R/S*T ×

3. P*Q/R*S-T √ 0+1+0+1=1
0 1 0 0

4. P/Q*R+S-T ×

c. P*Q-R/S-T*U P is U’s
a. Father b. Mother c. Aunt d. Maternal aunt
e. Grand Mother
Shortcut P Q R

P ×Q–R/S–T×U

S U
T
0 0 1 0 0=1

32
8. P $ Q P is father of Q
P *Q P is daughter of Q
P@Q P is sister of Q

a. A@B$C*D how B related to D?


a. Husband b. Uncle c. Father d. Grand father e. Aunt
A@ B &C×D

1 -1 1–1=0

Pointing to a person
9. Pointing to a women in a photograph a man says “she is the
mother of the father of my wife’s only son”. How is the women
related to man?

Man ⇌ mother

10. Pointing to a joker, madhu says “He is the son of my father


brother only sister – in –law”. How joker is related to madhu?

Madhu Brother

11. Pointing to a women in a photograph a man says “she is the


mother of my daughter- in – law’s husband only sister”. How
women related to man?

33
⇌ man wife

12. Pointing to a man in a photograph a women says “he is the


father of my only daughter – in –law’s father – in –law. How
man is related to women?
Father-in-law

Women ⇌


13. Pointing to a man in a photograph a women says “he is the
only son of the only daughter – in –law of my only son’s father”.
How man is related to women?
Women ⇌ grand son

14. Pointing to a women in a photograph a man says “she is the


sister- in –law of the only daughter of my mother- in –law’s
husband”. How women is related to man?

34
⇌ sister

Man ⇌
15. Pointing to a man in a photograph a women says “he is the
father – in –law of the wife of the only grandson of my own
father- in- law”. How man related to women?

women ⇌ Husband


16. Pointing to a lady Arun said “she is the mother of my
brother’s son’s wife”. How lady related to brother’s son?
Arun mother-in-law

17. Pointing to a gentleman, shalini says “ his only brother is the


father of my son’s father”. How gentle man related to shalini?

Uncle

Shalini ⇌

35
CALENDAR
ODD DAYS:- In a given period, the number of days more than the complete weeks are called
odd days.
Per example, January = 31 days = 4 weeks + 3 days (extra)
February = 28 days = 4 weeks (Here odd day = 0)
In leap year, February = 29 days = 4 weeks + 1 day (extra)
Likewise April =30 days = 4 weeks + 2 days (extra)
Hence we can notes that odd day can be obtained by dividing a number by 7 and
considering only the REMAINDER.

LEAP YEAR:-
Every year divisible by 4 is a leap year, if it is not a century.
Every 4th century is a leap year and no other century is a leap year.
NOTE:- A leap year has 366 days.

Examples:-

• Each of the years 1948, 2004, 1676 etc. is a leap year.


• Each of the years 400, 800, 1200, 1600, 2000, etc. is a leap year.
• None of the years 2001, 2002, 2003, 2005, 1800, 2100 is a leap year.

ORDINARY YEAR:-
The year which is not a leap year is called an ordinary year. An ordinary year has 365
days.

COUNTING OF ODD DAYS:-

• 1 Ordinary year = 365 days = (52 weeks + 1 day).


1 ordinary year has 1 odd day.
• 1 leap year = 366 days = (52 weeks + 2 days).
1 leap year has 2 odd days
• 100 years = 76 ordinary years + 24 leap years.
= (76 × 1 + 24 × 2) odd days = 124 odd days.
= (17 weeks + 5 days) = 5 odd days
Number of odd days in 100 years = 5

36
Number of odd days in 200 years = (5 × 2) = 3 odd days.
Number of odd days in 300 years = (5 × 3) = 1 odd days.
Number of odd days in 400 years = (5 × 4 + 1) = 0 odd days.
Similarly, each one of 800 year, 1200 years, 1600 years, 2000 years etc. has 0 odd
day.

CENTURY CODE
1600 - 6 1200 - 6 800 - 6
1700 - 4 1300 - 4 900 - 4
1800 - 2 1400 - 2 1000 - 2
1900 - 0 1500 - 0 1200 - 0

MONTH CODE
Jan - 0
Feb - 3
Mar - 3
Apr - 6
May - 1
Jun - 4
Jul - 6
Aug - 2
Sep - 5
Oct - 0
Nov - 3
Dec - 5

37
DAYS CODE
Sun - 0
Mon - 1
Tue - 2
Wed - 3
Thu - 4
Fri - 5
Sat - 6

1. What was the day of the week on 15th August, 1947?

1947 1900 + 47 0 + 47 + 11 = 2 odd days


Aug =2
15 = 1
5 Friday
2. What was the day of the week on 1st Jan 2016?

2016 2000 + 16 6 + 16 + 4 = 26 = 5 odd days


Jan =0
1 =1
6 – 1 = 5 Friday

NOTE:- Given year is leap year and month is Jan (or) for then subtract ‘1’ from final answer.
3. On what dates of Nov 2013 dis Sunday fall?

2013 2000 + 13 6 + 13 + 3 = 1 odd day


Nov =3
1 =1
=5 Friday
1st Nov 2013 Friday
2nd Sat
3rd Sun
3rd , 10th, 17th, 24th.

38
4. On what dates of Jan 2012 did Wednesday fall?

2012 2000 + 12 6 + 12 + 3 = 0 odd days


Jun =0
1 =1
1
(1 – 1 = 0 Sunday)
1st Jan 2012 Sunday
2nd Mon
3rd Tuesday
4th Wed
4th,11th,18th, 25th.

5. January 1, 2007 was Monday. What day of the week lies on Jan1, 2008?

Jan 1, 2007 Mon


+1
Jan 1, 2008 Tue

6. On 8th Dec, 2007 Saturday falls. What day of the week was it on 8th Dec 8, 2006?

8 Dec, 2006 Friday


-1
8 Dec, 2007 Sat

7. It was Sunday on Jan 1, 2006. Find the day of the week on Jan 1, 2010?

Jan 1 2006 Sun


+1
Jan 1 2007 Mon
+1
Jan 1 2008 Tue
+2
Jan 1 2009 Thu
+1
Jan 1 2010 Fri

39
SHOTCUT – TO HAVE SAME CALENDAR
Leap year = +28
Leap year+1 year = + 6
Leap year+2 years = + 11
Leap year+3years = + 11

8. Which year subsequent to 1993 had the same calendar as that of the year 1993?
1993 + 6 = 1999

9. Which year subsequent to 1990 had the same calendar as that of the year 1990?
1990 + 11 = 2001

10. Which year subsequent to 1984 had the same calendar as that of the year 1984?

1984 + 28 = 2012

11. Gita 314 days older than Suman. Sapna 70 days older than Gita. If Supna was
born on Thursday then find the birth day of Suman?

Sapna > Gita > Suman


70 + 314 = 384
384/7 = 6
Thursday + 6 = Wednesday.

40
CLOCKS
A clock is just like a complete circle having 360°. It has been divided into 12 equal parts i.e.
each part is 360/12 = 30°

• The clock has two hands – smaller hand and bigger hand.
• The smaller hand is called Hour hand and the bigger hand is called Minute hand

Relation between Minute hand and Hour hand

• As minute hand takes a complete round in one hour i.e. 360°, so you can that it
covers 360° in 60 min. in 1 min. it covers 360/60 = 6°
• As hour hand covers just one part out of the given 12 parts in one hour, this implies
it covers 30° in 60 min. in 1 min. it covers 30/60 = 1/2°

Time Minute Hand Hour hand

1 Hour 360° 30°

1 Min 6° 1/2°

Relative speed
Relative speed of the minute hand and hour hand is 6 - 1/2 = 5 1/2° (11/2°) in one minute.

Important points

• Min. & Hr. hand coincide (0°) once every hr., but in a 12 hr. period they coincide 11
times. It is also known as hands are in a straight line in the same direction.
• In every hour, there is one angle of 180° between the hands of the clock, but in 12
hr. period there are 11 such angles. It is also known as hands are in a straight line in
the opposite direction.
• In every hr. there are two right angles (90°). But in 12 hr. period there are 22 such
angles.
• Any angle between 0 & 180° is formed twice in every hour. But in 12 hr. period there
are 22 such angles

41
Θ 1 Hour 12 Hours 12 Hours

0°,180° 1 11 22

0°< θ <180° 2 22 44

Straight line (0°) 1 11 22

Straight line (180°) 1 11 22

Straight line (180°) 1 11 22


but not 0°

Straight line (0°) but 1 11 22


not 180°

Straight line 2 22 44
(0°,180°) both
conditions

Type-1
1. what is the angle covered by the minute hand in the time in which the hour hand
covers 19°

H M
Speed 1/2° 6°
Ratio 1 : 12
| |
19° ? i.e.12 * 19 = 228°

42
2. what is the angle covered by the hour hand in the time in which the minute hand
covers 312°

H M
Speed 1/2° 6°
Ratio 1 : 12
| |
? 312° i.e. 312/26 = 26°

Type-2 : concept of mirror image


To find the mirror image of any given time, subtract the value from 11:60
i.e. Given Time + Mirror Image Time = 11:60

3. Find the mirror image of 3:20 on a wall clock?

11:60 - 3:20 = 8:40

4. Find the mirror image of 11:10 on a wall clock?

11:60 – 11:10 = 0:50 i.e. 12:50

Type -3: Angle between the Hour hand and Minute hand

θ = |30 H – 11/2 M|
where θ = angle b/w two hands,
M = minutes and H = hours

Note:

• When the two hands are coinciding or overlapping, means θ = 0°


• When the two hands are at right angles or perpendicular, means θ = 90°
• When the hands are in opposite directions, means θ = 180°

43
5. Calculate the angle between the hour hand and the minute hand when the clock
reads 10:10

θ = |30 H – 11/2 M|
θ = |30 (10) – 11/2 (10)|
θ = |300 – 55|
θ = 245
θ = 360 – 245 = 115°
6. The reflex angle between the hands of a clock at 10:25 is

θ = |30 H – 11/2 M|
θ = |30 (10) – 11/2 (25)|
θ = |300 – 137.5|
θ = 162.5
θ = 360 – 162.5
θ = 197.5

Note
• Angle is always less than or equal to 180°
• Reflex angle is greater than 180°
Type 4:

7. At what time between 7 and 8 the hands of a clock coincide with each other?

θ = 30 H – 11/2 M
0 = 30(7) – 11/2M
11/2 M = 210
M = 420/11
M = 38 2/11 min.
i.e. 7: 38 2/11 min.

8. At what time between 7 and 8 the hands of a clock be in a straight line in the
opposite direction?

θ = 30 H – 11/2 M
180 = 30(7) – 11/2M
11/2 M = 210 – 180 = 30

44
M = 60/11
M = 5 5/11 min.
i.e. 7: 5 5/11 min.

9. At what time between 7 and 8 the hands of a clock be at right angles?

Case 1 case 2

θ = 30 H – 11/2 M θ = 30 H – 11/2 M
+90 = 30(7) – 11/2M - 90 = 30(7) – 11/2M
11/2 M = 210 – 90 = 120 11/2 M = 210 + 90 = 300
M = 240/11 M = 600/11
M = 21 9/11 min. M = 54 6/11 min.
i.e. 7: 21 9/11 min. i.e. 7: 54 6/11 min.

Type 5

10. A clock takes 10 seconds to strike 6, how many seconds does it take to strike 10

. . . . .

5 int ---- 10 sec


1 int ---- 2 sec
9 int ---- (9 * 2 ) = 18 sec

11. A gong clock strikes with an interval of 1 sec. find in how much time it will strike the
whole day ?

Between 2 gongs ----- 1 sec

Intervals
1 o’ clock 0
2 1

45
3 2
.
.
.
12 11
(1+2+3+…….+11) = (11 * 12)/2 = 66
In a day = 66* 2 = 132

12. A watch which gains uniformly, was observed to be 6 minutes slow at 6 p. m. on


Monday, it was noticed 12 minutes fast at 6 a. m. the following Monday morning
when did the watch show the correct time ?

156 hr --- 18 min


52 hr --- ? (52/156)*18= 6 min
Monday 6:00 pm + 52 hr
6:00 pm + 2 days 4 hr
Wednesday, 10 p.m.

13. How much does a watch lose or gain if its hands coincide every 64 minutes?

65 5/11 – 64 = 1 5/11 = 16/11


16/11 min gain for every 64 min

64 – 16/11
1- ? (16/11)* 1/64
Gain in 24 hrs = (16/11)*(1/64)*24*60 = 32 8 /11 min

46
TIME & WORK
NOTE:-
a. If a work is completed in N days, then how much part of the work is completed in one
day?
b. If one day is 1/N, then total work is completed in how many days?
Total work completed in N days.
1. A student can write a record in 6 hours what part of the record he can write in 72
minutes?
Total record can be completed in 6h.
6 hours = 6 × 60 = 360 min
1 min part = 1/360
72 min = 72/360 = 1/5 part
So that men’s increases then number of hours decreases men’s inversely
proportional to hours.
M ∞ 1/H
M1H1 = M2H2
Relation between men and work
Ex 1. : - 10 men can prepare 10 toys – 10 days
20 men can prepare toys? – 5days.
So that men’s increases then work also increased men’s are directly proportional to
work
M∞W
M1 /W1 = M2/W2
Conclusion: M ∞ W/DH
MDH/W = constant
M1D1H1/W1 = M2D2H2/W2

M1D1H1W2 = M2D2H2W1
2. 30 men can do a work in 18 days working 7 hours per day. In how many days 21
men can do the same work, work in 8 hrs. per day?
30 × 18 × 7 = 21 × D2 × 8
D2 = (30 × 18 × 7)/(21 × 8)
D2 = 45/2 days.

47
3. 5 men can prepare 10 toys in 6 days working 6 hours per day. In how many days 12
men can prepare 16 toys working 8 hours per day?
5 × 6 × 6 × 6 = 12 × D2 × 8 × 10
D2 = (5 × 6 × 6 × 6)/(12 × 8 × 10) = 3 days
4. 40 men can cut 60 trees in 8 hours. If 8 men leave the job, then how many trees will
be cut in12 hours?
40 × W2 × 8 = 32 × 12 × 60
W2 = (32 × 12 × 60)/(40 × 8) = 72 trees

5. A certain number of men do a work in 60 days. If there 8 men more, it will be


finished 10 days less. Then find how many men are initially?
M × 60 = (m + 8)50
6m = 5m + 40
m = 40 men

6. 15 men can do a work in 20 days. If some more men joined, then they completed
1/3 work in 4 days. How many more men joined?
15 × 20 × 1/3 = (15 + x ) × 4 × 1
25 = 15 + x
x = 10 men

7. There is a sufficient food for 400 men for 31 days, after 28 days 280 men leave the
place. How many days the remaining food will be sufficient for the rest of the men?
400 men – 31 days
After 28 days, 400 men – days
120 men - ?
400 × 3 = 120 × d2
D2 = 10 days

8. A contractor employee the 120 men to complete a project in 15 days. But after 5
days he was noticed that only 1/4th of work is completed, then how many more men
are required to complete the project on men?

¼ ¾
120 men – 15 days
? - 10 days
m1d1 = m2d2
m1 = 120, d1 = 15
m2 = ?, d2 = 10
120 × 15 = m2 × 10
m2 = 180
m1 = 120
Required = m2 – m1 = 60

48
9. 6 men or 8 women can do a work in 42 days. In how many days 9 men and 2 women
Working together can do the same work?
6 men or 8 women – 42 days
6 men = 8 women – 42 days
9 men + 2 women –?
6m = 8 women
m = 8/6 women = 4/3 women
9m + 2 women = 9 × 4/3 women + 2women
= 12 women + 2 women = 14 women
8 women – 42 days
8 × 42 = 14 × d2
D2 = 24 days

10. 1 men or 2 women or 3 boys can do a work in 36 days. In how many days 1 men and
y women and 9 boy can complete the work?
1 men = 2 w = 38
1m + 1/2 × 4m + 9 × 1/3
6m
1m – 36 days
6m - ?
1 × 36 = 6 × d
D = 6 days

11. 3 men and 6 women can do a piece of work in 12 days. 5 men and 6 women can do
the same work in 9 days. In how many days 3 men and 3 women can do the same
work?
3m+6w 12
3m+6w 9
3m+3w ?
(3 m + 6 w) × 12 = (5 m + 6 w) × 9
12 m + 24 w = 15 m + 18 w
6w=3m
m=2w
3(2 w) + 6 w 12
6w+6w 12
12 w 12
6w+3w ?
9w ?
12 × 12 = 9 × d2
d2 = 16 days

12. 6 men & 9 women can do a piece of work in 4 days, 4 men and 3 women can do the
some work in 8 days. In how many days 20 men and 6 women can do the same
work?

49
6m+9w=4
4m+3w=8
20 m + 6 w = ?
(6 m + 9 w) × 4 = (4 m + 3 w) × 8
6m+9w=8m+6w
3w=2m
m = 3/2 w

13. A can do a work in 12 days and B can do the same work in 24 days. If they work
together, in how many days the work will be complete?
LCM 12, 24 24
a/12 b/24 a + b 8 days

2p 1p 3p

24

14. A 10 days, B 20 days, C 60 days


Then A + B + C =?
LCM 10, 20, 60 60
A/10 B/20 C/60 (A + B + C)/6 days

6 3 1 10

60
6 + 3 + 1 = 10
15. A and B can do a work in 8 days and A alone can do the same work in 10 days. Then
in how many days B can complete the work?
LCM 8, 10 40

(A + B)/8 A/10 B/40 days

5p 4p 1p

4 5p – 4p = 1p

50
16. A and B together can do the work in 24 days. B and C together can do the same
work in 40 days. C and A together in 30 days. Then find how many days the work will
be completed with A, B and C?
A+B 24 d
B+C 40 d
C+A 30 d
LCM 24, 40, 30 120
2(A + B + C) (A + B)/24 (B + C)/40 (C + A)/30 (A + B + C)/20days
10 days

12 p 5p 3p 4p 6p
3 A(6 – 3) = 3

2 B(6 – 4) = 2
120 1
120 C(6 – 5) = 1

17. A and B can complete a work in 10 days and 20 days respectively. If A work for 5
days and then leaves, in how many days the total work can be completed?

A/10 B/20
2p 1p

20

5 days 10p
A B 10 days
2(5) = 10p
Remaining work = 10 days
Total work = 10 + 5 = 15 days
18. A can do a work in 18 days. B can do the same work in 9 days. If A and B work for 2
days, then B leaves. Find in how many days the entire work can be completed?
LCM 18, 9, 2 18
A B
A/18 B/9 (A + B)/2
A + B = 3P
2(A + B) = 6P
1 14 days

51
19. A can do a work in 15 days. B can do the same work in25 days. If B works 9 days and
then A joined. In how many days the total work can be completed.
LCM 15, 25 75 9 × 3 = 27
A/15 B/25 (A + B) 75 – 25 = 48
5 3 8p A + B = 48 8p = 48
8p = 48 9 + 6 =15
7

ALNERNATIVE DAYS

20. A and B can do a work in 10 days and 20 days and respectively. If they work on
alternatively days, starting with A, in how many days the work can be completed
and by whom?

A/10 B/20 (A + B) 1 2
1p 3p A B
2p 2p 1p
2 1cycle = 2d 6 cycle = 12d
1cycle = 3p 6 cycle = 18 p
Remain (2p) = 13th 13th
A B
2p 1p

EFFICIENCY (Efficiency – one day part ratio)


21. A is twice as fast as B. It they work together the work can be completed in 14 days.
In how many days B alone can complete the work?
A = 2B A:B=2:1 Efficiency ratio
(A + B) = 14 A/21 B/42
3 2
1
4

22. A can do a work 3 times as fast as B. B can work half as fast as C. If all of them
together can complete the work in 10 days. In how many days B alone can complete
the work?
A = 3B B=½C C=2 B=1 A=3
A B(60) C A+B+C
1 2
3 6
6

52
23. A can do a work in 12 days. If he joined by B who is 50% more efficient than A. In
how many days the work can be completed?
A B A+B
100 150 24/5
2 : 3

WAGES
Wages ratio = 1 day part ratio
24. A and B can do a work in 50 days and 40 days respectively for doing the work they
get an amount of Rs. 3600. Find the shares of A and B.
A/50 B/40 9p = 3600/-
4p = ?
4 : 5 p = 1600/-
B = 2000/-
200 A = 1600/-

25. A and B can do a work in 12, 24 days respectively. If a started the work after 6 days
B joined him then they complete the work. How much amount should A take as his
share out of total Rs. 180?
A/12 B/24 24
A 12 p A + B 4 days
A : B 6p = 180
24
20 : 4 5p = ?
5:1 150/-
26. A can do a work in 24 days, with the help of B. he completed the work in 16 days for
doing this work they got an amount of Rs. 360, then find the share B.
A/24 A + B – 16 days
Per day = 360/25 = 15/- 16 × 15 = 240/- A
= 120/- B

53
PIPES & CISTRENS
27. Pipe A fills an empty tank in 14 hours, another pipe B empties the full tank in 21
hours. If both pipes are opened together, in how many hours’ time the tank is fully.
Tank becomes empty in 60 minutes.
28. A cistern is normally filled in 8 hours but it takes 2 hours longer due to leakage at
the bottom. If the cistern is full in what time the leak can empty the full tank?
40 hours.

29. Two pipes A and B can fill a tank in 24 hours and 32 hours respectively. If both pipes
are opened together then after how much time the pipe B should be closed so that
the tank is full in 18 hours?
A/24 B/32 A+B B 8 hours
3
4 18 hrs

96

30. A and B together can do a piece of work in 30 days. A having worked for 16 days, B
finishes the remaining work alone in 44 days. In how many days shall B finish the
whole work alone?
A + B = 30
Let total work = 30 parts
A B A+B
30 1part
16 days of A + 44 days of B = 30
16 days of A + 16 days of B + 28 days of B = 30
16 (A + B) + 28 days of B = 30
16(1) + 28 days of B = 30
28 days of B = 14
B = 14/28 = 1/2 = 0.5
So, total work 30 parts ‘B’ can be done in 60 days [30/0.5 = 60]

31. Two pipes can fill a tank with water in 15 hr & 12 hr respectively and a third pipe can
empty it is 4 hour. If the pipe opened in order at 8, 9 and 11 A.M mespectively, At
what time the tank will be emptied?
A/15 B/12 C/4
5 -15 Pipe ‘A’ full in ‘3’ hours = 4 × 3 = 12 (8 to 11 a.m)
4 pipe ‘B’ fill in ‘2’ hours = 5 × 2 = 10/22 (9 to 11
a.m) At 11 a.m three pipes are opened i.e.
60
resultant =
= 4 + 5 – 15 = -6
22 lt can be empty in pipe ‘C’ = 22/6 = 34/6 = 32/3 × 60 = 3 hr 40 min

54
11 + 3 hr 40 min = 2 : 40pm.
32. If A, B and C working together can complete a piece of work in 40 days. They
together work for 16 days and A leaves the remaining work is completed by B and C
in 40 days. How much time it would take for ‘A’ alone to complete the whole work?
(A + B + C)/40d 1p/day
(A + B + C) worked for 16 days = 16(1) = 16 parts completed
(B + C) Completed remaining work (24) = 40
(B + C) 1 day work = 24/40 = 0.6 par day
A+B+C=1
A + 0.6 = 1
A = 0.4
‘A’ alone can complete the whole work 40/0.4 = 100 days.
33. A and B together can do a piece of work in 12 days, which B and C together can do in 16
days. After A has been working at it for 5 days and B for 7 days, C finishes it in 13 days. In
how many days C alone will do the work? (GATE)
A+B B+C
12 16 total work = 48 units (LCM of 16 and
12)

4 3

5A+7B+13C = 48
5(A+B) +2(B+C) +11C = 48
5(4) +2(3) +11C = 48
20+6+11C = 48
11C = 22
C = 2units/day
So 48 units C can be completed in 24 days

34. 5 skilled workers can build a wall in 20 days, 8 semi-skilled workers can build a
wall in 25 days, and 10 unskilled workers can be build a wall in 30 days. If a team has
2 skilled, 6 semi-skilled and 5 unskilled workers, how long will it take to build the
wall?
A – Skill 5A – 20 A – 100
B – Semi skill 8B – 25 B – 200
C – Unskilled 10C – 30 C – 300

A B C 2A+6B+5C
100 200 300 total work 600 units
6 3 2 12+18+10 = 40 units in 1 day
So 600 units can be completed in 15 days

55
TIME SPEED AND DISTANCE

Distance = speed × Time


Speed =
Time =

1. When speed is constant


Distance ∞ Time

2. When Time is constant


Distance ∞ Speed

3. When distance is constant

Speed ∞

S1T1 = S2T2
Units
Speed Distance Time
Kmph km hrs
Mps m sec

To change km/hr in m/s we multiply by 5/18


e.g. 72 k/h means
72 × = 4 × 5 = 20 m/s
To change m/s in kh/hr we multiply by 18/5
e.g. 45 m/s means
45 × = 162 k/h

Average speed =

Suppose a man covers a certain distance at x km/hr and an equal distance y km/hr.
then, the average speed during the whole journey is ( ) km/hr.
Problems
1. A man travels at speed of 15kmph from his house to office and returns at the speed
of 45kmph. Find his average speed for the entire journey

56
Here take LCM of speeds and assume that is our distance
LCM of 15 and 45 is 45. So here 45 is our Distance

T = (45/15) = 3hr

H "O
!
T = (45/45) = 1hr

Average speed =
! !
=
#
= 22.5 kmph
2. A car travels x km at speed of 60kmph and travels another 2x km at 40 kmph. Find
average speed?

LCM of 60 and 40 is 120.


X 2X

"
120 240

T = (120/60) = 2hr
T = (240/40) = 6hr

Average speed =
#$%
= = 45kmph
$
3. The ratio of the distance from A to B and the distance from B to C is 3 : 2. A man
travels from A to B at speed of 60kmph B to C at speed of 50kmph. Find the average
speed.

LCM of 60 and 50 is 300



A "C

3 B 2
300 200
T = (300/60) = 5hr
T = (200/50) = 4hr

Average speed =

%%
= !
= 55 & kmph

57
4. A car can travels a certain distance covers in 12hr. If it covers the half of the distance
at 80 kmph and the second half of the distance at 40 kmph then find the total
distance travelled by the car

LCM of 80 and 40 is 80

"

80 40
T = (80/80) = 1hr
T = (80/40) = 2hr
3hr = 160 km
12hr = ? 640 km
5. A boy goes to school at speed of 3 kmph and returns to the house at speed of 2
kmph. If he takes 20hr for the entire journey then find the distance between his
house and school

LCM of 2 and 3 is 6
# ( ()
H "S
(# ()

5hr = 12km
20 = ? So, total distance 48km
So distance between his house to school is (48/2) = 24km
6. If a person travels in a car at speed of 30kmph then he can reach his office in time.
He covers half of the journey in 4/5 th time. What should be his speed for the
remaining part of the journey so the he reaches his office in time
4/5 1/5

"

Total distance 5 * 30 = 150km


Half of the distance (75) = 4/5 th time (4 hr)
Remaining half of the distance (75) = 1/5 th time (1hr)
So 75 kmph
7. A bus travels a distance of 300 km in 8 hr, partly at 40kmph and remaining part of
35kmph. Find the distance travelled at 35kmph

D = d1 + d2
300 = 40 T + 35 (8 – T)
300 = 40 T + 280 – 35T

58
20 = 5T
T = 4hr
35 km distance 4 * 35 = 140km

8. A person on scooter traveling at a speed of 44kmph and takes 150 minutes to cover
a certain distance. At what speed should he travels to cover the same distance in 165
min

S1T1 = S2T2 (D is constant)


44 * 150 = 165 * S2
S2 = 40 kmph

9. A person travels 20 minutes to cover a certain distance. If B is 20% faster than A then
how much time will be taken by B to cover the same distance
A B
100 120
100 * 20 = 120 * T2
T2 = 16 # min

10. A person travels a certain distance at speed of 20 kmph. If the speed is increased by
5 kmph then he will take 90 minutes less. Find the distance travelled by the person
20 kmph 25 kmph
t t - 90
20 t = 25 t – 2250
t = 450
D = S * t = 20 * (450/60) = 150 km

11. A and B start together from a point with speeds in the ratio of 3:2. If B takes 2 ½ hr
more than A to reach their destination then how much time A will take to the
destination

A B
3 2
t t+5/2

S 1T 1 = S 2T 2
3 t = 2t +5
T = 5hr

59
12. Walking ¾ th of actual speed a man reaches his office by 10 min late. What is the
actual time to reach his office
Difference between Nr and Dr is 1 (3 ~ 4)
So 1 part = 10 min
Always multiplied with Nr
i.e. 3 parts = 3 * 10 = 30 min

13. Walking 7/5 th usual speed a man reaches his office by 12 min early. Find usual time to
reach his office
Difference between Nr and Dr is 2 (7 ~ 5)
So 2 parts = 12 min
Always multiplied with Nr
i.e. 7 parts = 7 * 6 = 42 min
14. Walking 20% more than the usual speed a man reaches his office by 10 min early find
usual time
120/100 = 6/5
Difference between Nr and Dr is 1 (6 ~ 5)
So 1 part = 10 min
Always multiplied with Nr
i.e. 6 parts = 6 * 10 = 60 min

15. A boy walking at a speed of 50 kmph he can reach his school by 20 min late. If he travels
at 60 kmph he can reach his school by 10 min early. Find the distance between his house
and school
LCM of 50 and 60 is 300.

T = (300/50) = 6hr
%
H
$%
"O
T = (300/60) = 5hr
1hr gap = 300
½ hr gap = ?
D = 150 km

60
16. A man missed a train by one hour if he travels at speed of 4 kmph. If he increases
his speed to 5kmph still he miss the train by 24 minutes. find the distance between
his house and station and at what speed should be travel so that he reaches the
station exactly in time
LCM of 4 and 5 is 20.

T = (20/4) = 5hr
!
H "O
T = (20/5) = 4hr
1hr gap (60 min) = 20
36 min gap = ?
D = 12 km
With 4 km late 1 hr = 3hr
In time = 2hr
Time = 2hr D = 12km
Speed = 12/2 = 6kmph

Relative speed
If two vehicles are moving same or opposite direction speed of one vehicle with
respect to another vehicle is called relative speed.

Same direction
Relative speed = S1 ~ S2

Opposite direction
Relative speed = S1 + S2

17. Two persons A and B starting at the same point traveling with speeds of 6kmph
and 9kmph respectively to cover a distance of 6km. If they travel in the same
direction after how much time they will meet each other

Relative speed = S1 ~ S2 = 6 ~ 9 = 3kmph


D = 6km
T = D/S = 6/3 = 2hr

18. Two persons A and B start from a point traveling with speeds of 30kmph and
50kmph respectively. How many kilometres a part they will be after 5hr if they are
traveling in the same direction

Relative speed = S1 ~ S2 = 30 ~ 50 = 20kmph

61
T = 5hr
D = 20 * 5 = 100 km

19. The distance between two stations Ongole and Hyderabad is 270 km. A train
starts from Ongole at 6 am with speed of 36 kmph towards Hyderabad. Another
train starts from Hyderabad at 6 am speed of 54 kmph towards Ongole. At what time
and what distance from Ongole the two trains will meet each other.


36kmph O "H 54kmph
6 am 270 km 6am

Relative speed = S1 + S2 = 36+54 = 90kmph


D = 270km
T = 270/90 = 3hr
Time they will meet each other = 6 am + 3 = 9am
The distance from Ongole = 36 * 3 = 108km

20. The distance between two stations Ongole and Hyderabad is 522 km. A train
starts from Ongole at 6 am with speed of 36 kmph. Another train starts from
Hyderabad at 8 am speed of 54 kmph. At what time and what distance from Ongole
the two trains will meet each other.
72km 8am 450km

36kmph O "H 54kmph
6 am 522 km 8am

Relative speed = S1 + S2 = 36+54 = 90kmph


D = 450km (522 – 72)
T = 450/90 = 5hr
Time they will meet each other = 8 am + 5 = 1pm
The distance from Ongole = 36 * 7 = 252km

21. A train leaves Hyderabad at 6 am traveling at 60kmph another express train


leaves Hyderabad by one hour late traveling at 75kmph. At what time and what
distance from Hyderabad the two trains meet each other


6 am (60kmph)
"
7 am (75kmph)

Relative speed = S1 ~ S2 = 60 ~ 75 = 15kmph


T = D/S = 60/15 = 4hr
After 4hr = 7 + 4 hr = 11 am

62
Meeting distance = 60 * (1 + 4) = 300 km

22. A thief escape from police station starts at 9am with speed of 30 kmph. The
police man chasseing the thief starts at 11 am with speed of 40 kmph at what time at
what distance from police station


(T) 9 am (30kmph)
"
(P) 11 am (40kmph)

Relative speed = S1 ~ S2 = 30 ~ 40 = 10kmph


D = 2 * 30 = 60km
T = 60/10 = 6hr
Meeting time = after 6hr = 11 am + 6 hr = 5pm
Distance = 30 * 8 = 240 km

23. A train M leaves Meerut at 5am and reaches Delhi at 9am. Another train leaves
Delhi at 7 am and reaches Meerut at 10:30 am. At what time do the trains cross each
other?
2hr(3.5 * 2)

5 am M " D 7 am

M – D = 5 ~ 9 = 4hr
D – M = 7 ~ 10:30 = 3.5hr
Take LCM of 4 and 3.5 is 14
Assume that is distance between two stations = 14 km
Speed (M – D) = 14/4 = 3.5kmph
Speed (D – M) = 14/3.5 = 4kmph
Relative speed = S1 ~ S2 = 3.5 + 4 = 7.5kmph
T = (7/7.5) * 60 = 56 min
The two trains will meet each other = 7:56am

24. Excluding stoppages, the speed of a bus is 54 kmph and including stoppages, it is
45 kmph. For how many minutes does the bus stop per hour?

Due to stoppages, it covers 9 km less.


Time taken to cover 9 km = (9/54)*60 = 10 min

25. A man takes 8 hr to cover a distance from A to B on bike and returns from B to A
by car. If he travels both ways by car he saved 2hr. how much time he will be taken
to cover the both ways on bike

63
B+C = 8hr
C+C = (8hr – 2hr) = 6hr
2C = 6hr
C = 3hr
B+3 = 8
B=5
B+B = 10hr
To cover both ways on bike = 10hr

TRAINS

Speed =

To change km/hr in m/s we multiply by 5/18


e.g. 72 k/h means
73 × = 4 × 5 = 20 m/s
To change m/s in kh/hr we multiply by 18/5
e.g. 45 m/s means
45 × = 162 k/h

Case 1: When a train crosses a man/pole/tree (in stationary form)

Let a train having speed is S and length L, crosses a pole in T time then,
*
S = (Here the breadth of man, tree, pole is negligible with respect to the train)

1. Find the time taken by a train 180 m long, running at 72 km/h in crossing an
electrical pole.
*
S = (because pole has no breadth)
* % %
T= = +, ×- =
/ %
./

= 9 sec

64
Case 2: when a train crosses a platform
Let a train having speed is S and length L1, crosses a platform whose length is L2 in time T,
then
1 1
S=

2. A train 140 m long is running at 60 km/h. In how much time will it pass a platform
260 m long?
1 1 1 1
S= T=
!% $%
= -
$%×
./

!%% ×
=
#%%

= 24 sec
Case 3: when a train crosses another running train
Let a train having speed S1 and length L1, crosses another train which is travelling in opposite
direction at a speed of S2 and length L2 in T time, then
1 1
S1+S2= (both object having length and speed)

Note:

• If both object having length (L1 and L2) then the length is always sum (i.e. L1 + L2)
• If both object having speed S1 and S2 then the speed are added (when direction is
opposite) and subtracted (when direction is same)

3. Two trains, 121 m and 99 m in length respectively, are running in opposite direction,
one at the rate of 40 and the other at the rate of 32 km an hour. In what time will
they be completely clear of each other from the moment they meet?
As the two trains are moving in opposite directions their relative speed
(S1+S2) = 40+32 = 72 km/hr

72× = 4 × 5 = 20 m/s
1 1
T= 2
2
&&
=
%

= 11 sec

65
4. In ex 3, if the trains were running in the same direction, in what time will they be
clear of each other?
Relative speed (S1- S2) = 40 - 32 = 8 km/hr
%
=8× = m/sec
&
1 1
T=
2 32
&&
= ×9
%

= 99 sec
Case 4: when a moving train crosses a man who is running / walking
Let a train having speed S1 and length L1, crosses a man, who is running at a speed of S2 in
the same direction in T time, then
1
Same direction (S1- S2) = (since man has negligible breadth)
1
Opposite direction (S1+ S2) =

5. A train, 110 m in length, travels at 60 km/hr. In what time will it pass a man who is
walking at 6 km an hour
(a) Against it
(b) In the same direction
(a) Relative speed = 60 + 6 = 66km/hr = #
m/sec
1
T = 2
2
%
= × 3

= 6 sec

(b) Relative speed = 60 – 6 = 54km/hr = 15 m/sec


1
T = 2
32
%
=

= 7 # sec

Case 5: when a moving train crosses a man who is sitting into a moving train
Let a train having speed S1 and length L1, crosses a man, who is sitting in another train which
travelling in opposite direction at a speed of S2 and length L2 in T time, then

66
1
Opposite direction (S1 + S2) =
1
Same direction (S1- S2) =

6. A goods train of 50m long is moving with speed 54 km/hr. A man is sitting in a
passenger train which is moving with 18 km/hr. with same direction. In how many
seconds the goods train crosses the man sitting in a passenger train?

Relative speed S1- S2 = 54 – 18 = 36 km/hr = 10 m/sec


1
T=2
3 2
%
=
%
= 5 sec

7. A man is standing on a railway bridge which is 50 m long. He finds that a train


crossed the bridge in 4 sec but himself in 2 sec. find the length and speed of train
* %
S= ….. (1)
!.
*
S= ….. (2)

From 1 and 2
* * %
= !.

L = 40 m
S = 40/2 = 20 m/s

8. A train running at a speed of 25 km/hr. takes 18 sec to pass a railway platform and it
takes 13.5 sec to pass a man who is running at a speed of 5 km/hr. in the same
direction. Find the length of the train and length of the platform.
Let length of the train is L
And the length of the platform P
* 5
25 × =

L +P = 125
Again
*
(25 – 5) × =
#.

67
Length of the train = 75 m
Length of platform = 125 – 75 = 50

BOATS & STREAMS


Ex:-1. If boat and water are traveling in the same direction, then the boat is said to
be in downstream. (Along with)
Sol:- Boat speed in still water (2x) = 6 kmph
Water speed (v) = 4 kmph
Boat speed in down steam (DS) = u + v
= 6 + 4 = 10 kmph

Ex:- 2. If boat and water are traveling in the opposite direction, then the boat is said
to be in upstream. (Against with)
Sol:- 2l = 6 kmph
v = 4 kmph
Boat speed in up steam us = u – v
= 6 – 4 = 21 kmph

6
Ex:- If DS and us are given then boat speed in still water (u) =
36
Water speed (v) =

1. A man can swim downstream at 12 kmph and upstream at 6 kmph. Find the speed of
the man in still water and the speed of the current.
ds = 12 kmph
us = 6 kmph
$
u=
= = 9 kmph
3$ $
v= = = 3 kmph

2. A men can row 36 km downstream in 6 hr and 40 km upstream in 8 hr. find the


speed of man in still water and speed of current.
#$
us = = 6 kmph
$
!%
us = = 5 kmph
7
u= = kmph
v= kmph

3. A boat can row 24 km downstream M 6 km/hr. If the speed of the current is 2 kmph
find the time required for the boat to travel the same distends against the current.
!
ds = = 6 kmph
!

68
ds = 6 kmph
v=2
u=4
! !
Again T = = !3
= = 12 hr

4. A man can row 8 kmph in still water and speed of the current is 4 kmph. He takes 2
hr to row to a place and back to the starting point how far in the place.
u=8
v=4
ds = 12 kmph
us = 4 kmph
(7, 3 8, )
d=
7
( , 3 !, )
=

($!3 $)
= $
!
=2× $
= 6 km

5. A man can row 18 kmph in still water, speed of the stream is 6 kmph. Then find the
time required for the man to row a distance of 6 km and rete run to the starting
point.
( $, 3 $, )
6=
×
( ) (7, 3 8 , )
6= #$ !
$ ×#$
T=
# #
T= hr × 60 = 45 min
! !

6. A man can row at 9 kmph in still water he takes twice as much time to row upstream
as compared to the down upstream as compared to the downstream what is speed
of the current.
d = 1 hr × (9 + v)
d = 2 hr × (9 – v)
9 + v = 2 (9 – v)
9 + v = 18 – 2v
3v = 9
V=3

7. A man can row 20 km upstream and 30 km downstream in 4 hr. he can row 40 km


upstream and 45 km downstream in 7 hr. find the speed of the man in still water.

% #%
4= +
7
!% !
7= 7
+

69
x= y=
7
4 = 20x + 30y × 2 = 40x + 60y = 8
7 = 40x + 45y × 1 = 40x + 45y = 7
= 15y = 1
20x + 30 × =4
Ds = 15 20x = 2
X= U = 10
%
7
Us = 10, ds = 15 u= =

70
CUBES & DICE

♠ A cube is a symmetrical prism with 8 corners 12 edges,6 faces.


corners – points
edges – lines
faces – planes

CUTTING OF A PLANE CUBE:- If we cut a cube ‘n’ times in a single direction i.e.
Parallel cuttings. Then we will get ‘n + 1’ no.of pieces.

♠ To get minimum no.of pieces with maximum cuts then we have to cut the cube
in single direction (parallel cuttings)
♠ To get maximum no.of pieces with minimum cuts then we have to cut the cube
equally along 3 – mutual perpendicular directions (cuttings)
Ex:-1. If a cube is cut for 15 times then find the max & min no.of pieces possible.
Parallel cuts = 15 cuts = 16 pieces (minimum)
x y z

⊥ ar cuts 5p 5p 5p

6p × 6p × 6p = 216p ( maximum)
Ex:-2. 20 cuts
Parallel cuts = 20 cuts = 21 pieces (minimum)
x y z 21 cuts – 7 7 7

⊥ar 18 1 1 20 cuts – 7 7 6

19 × 2 × 2 = 76 19 cuts – 7 6 6
10 5 5

11 × 6 × 6 = 396
7 7 6

8 × 8 × 7 = 448 (maximum)

71
Ex:- 3. A cube is cut and made into 343 identical pieces then find the
maximum and minimum no.of cuts.
Parallel cuts = 343p = 342cuts
x y z
ar
⊥ cuts 343p = 7p × 7p × 7p

6 × 6 × 6 = 18 cuts
Ex:-4. 120p
Parallel cuts = 120p = 119 cuts
⊥ar cuts = x y z
120 = 5p × 6p × 4p

4 + 5 + 3 = 12 cuts
Ex:-5. 53 pieces
11al cuts = 53p = 52 cuts
⊥ar = x y z
53p = 53 × 1 × 1

52 + 0 + 0 = 52 cuts
Note:- If no.of pieces are prime then min & max no.of cuts are same.

Ex:- 6. 3n cuts

llal cuts = (3n + 1)p

equally ⊥ar = n cut n cut n cut

(n + 1) (n + 1) (n + 1) = (n + 1)
p + q + r cuts = x y z
p q r

(p + 1) (q + 1) (r +1) = (p + 1) (q + 1) (r + 1) pieces

CUTTING OF PAINTED CUBE:- A cube is painted with colour and cut for 3n
times b cuts (equally along 3 mutually ⊥ar directions) then

x y z
1. Total no.of pieces = n cut n cut n cut
(n + 1) (n + 1) (n + 1) = (n + 1)3p
2. No.of pieces with paint on 3 faces – 8 corners × 1p = 8p
3. No.of pieces with paint on 2 face – 12 edges × (n – 1) = 12(n – 1)

72
4. No.of pieces with paint on 1 face – 6 faces × (n – 1)2
5. No.of pieces with paint no – painted face (n – 1)3 (occurs in the miiddle)
Unequal x y z
P q r
p cuts + q cuts + r cuts
1. (p + 1) (q + 1) (r + 1) – Total no.of pieces
2. 3 faces – 8 corners × 1p = 8p
3. 2 faces – 4(p – 1) + 4(q – 1) + 4(r – 1)
4. 2(p – 1) (q – 1) + 2(q – 1) (r – 1) + 2(r – 1)(p – 1) – 1 face
5. No.of painted = (p – 1)(q – 1)(r – 1)

♠ A painted cubs is cut for 3d times equally along 3 mutually ⊥ar


directions and made into 1331 identical pieces. Find the no.of pieces
with 3 painted faces.
2. No.of pieces with two painted faces.
3. No.of pieces with one painted faces
4. No.of pieces with on paint,
a. 8 pieces
b. 2 – faces – 12(n – 1)
- 12 × 92 = 108 pieces
c. 1 – face – 6(n – 1)2
- 6 × 92 = 81 × 6 = 486 pieces
d. no paint = (n – 1)3
= 93 = 729

2. 7 cuts, 9 cuts, 10 cuts


Unequal x y z
7 cuts 9 cuts 10 cuts
a. Total = (7 + 1) (9 + 1)(10 + 1)
= 8 × 10 × 11 = 880 pieces
b. 3 faces = 8 corners × 1p = 8p
c. 2 – painted faces – 4(6) + 4(8) + 4(9)

73
- 24 + 32 + 36 = 92 pieces
d. 1 – painted face – 2(6)(8) + 2(8)(9) + 2(9)(6)
- 2[48 + 72 + 54] = 348 pieces
e. No.of painted = (p – 1)(q – 1)(r – 1)
♠ A cube is painted such that red colour on 2 adjacent faces, black
colour on 2 adjacent faces, green colour on two opposite faces. The
cube is cut and made into 512 identical pieces find the no.of pieces
with exactly.
a. 2 colours b. 1 colour c. 3 colours d. no red colour
512 = 8p × 8p × 8p
7cut corner (8p)
n=7 G–B–B G–B–R G–R–R
1+1 1+1+1+1 1 + 1Edges (12
edges × (n – 1))
G–B G–R B–R B–B R-R
Edges - 1+ 1 + 1 + 1 1 + 1 + 1 +1 1+1 1 1
Pieces – 4(n – 1) 4(n – 1) 2(n – 1) (n – 1) (n – 1)
4×6 4×6 2×6 6 6
24p 24p 12p 6p 6p
Faces (6faces × (n – 1)2)
B R G
2 faces 2 faces 2 faces middle (no-paint)
Pie - 2(n – 1)2 2(n – 1)2 2(n – 1)2 (n – 1) = 63
2×6 4×6 2 × 62 = 216
72 72 72
♠ A cube painted yellow on all faces in cut into 27 small cubs of equal
sizes. Answer the questions that follow.
27 = 3p × 3p × 3p

2 cut
n=2
a. How many cubes are painted on one face only?
6 faces × (n – 1) 2
6 × (2 – 1) 2
b. How many cubes are not painted on any face?

74
(n – 1)3
(2 – 1) 3
♠ A cube is coloured red on all faces. It is cut into 64 smaller cubes of
equal size. Now, answer the following questions based on this
statement.
64 = 4p × 4p × 4p

3 cut
N= 3
a. How many cubes have no face coloured?
(n – 1)3
(3 – 1)3 = 8
b. How many cubes are then which have only one face coloured?
6 faces × (n – 1) 2
6 × (3 – 1) 2 = 24
c. How many cubes have three faces coloured?
8 corners × 1p = 8p

75
S
5. A compass is damaged such that the pointer south which shows north is now
showing west. If a person is moving towards S/E by using damage compass. Then
find the direction in which he is moving actually.
He is moving N/E.
WEST

N
S N
W E

S
N/E
E

6. A compass is damaged such that the pointer which shows west is now showing
north. If a person wants to go towards South – East. Then in which direction, he
has to go by using the damaged compass.
North/East
E
S/E
N
N S Morning shadow towards - West
W E
Evening shadow towards - East
S

7. Sita is walking on a road one fine morning. If her shadow falls in her right side
then find the direction in which she is moving.
She is moving on south side

South

Shadow

East Sit west


a

North

78
8. Rani is walking in a park along with her dog. On one evening. When she left her
dog it rotates 135° clockwise direction and runs away from her find the direction
in which the dog is running. If her shadow fails her backside.
North – East]

. W

RANI 135°
S N
SHADOW
E N/E

9. Ravi and Gita are talking to each other by standing face to face on one evening. If
Ravi shadow falls on back of Gita then find the direction in which Gita is standing
west.
Gita left side south
Gita right side north
R West

S G N

Shadow
E

Problems on tracing of path:-


Pythagoras theorem:- a √= + ?
Where a = effective vertical distance
b = effective horizontal distance
b
10. Ravi started walking from his house, after walking 10m towards East he to left
term walks for 10m then he took Right term walks for 20m, finally he took Right
turn walks for 40m & reached the college. Find the distance b/w house & college
and also the direction of college with respect to house.
= 42. 42 S/E 20m
10m
H 40m
30 30√2 10m
√30 + 30

79
30 30
E N E S E S 30√2 = 30 × 1.414
10 10 20 40 30 30 = 42.42m
√900 + 900 = √30 + 30
√1800 = √900 × 2
= 30√2 S/E

11. Kishore started walking from his house, after walking 40m towards North, he
took right turn walks for 70m followed by 80m towards is right and walks for
100m after taking the right turn & reached the library. Find the distance b/w
house & library and also the direction of house to library.
50m N/E 70m
N E S W S W 40m 80m
40 70 80 100 40 30 H
√1600 × 900 N/E 40m
√2500 = 50 L 50m
100m

12. Kishore started walking from her house, after walking 40m. He took right turn
walks for 10m then took right turn walks for 120m, finally he took left turn walls
for 10m and reached the park. Find the direction in which he started walking if
the park is in the direction of N/E. with respected to house.
From he started West direction.
S
E H W 40m
N/E N 10M

120
10M
P

13. Ramesh and Suresh started walking from their college, after walking 30m
towards West Ramesh took left turn walks for 20m. Then took right turn walks
for 30m and reached his house. Suresh took right turn walks for 20m then took
right torn unlike for 40m and reached his house. Find the distance b/w Ramesh
house and Suresh house.

40m S

20m s 20 + 20 = 40m

30m

80
R R&S
20m
R 30 30 + 40 = 70
RS = √40 + 70
= √1600 + 4900
= √6500
= 1065
= 10 × 8.1
= 8.1m

14. Two cars A & B moving towards each other which are separated 1000m, an a
straight road. Car A money for 200m. Then took left turn travel for 100m then
took right turn travels for 50m and stop cars travels for 150m then took left turn
travels for 600m. Then took right turn travels for 200m and stopped. Find the
distance b/n car A & car B. A
100m 50m
A 20m 150m
=1000 – [250 +350] 100m + 600m
= 400 =700 600m

700 √490000 = 16000


B1 120m

400

= √650000

= 100√65
= 100 × 8.1
= 810m
15. Point M is towards the North of point N and is 10m away from N point P is 6m to
the West of point N & 8m to East of point Q. what is the shortest distance b/w
the points M & Q?
M

√10 + 4 10m = √296


= √296

Q 8 P 6 N

81
16. Point A is 5m away towards the East from point z. which is 6m away towards the
North from point B which is 8m away towards the East from point y.If point x is
4m away towards the south from point y, then in which direction is point x with
respect.to point A?
N
5
Z E
W
6 S/W S
8 B
Y
4

82
PERMUTATIONS & COMBINATIONS

PERMUTATIIONS & COMBINATIONS IS NOTHING BUT COUNTING


FUNDAMENTAL PRINCEPLE OF COUNTING:- If the task T1 can be done in “m” ways & task T2
can be done in “n” wags.
Then task T1 & T2 can be done in m × n ways.
Task T1 or T2 can be done in M + n ways.
Ex:- There are 5 diff ways to travel from Nellore to Ongole and there are 7 diff ways to travel
from Ongole to Vijayawada. In how many ways a person can travel from Nellore to
Vijayawada via Ongole.

No.of ways = 5 × 7 = 35 ways


a p q ap, aq, ar,…….
b r bp, bq, ……..
N c O s cp, cq,…….
5 d t dp, dq,…….
e v u 7 ep,…… Total = 35 ways

RE- ARRANGEMENTS:- We can re-arrangement “r” persons in “r” places in r! ways 5 persons
5 chairs 5! Ways
n! = 1 × 2 × 3 × 4 × ………(n – 1) × n 1! = 1
n! = (n – 1!) × n 2! = 2
0! = 1
3! = 6
n! = (n – 1!) × n 4! = 24
10! = 9! × 10 =10! 5! = 120
6! = 720
7! =5040

PERNITATIONS:- (SELECTION & ARRANGEMENT)

• We can select & arrange “r” charge from “n” things in npr ways.
• This is done when no.of persons or things & no.of places is not equal.

83
n
pr = n!/(n – r)!

• 8 persons in 4 places
Possibility = 8 × 7 × 6 × 5
= (8 × 7 × 6 × 5 × 4 × 3 × 2 × 1)/(4 × 3 × 2 × 1)
= 8!/4! = 8!/(8 – 4)! = 8p4

7
p2 = 7!/(7 – 2)! = 7!/5! = 7 × 6
7
p2 = 7 persons in 2 places = 7 × 6
8
p3 = 8 in 3 = 8 × 7 × 6
n n
po = n!n= (n – 1)! = n!/n! = 1 p1 = n
po = 1 n
p2 = n × (n – 1)
n
pn = n!

COMBINATION (ONLY SELECTION)


• We can select “r” things from “n” things in npr ways.
• As we know select & arrangement = npr
= select × arrangement = npr
n =nselection = npr/r! = n!/(n – r) ! r!
Cr = pr/r! = n!/(n – r)! r!

7
C2 = 7P2/2! = (7 × 6)/(1 × 2) = 2!
8
C3 = 8P3/3! = (8 × 7 × 6)//(1 × 2 × 3) = 56
n
Cr = nC(n - r)

11
C8 = 11C11- 8 = 11C3n
cp = ncq employs p = q (OR) n = p + q

n
Co = 1
n
C1 = n
n
C2 = n × (n – 1)/(1 × 2)

n
Cn = n!
n
Co + nC1 + nC2 +……….+ npn = 2n
**
n
Co + nC2 + nC 4+………. = np1 + nC3 + nC5 + ….= 2n -
1 84
2n/2

Ex:- 1. There are 15 railway stations b/w Nellore & Ongole. To run a train b/w Nellore
& Ongole low many diff types of tickets to be printed?

No.of stations = 17
From each station 16 diff tickets are printed
Total no.of diff tickets = 17 × 16
(or)
Each tickets requires 2 stations to be panties

17
Solution as well as we need rearrangement because NLR – ONG, ONG –
NLR will be 2 diff
2 17
P2 = 17 × 16 = 272

1. In how many ways we can arrange 5 boys a 5 girls in a row such that.
a. Without any reputation.
b. All boys come together.
c. All girls come together.
d. All boys girl come together.
e. No two boys come together.
f. No two girls wine together..
g. Boys occupy come places.
h. Boys, girls come alternatively.
i. Goes occupy end places.
j. Two particular girls always together.
k. 1particular boy x girl are not together.

a. 10!

b. 5! × 6!

c. 5 B + 5 G = 6 persons
= 5! × 6!

d. 5 B + 5 G = 2 persons
5! × 5! 21 = 5! ×5! × 2!

e. 1 G2 G3 G4 G5 G6 = 5!

85
6 places – 5 places = 6P5 5! × 6P5

f. ~B~B~B~B~B = 5!
6 places – 5 person = 6P5 5! × 6P5

g. 5! × 5! 1 2 3 4 5 6 7 8 9 10
B – 5 even – 5! G – 5 places – 5!
h. B G B G B G B G B G 5! × 5!

G B G B G B G B = 5! × 5! 2 × 5! × 5!

i. G _ _ _ _ 8! _ _ _ _ G 5
P2 × 8!

5
P2

j. 2G + 3G + 5B = 9 persons

2! 9! 9! × 21

k. Total – together
10! – (2! ×9!)
= 9! (10 - 2)
= 8 × 9!

2. In how many ways we can arrange the letters of the word ARTICLE that
a. Without any resolution
b. All vowels come together
c. No two vowels come together
d. Vowels occupy even places
e. Consonants occupy even places
f. Vowels & consonants come alternatively.
g. Meddle place occupied by vowel
h. Slating with vowel & ending with constant

a. ARTICLE = 7 letters 7!

b. AIE RTCL = 3! × 5!

c. 1 R 2 T 3 C 4 L 5
5
P3 × 4!

86
I J
d.
# ! $ K
3! × 4!5

e. 1 2 3 4 5 6 7 { not possible}
4
P3 × 4!

f. ARITECL × = 3! × 4!
g. 4 = 3 × 4!

h. 3 × 4 × 5! (3P1 × 4M1 × 5!)

i. Vowels occupy odd places 4


P3 × 4!

3. In how many ways we can arrange the letters of word ASSASSINATION such that
a. Without any resolution
b. All s’s together
NO$
As rearrangements are not necessary here we divide (( P

3 4 2 2 0 1 1
a. Total digits = 13
Total reductions = A = 3, S = 4, I = 2, N = 2, T = 1, O = 1
#!
No.of arrangement = #! ×!! × ! × !

b. All s’s together =


9 letters 10 persons
ssss 3A 2I NI
%!
4s 4 × #! × × 9!
! × !

%!
H7D SSSS AAA II NN 7 0 = 10 DAYS = #! ×
! × !

• How many word can be formed by using all the letters of the word BIHAR =
5!
DIRECTOR all vowels

4. How many 4 degited no > can be formed by using{1, 2, 3, 4, 5, 6, 7, 8} by using


each digit for at most once find how many of them are divisible by 2?

No.of digits formed = 7P4 = 7 × 6 × 5 × 4


To be divisible by 2 last digit

87
Should be 2, 4, 8
6 × 5 × 4 2 = 120
6 × 5 × 4 4 = 120
6 × 5 × 4 8 = 120
360
5. How many 4 digital no’s by using {1, 2, 3, 4, 5, 7,8} by using each digit for any
no.of time and how many of them are divisible by 2?

As the digits are repeated no.of possibility at each digit will be same.
No.of 4 days formed = 7 × 7 × 7 × 7 = 74
No.of digits divisible by 2 = 7 × 7 × 7 2 = 343
= 7 × 7 × 7 4 = 343
= 7 × 7 × 7 8 = 343
1029

6. How money 4 digit no’s can be formed by using {0, 1, 2, 3, 4, r } each digit is used
for almost once how mans of then are divisible by 2?

5 × 5 × 4 × 3 = 25 × 12 = 300
5 4 3
4 × 3 × 2 × 0 = 24 = 60
4 × 4 × 3 2 = 48
4 × 4 × 3 4 = 48
4 × 4 × 4 8 = 48
204

7. {0, 1, 2, 3, 4, 6} any digit can be used for any no of times.

5 × 6 × 6 × 6 0 = 180 180
(Because 0 cannot the of staling of 4 digit no
Divisible by 2 :- 5 × 6 × 6 0 = 180 180
5 × 6 × 6 2 = 216 180
5 × 6 × 6 4 = 216 180
5 × 6 × 6 6 = 216 180
828

8. How many 5 digital no’s can be formed which are divisible by using {1, 2, 3,
4, 5, 8}

4 × 3 × 2 1 2 = 24 × 7
_ _ _ 2 4 = 180
_ _ _ 28
_ _ _ 32
_ _ _ 48

88
_ _ _ 52
_ _ _ 84

9. If the no.of words that can be formed by re-arranging the letters of the word
“POSTAL” are arranged in alphabetical order then find the rank of postal?

POSTAL
Alphabetical order = A, L, O, P, S, T
A _ _ _ _ _ = 5! = 120
L _ _ _ _ _ = 5! = 120
O _ _ _ _ _ = 5! = 120
P A _ _ _ _ = 4! = 24
P L _ _ _ _ = 4! = 24
P O A _ _ _ = 3! = 6
P O L _ _ _ = 3! = 6
P O S A _ _ = 2! = 2
P O S L _ _ = 2! = 2
POSTAL=1
425

10. Find the num of all 5 digital numbers that can cam be formed by using {1, 2, 3,
4, 5, 6, 7} in which each digit is used at most once?

Num = Num of all digit × 5 digeted 1 × (5 – 1)!


= (1 + 2 + 3 + 4 + 5 + 6 + 7) × (11111) × 4!
= 28 × 11111 × 4!

11. In low many way we can arranged crow of a boat of 12 members in which 6 can
seat on stroke side & 6 can seat on non-stroke side? Among 12 members 2
persons are not witting?
To not on stroke side and 3 persons are not willing to set on non-stroke side?
%! &!
10
P 6 × 9P 6 = ×
!! × $! #! × $!
% ×& × ×K ×$ × & × ×K ×$
= × $ ×
$ × ×! ×# × × ×! ×# × ×
= 210 ×
= 21 × 21 × 2
= 441 × 2 = 884
$! $!
6
P2 × 6P3 × 7! = ! ! × #! × 7!

=6×5×6×5×4×

89
CIRCULAR ARRANGEMENTS:-
We can arrange ‘n’ things along a circular table in (n – 1)! Ways.
If we get same arrangement in clock wise & ante clock wise direction i.e.
necklace, garland, …………
So on
( 3 )!
No.of ways =
12. In low many ways we can arrange 5B & 5G along a circular table sock that
a. Without any reputation.
b. All boys come together.
c. No two boys come together.

a. 9!
b. 5! × 5! – 5B + 5G 6persons
5! (6 – 1)!
5!
c. 5 girls ( 5 – 1) 4!
5 plans 5 boys – 5!
5! × 4!

13. In low many ways we can select 5 persons from a group at 10 persons such that.
a. Without any resolution.
b. 1 person must be included.
c. 2 particular person must be included.
d. 1 person included & 2 persons

a. 10P5 10
C5
b. 1 × 9P4 1 × 9C4
c. 9P5 10 – 2 = 8
d. 1 × 7P4 8
C5
5 5
e. 10 - 2 - 1
7
C4
5 -1

14. In how many ways we can select a cricket 11 from a group of 7 batsmen, 6
bowlers & 3 w.k such that the team included exactly 6 batsmen & 4 bowlers.
a. Without any reputation.
b. There will be exactly 3 men.
c. There is a majority of women.
d. There will be almost 2 women.
e. There will be all-out 1 men & all-out 1 women such that.
a. 9C5
b. 5C3 × 4C2

90
c. 4
C3 × 4C4 × 4C4 × 5C1
d. 5
C5 × 5C2 × 4C1 × 5C3 × 4C2
e. 5
C1 × 4C1 × 7C3

5M 4W
1 4 5
C1 × 4C4
2 3 5
C2 × 4C3
3 2 5
C3 × 4C2
4 1 5
C4 × 4C1
Both are correct. But the latter helps in further simplification.

15. There are 100 points on a plane among them 30 points are collinear & among
the remaining no 3 points are collinear
a. Find the no.of str 8 lines that can be formed
b. Find the no.of Dles that can be formed.
a. No.of strait lines = 100C2 – 30CQ + 1
(as 2 points are requested to from a str 8 line)
b. No.of Dles = 100C2 – 30C3
(3 points are requested to from a str)
R(R3Q)
No.of diagonals of a ‘n’ polygon = nC2 – n =
S
No.of point of intersection of diagonals = nC2
n = no.of sides or no.of values (applicable to convers polygons)
If x1 + x2 + x3 + x4 ---------- nr = r
Then no.of non-negative integral solutions
(0, 1, 2, 3, 4…) = n + r + 1Cr – 1
No.of the integral solutions = n – 1Cr – 1
(1, 2, 3, 4, -------)

DE – ARRANGEMENTS:-
T T T T T
D(n) = n! [ -
S! Q! U! V! R!
+ - +……….+ (-1)n ]

16. In how many ways we can arrange to similar things equally among 4 persons.
SW! TV! TW!
20
C5 × 15C5 × 10C5 × 5C5 = TV!V! × TW!V! × V!V! × T
SW!
= (V!)4
No.of squealed that can be formed on 8 × 8 chars board = ∑ R2
No.of rectangles that can be formed on 8 – 8 chars board = ∑ R3

Note: N members gives shakes hand to each and every person then number of total
number of shakes hands = NC2 = N(N-1)/2

91
PROBABILITY
RANDOM EXPERIMENT:- An experiment done without knowing. The exert output is known
as random experiment.
Ex:- Tossing a coin.
Rolling a die so on.

SAMPLE SPACE:- Set of all possible outcomes of a random experiment is called sample
space.
Ex:- {head, toss}
S = {1, 2, 3, 4, 5,6} when a die is rolled.}

EVENT:- Any number of sample space is known as event.


EX:- Getting an even no. when a card is drawn.

BIAS BIASED EXPERIMENT:- An experiment by giving priority to a particular output is known


as biased experiment.

UNBIASED EXPERIMENT:- An experiment without any priority to a particular outcomes is


known as unbiased experiment.

PROBABILITY:- Probability of happening an event.


Y(Z)
P(E) = Y(Z) f – fordable

(or) u – unfavorable
[ ]
P(E) = [
\
= ^ t – total

Probability of unfavorable

92
_
event is not happening of an event. P(E) =
` _

` _
+
` _ ` _
P(E) +P(E) = =1

P(E) =1 - P(E)

Rang of probability of any event P(E) = [0, 1]

0 ≤ P(E) ≤ 1

If P(E) = 0 then E is an impossible event. N(E) ≤ N(S)


Y(Z)
If P(E) = 1 then E is a definite event or sure event. ≤T
Y(b)

c
If P(E) lies
d
o to 1 then E is an uncertain event. P(E) ≤ T

e) = f : u
ODD IN FAVOUR OF AN EVENT:- P(E) : P(Z
Odds against of an event :- P(E) : P(E)= u : f

MUTUALLY EXCLUSIVE EVENT :- If E1, E2 are mutually exclusive event then


P(E1 ∩ E2) = 0

COLLECTIVELY EXHAURTAVE EVENT:- If E1, E2 are collectively exhaustive events then P(E1 ∪
E 2) = 1

EQUALLY LITELY EVENT:- If E1, E2 are equally likely events then P(E1 ) = P( E2)

INDEPENDENT EVENT:- If E1, E2 are independent events then


P(E1 ∩ E2) = P(E1 ) × P( E2)

ADDITION THEOREM OF PROBABILITY:- P(E1 ∪ E2) = P(E1 ) + P( E2) - P(E1 ∩ E2)


e 1, Z
If E1, E2 are independent event then Z e 2 are also independent events.

93
TYPES OF PROBLEMS:-
a. Problems on coins
b. Problems on dice
c. Problems on cards
d. Problems on calendar
e. General problems

a. PROBLEMS ON COINS:- Single coin S = {H, T}


Total n(s) = z1 = z

1. Find the probability of


a. Head t = 21 = 2 (uncertain event)
f=1
T ]
Probability = S P(E) = ^

b. Head or tail t = 21 = 2 (sure event)


f=1+1=2
Probability P(E) = 1

c. Head & tail t=2 (impossible event)


f=0
]
P(E) = ^ = 0

d. Two coins S = { HH, HT, TH, TT}


Total n(s) = 22 = 4

2. Find the probability of getting both


T
a. Both heads t=4 P(E) = U
f=1

T
b. One head (Exactly) t=4 P(E) = S
f=2

3. Alteart one head t=4


f=3

94
Q
U
P(E) = (or) (1 – no head)
T Q
1 - U = U

4. N coins n(s) = t = 2n
R i
Probability of getting & head = hR
S

R T
hi
Alleast one head = 1 – no head = 1 -
SR SR
= [1 - ]
Probability of atleout two leads = 1 – [no head + 1 head]
T R T
= 1– + ShR
SR

T R
= 1-
SR
+ SR

Rh R⁄S
Equal no.of heads l tails = if n is even
SR

= 0 if n is odd

Rh R⁄S
Unequal no.of heads tails = 1 - if n is even
SR

= 1 – equal no.of heads


= 1 – 0 = if n is odd

Rh R⁄S
T3
No.of heads > no.of tails = S
SR
(as no.of unequal cares with
R
l h R⁄S
= SR
H > T = T > H)
SR 3S
R
l h R⁄S
SR
= if n even
SR mT

= if n is odd

1. If biased coins are toned find the probability of getting heads, tails alternately.
S T T
P = STW =
So
= VTS

2. A biased coin is such that three the chairs of gelling head = time the chances of
getting tail if a coins is tossed time find the probability of getting both head.
3H = 2T
p
=#

95
#
P(H) = , P(H) =
!
P(H) P(H) = × =

3. PROBLEMS ON DICE:-
SINNGLE DIE:- S = {1, 2, 3, 4, 5, 6}
T = n(s) = r! = 6

a. Probability of getting no 4 = P(4) =


$
q ( , ,#,!) !
b. Probability of getting divisor of r P(E) = = = =
$ $ #
c. Probability of getting composite no P(E) = =
$ #

TWO DICE:- S= (1, 1) (1, 2) (1, 3) (1, 4) (1, 5) (1, 6)


(2, 1) (2, 2) (2, 3) (2, 4) (2, 5) (2, 6)
(3, 1) (3, 2) (3, 3) (3, 4) (3, 5) (3, 6)
(4, 1) (4, 2) (4, 3) (4, 4) (4, 5) (4, 6)
(5, 1) (5, 2) (5, 3) (5, 4) (5, 5) (5, 6)
(6, 1) (6, 2) (6, 3) (6, 4) (6, 5) (6, 6)

&
b. Both even no. P = #$ = !
# # &
P(both even) = P(even) × P(even) = × = =
$ $ #$ !

$
a. Both equal no. = P = #$

b. Find the no. more than second no. equal = 6


Unequal chances = 30
In this ‘n’ equal chances of find no. > 2, x 2 > 1 will be same.
P=
#$
c. LCM = any one of the no’s P = #$

d. GCD = 1

$
e. No.of n0 = 7 = #$ = $

96
Sum no.of chances

1 0

2 1

3 2

4 3

5 4

6 5

7 6

8 5

9 4

10 3

11 2

12 1

13 0

(only for two dies)


%
f. Find probability of sum at heat 9 = #$
&
#
or 10 or 11 or 12
%
4 3 2 1 = #$ =

THREE DICE:-
t = n(s) = 63 = 216
$
a. Find probability of getting all equal no P = = #$
$

%
b. Probability of gelling all diff no. P =
$

Favorable chances = 6 × 5 × 4 = 120

% #
c. All not equal no. P = = #$
$
(1 – equal)

d. Find the probability of getting sum = 16

97
(6, 6, 6) - 18 - max lw 2
#!
(6, 6, 3) - =3
!
#!
(6, 5, 5) - =3
!
$
P = =
$ #$

Num – 15 6, 6, 6 - 18 max
#!
6, 6, 3 - !=3
6, 5, 4 - 3! = 6
%
5, 5, 5 - 1! = 1 P=
$
10

Num – 14 6, 6, 6 - 18 max
#!
6, 6, 2 - ! =3
6, 3, 5 - 3! = 6
#!
6, 4, 4 - ! =3
#!
5, 5, 4 - !
=3 P= $

Num No.of term

3 1

4 3

5 6

6 10

7 15

8 21

13 21

14 15

15 10

16 6

17 3

18 1

98
1. Biased die is such that the probability of getting 2 = half of the probability of getting
any one of the other no. If the die is rolled once find the probability of getting even
no.


P(2) = P(3) = P(4)= P(5) = p(6) = x

P(1) = _ _ _ _ _ P(6) = 2x

P(2) = x


P(E) = E is even = + + =

PROBLEMS ON CARDS:-
52

26 Blacks (colures) 26 reds

13 clubs 13 spades 13 diamond 13 hearts


(suites)

13 2, 3, 4, 5, 6, 7, 8, 9, 10 j, q, k, a

No. cards hour cards


Face cards = J, Q, K

2. SINGLE CARD:-
If a card is drawn from a well shuffled pail of 52 cards. Find the probability of getting.
a. Black card P(E) =
q (s!
= = =
t!

&
b. Spade no. card =

$
c. Black no. card = =
k b bk
! $ 3
d. King or black card = =

99
#$ $ !
e. No.of or red face card = = =
$

$ $3 #!
f. Honor card or red card = =

TWO CARDS:- If 2 cards from a well shuffled pack of 52 cards. Find the probability of
getting.
!t.
a. Both king -
u.
!t. × !t.
Successively with replacement =
t. × t.
!t. × !#t.
Successively without replacement =
t. × t.

#$t. × t.
b. One no. card, one face card S =
t,
#$t. × t.
S with R =
t. × t.
#$t. × t.
S with out R =
t. × t.

3. If 4 cards are drawn from a well shuffled path of 52 cards find the probability of
getting all cards from differed suits
#t. × #t. × .vt. × .vt.
=
tw
S with R = 13u × 13u × 13u × 13u

u. ##u. $u. #u.


= u.
× u.
× u.
× u.

u. × ##u. × $u. × #u.


S without R =
u. × u. × %u. × !&u.

4. If 3 cards are drawn from a well shuffled of 52 cards similloreouly then find the
probability of getting all doffing suits.

#u. × #u. × #u.



uv

4x# this is become in 4 suits we want 3 suites.

5. A ling contains 4 red balls 5 green balls 6 black balls. It 3 balls are drawn from the bag
simultaneously then find the probability of getting.
a. All side of same color.
b. All are of different color.
c. All-out 1 green.
d. Exactly 2 green.
3 !uv uv $uv
a. u v

100
!u. × u. × $u.
b.
uv

c. All-out green = 1- no green

%uv
=1-
uv
u, × %u.
d. uv

PROBLEMS ON CALENDARS:- 365 days 52 weeks + 1 odd day.


Jan 1 - Dec 30 Dec 31
Mon Sun Mon
52 wks.
Jan 1 53 times 52 times

Mon Mon Remain days

Tue Tue „

Wed Wed „

Thu Thu „

Fri Fri „

Sat Sat „

sun sun „

1. 53 Mondays = K
$
2. 52 Monday = K
3. 53 Mon, 53 Tue = 0
4. 53 Mon, 52 Tue = K
5. 52 Mon, 52 Tue = K

Ly:- 366 days = 52 weeks + 2 days


Jan 1 Dec 29 Dec 30 Dec 31
Mon Sun Mon Tue
52 weeks
Jan 1 53 lines 52 times
Mon Mon, Tue remaining
„ „ „
101
a. 53 Mondays =
K

b. 52 Mondays = K

c. 53 Mon, 53 Tue = K (side by side)

53 Mon, 53 Thu = 0 (not side by side)

d. 53 Mon, 52 Tue = (side by side)


K

53 Mon, 52 Thu = K (not side by side)

!
e. 52 Mon, 52 Tue = K (side by side)

#
52 Mon, 52 Thu = (not side by side)
K

6. GENERAL PROBLEMS:-
5B & 5G - 6 Persons

5! × 6!
! ×$! ×! ×# × ×
P= = =
%! % ×& × ×K %

7. P(A) = # then P(B) if


:) A, B are collectively exhaustive & mutually exclusive events.
P(A ∪ B) = P(A) + P(B) – P(A ∩ B)
1 0
(collectively exhaustive) (mutually exclusive)
1 = P(n) + P(B)

P(B) = #

::) A, B are collectively exhaustive & independent event


P(A ∪ B) = P(A) + P(B) – P(A ∩ B)
z
1=# P # + P(B) – P(A) × P(B)

1=#+x–(#×x)

1 = # + x -# x

1-#= #
x

102
x=1
8. The probability of getting a job by A is 0.7 { that of B is 0.6. If both attended an
interview then find the probability that
a. Both can get the job.
b. Exactly one gets job.
c. No job.
d. at least 1 job.
a. Both get job
P(A) × P(B) = 0.7 × 0.6 = 0.42
b. Exactly one get job.
P(A) = 0.7 P(|̅) = 0.8
P(B) = 0.6 P(~•) = 0.4

P(A) P(~•) + P(|̅) P(B) = O.7 × 0.4 + 0.6 × 0.3 = 0.46


c. No job = P(|̅) P(~•) = 0.12
d. already one job = 1 – no job
= 1 – 0.12
= 0.88

9. A speaks truth in 70% of cares & B speaks truth in 80% of the cares in slating a fast
find the probability that.
a. Both coincide
b. Both contradict
P(A) = 0.7 P(B) = 0.8
̅
P(|) = 0.3 P(~•) = 0.2
a. Both coincide = P(A) P(B) + P(|̅) P(~•)
= 0.56 + 0.06 = 0.62

b. P(A) P(~•) + P(|̅) P(B) = 0.38 (0.14 + 0.24)


c.

10. The odds in favors of solving a problems by A is 4 : 3 & odds against B is


2 : 5. If a problem is green to A, B find odds in favors of not solving other problem by
both of them.
P(|̅) = K
! #
(A) f : u = 4 : 3 P(A) = K
(B) u : f = 2 : 5 P(B) = K
P(~•) = K
P(|̅) P(~•) =
$ $
× K = =
K && $ &#

Odd in favors of no, solving = 6.43

11. A question paper writers 5 questions with 2 internal choice. If a student attends the
exam. Find the probability that he answers at least 1 questions?

= 1 – no answer
103
= 1 - ( )2

=1-
#
!
=
!#

12. Bag A contains 4 bank balls, 6 red balls bag B contains 7 black balls, 3 red balls. If a
ball is children at random from the bags. Find the probability that of is a black color.
I z
!z, $€ Kz, #€
P(A) = P(B) = (as then are 2 bags)
! K
P(black) = P(black) =
% %
I ! z K
P( z ) × P( ) ×
% z %
K
P( black) = + =
% % %


z* O(„) × O( †‡ )
I … †
O(„) O‚ †‡ ƒ O(ˆ). O( †‰ )
P( ) =

z* z
P( I ) = 0.4 P ‚ z* ƒ = 0.7

Fair unfavorable
z* %.! %.! !
P( I ) = %.! %.
= .
=

104
AVERAGE
The concept of average is equal distribution of the overall value among all the things or
persons present there. So the formula for finding the average is as follows
2Š‹ Œ• ŒŽ••‘’“”•ŒN•
Average =
–7 — ( q — (8

Eg: 10, 46, 60, 20


% !$ $% %
Average =
!

= 34

Consecutive numbers:
x, x+1, x+2, x+3, x+4 …….
Eg: 90, 91,92,93,94
Average = 92
Note:
If the numbers are in consecutive manner then the average is middle number or (first
number + last number)/2
Consecutive odd numbers:
x, x+2, x+4, x+6,x+8 ……
Eg; 91, 93,95,97,99
Average = 95
Consecutive even numbers:
x, x+2, x+4, x+6, x+8 ……
Eg: 92,94,96,98,100
Average = 96

Take equations when three consecutive numbers are given → x-1,x,x+1


Take equations when three consecutive even no.are given → x-2, x, x+2
Take equations when three consecutive odd no. are given → x-2, x, x+2

105
Basic Questions
1. The average age of A,B and C is 26 years, if the average age of A and C is 29 years,
what is the age of B in years?

I z u I
= 26 = 29
#

A+B+C = 78 A+C = 58

B = 78 – 58 = 20

2. The average of 7 numbers is 5. If the average of first six of these numbers is 4, the
seventh number is?

Total of 7 numbers is (7 * 5) = 35
Total of 6 numbers is (6 * 4) = 24
∴ Seventh number is 11

3. The average of 10 numbers is 7. What will be the new average if each of the number
is multiplied by 8?

Total of 10 numbers is (10 * 7) = 70


70 * 8 = 560

$%
%
= 56

4. A, B, C and D are four consecutive even numbers respectively and their average is 65.
What the product of A and D

A B C D

62 64 65 66 68

So product of A and D is 62 * 68 = 4216

5. A, B, C and D are four consecutive odd numbers respectively and their average is 42.
What the product of B and D

A B C D

39 41 42 43 45

106
So product of A and D is 41 * 45 = 1845

6. Of the three numbers. The first is twice the second and the second is thrice the third.
If the average of the three numbers is 10. The numbers are:

Third = x second = 3x first = 6x

$ #
= 10
#
%
=10
#
X= 3
∴ The numbers are 18, 9, 3

7. The sum of five numbers is 555. The average of the first two numbers is 75 and the
third number is 115. What is the average of the last two numbers?
A+B+C+D+E = 555
I z
= 75

A+B = 150 and C = 115


150+115+C+D = 555
C+D = 290
So average of last two numbers is (290/2) = 145

8. The average expenditure of a man for the first five months is Rs. 3600 and for next
seven months it is Rs.3900. if he saves Rs. 8700 during the year, his average income
per month is
Total exp of first five months = 3600 * 5 = 18000
Total exp of next seven months = 3900 * 7 = 27300
18000+27300+8700 = 54000
So average income per month is 54000/12 = 4500

9. The sum of three numbers is 98. If the ratio between first and second be 2 : 3 and
between second and third be 5 : 8, then the second number is

a+b+c = 98
a:b=2:3 b:c=5:8

107
∴ a : b : c = 10 : 15 : 24
So 49 parts = 98
1 part = 2
Second number 15 parts = 30

Based on Equation
10. The average of marks obtained by 120 candidates was 35. If the average of marks of
passed candidates was 39 and that of failed candidates was 15, the number of
candidates who passed the examination is :

Total marks = 120 * 35 = 4200

120

X (120 – x)
Passed failed

4200 = x * 39 + (120 – x) * 15
4200 = 39x + 1800 – 15x
2400 = 24x
X = 100

11. In a school, the average age of students is 6 years and the average age of 12 teachers
is 40 years. If the average age of the combined group of all the teachers and the
students is 7 years, then the number of students is:
Teachers = 12 students = x
(12 + x) * 7 = 12 * 40 + 6 * x
84 + 7x = 480 + 6x
X = 480 – 84
X = 396

12. The average monthly salary of all the employees in an industry is Rs. 12000. The
average salary of male employees is Rs. 15000 and that of female employees is Rs.
8000. What is the ratio of male employees to female employees?
Male = x female = y
(x + y) 12000 = x * 15000 + y * 8000

108
12x +12y = 15x + 8y
4y = 3x
X/y=4/3

13. In a school with 600 students, the average age of the boys is 12 years and that of the
girls is 11 years. If the average age of the school is 11 years and 9 months, then the
number of girls in the school is:
600

X 600 – x
Boys girls

600 * 11 9/12 = x * 12 + (600 – x) * 11

7050 = 12x+6600-11x
X = 450
Number of girls = 600 – 450 = 150

14. The average salary of all the staff in an office of a corporate house is Rs. 5000. The
average salary of the officers is Rs. 14000 and that of the rest is Rs. 4000. If the total
number of staff is 500, the number of officers?
500

X 500 - x
Officer rest officer

5000 * 500 = x * 14000 + (500 – x) 4000


X = 50

109
True/False Average

15. The mean of the marks obtained by 100 students is 60. If the marks obtained by one
of the students was incorrectly calculated as 75, whereas the actual marks obtained
by him was 65, what is the correct mean of the marks obtained by the students?
100 * 60 = 6000
A.M W.M
65 75
+10
(6000 – 10)/100 = 59.9

16. A mathematics teacher tabulated the marks secured by 35 students of 8th class. The
average of their marks was 72. If the marks secured by Reema was written as 36
instead of 86 then find the correct average marks up to two decimal places
35 * 72 = 2520
A.M W.M
86 36
-50
(2520 + 50)/ 35 = 73.43

17. The average of marks 14 students was calculated as 71. But, it was later found that
the marks of one student had been wrongly entered as 42 instead of 56 and of
another as 74 instead of 32. The correct average is
14 * 71 = 994
A.M W.M
56+32 42+74
88 116
+28
(994 – 28)/14 = 69

110
18. The average marks in science subject of a class of 20 students is 68. If the marks of
two students were misread as 48 and 65 of the actual marks 72 and 61 respectively,
then what would be the correct average?
20 * 68 = 1360
A.M W.M
72+61 48+65
133 113
- 20
(1360+20)/20 = 64

19. The average marks in English subject of a class of 24 students is 56. If the marks of
three students were misread as 44,45 and 61 of the actual marks 48,59 and 67
respectively then what would be the correct average?
24 * 56 = 1344

A.M W.M
48+59+67 44+45+61
174 150
-24
(1344+24)/24 = 57

Replacing a person

20. The average age of a committee of 8 members is 40 years. A member, aged 55 years,
retired and he was replaced by a member aged 39 years. The average age of the
present committee is
8 * 40 = 320
(320 – 55 +39 )/8 = 304/8
= 38

21. The average weight of 3 men A,B and C is 84kg. Another man D joins the group, and
the average weight becomes 80kg. if another man, E, whose weight 3kg. more than
that of D, replaces A, then average weight of B,C,D and E becomes 79kg. the weight
of A is

(A+B+C)/3 = 84 (A+B+C+D)/4 = 80
A+B+C = 252 A+B+C+D = 320

111
D = 320 – 252 = 68
E = 68 +3 = 71
(B+C+D+E)/4 = 79
B+C+D+E = 316
B+C+68+71 = 316
B+C = 177
A+177 = 252
A = 75

Including/Excluding

22. The average weight of 21 boys was recorded as 64kg. If the weight of the teacher
was added the average increased by one kg. What was the teacher’s age?

Boys Boys + teacher


(21 * 64) ~ (22 * 65)
1344 ~ 1430

76

23. The average age of 14 girls and their teacher’s age is 15yr. If the teachers age is
excluded then the average reduced by one kg. What was the teacher’s age?

Teacher + girls girls


(15 * 15) ~ (14 * 14)
225 ~ 196

29

24. The average age of 5 members of a family is 25yr. If the servant of the family is
included the average age increased by 40%. What is the age of the servant?

Family F +S
(5 * 25) (6 * 35) 25+(25 * (40/100)) = 35
125 ~ 210

85

25. The average age of the class is 35yr. 6 new students with an average age of 33 yr.
joined in that class, thereby decreasing the average by half year. The original
strength of the class was?

(x * 35 + 6 * 33)/(x+6) = 34.5

112
X = 18
Average speed
note
If the certain distance is covered at the speed of x km/hr and the same distance is
covered at y km/hr. then average speed during entire journey is (2xy/x+y)

26. A man goes to a certain place at a speed of 30km/hr and returns to original place at a
speed of 20km/hr, find out the average speed during the entire journey
(2 * 30* 20 /50) = 24km/hr

113
MIXTURES & ALLEGATIONS
Mixture:- It is the combination of two or more quantities which are of the same quantity
(or) different quantities.

Allegation:- It is the process of mixing to obtain the required quality of mixture.


Allegation rule:-
type -1 type - 2

Result

(diff res, type- 2) : (diff res, type – 1)


Quantity of type – 1 Quantity of type – 2
Res, type – 1, 2 are represents quality

1. In what ratio a rice costing 47/- per kg to be mixed with another variety of rice
costing 31/- per kg to obtain a rice of cost Rs 37/- per kg.
47 31

37

6 : 10
3 : 5
2. In what ratio 79% alcohol be mixed with 43% alcohol to obtain 54% alcohol.
Alcohol Water
79 43 21 57

54 46

11 : 25 11 : 25

114
3. In what ratio 93.5% concentrated alcohol be mixed with water to obtain 73%
concentrated alcohol.
Alcohol
93.5 0

73

73 : 20.5
730 : 205 = 146 : 41

4. In what ratio rectified spirit be mixed with flavored water to obtain an alcoholic
beverage of 42.8% of alcohol 98.9% alcohol.
Alcohol
98.8 0

42.8

42.8 : 56
428 : 560
107 : 140

5. In which ratio an edible oil of cost 130/- per liter be mixed with another variety of
cost 180/- per liter. In such a way that by selling resultant at Rs. 165 per liter means a
profit pf 10%.
130 180 110 : 165
100 : 150

150

30 : 20
3 : 2

115
6. How many kgs of sugar variety Rs. 42/- per kg mixed with another variety of 25 kg of
sugar costing 24/- per kg in such a way that by selling the mixture at Rs. 40./- per kg
make a profit of 25%.

42(?) 24 (25 kg) 120 = 40


100 = 32
32

8 : 10 5 – 25 kg
4 : 5 4 – 20 kg

7. 3 vessels contains milk & water solution in which milk & water are in the ratio 5 : 2,
4 : 3 & 6 : 1. A mixture is formed by drawing 3lt from 1st, 4th from 2nd, 5lt from 3rd,
what ratio of milk and water in the mixture.
5(3) + 4(4) + 6(5) : 2(3) + 3(4) + 1(5)
15 + 16 + 30 : 6 + 12 + 5
61 : 23 (or) % of milk
$ $ ×
!
× 100 = = = 72.6%

8. 3 vessels contains milk and water in the ratio 5 : 1, 7 : 2, 7 : 5 these 3 solution are
mixed in the ratio 1 : 2 : 3. What is the ratio of milk & water in the resulting solution?
Here the 5 : 1 = 6p, 7 : 2 = 9p, 7 : 5 + 12p not same to equal ate
5:1 7:2 7:5 Lcm
6 9 12 6, 9, 12 = 36
30 : 6 28 : 8 21 : 15
30 (1) + 28 (2) + 21 (3) : 6 (1) + 8 (2) + 15 (3)
30 + 56 + 63 : 6 + 16 + 45
149 : 67

9. A cast contains 150lt of pure milk 30lt of milk is removed & replaced with water. This
process is repeated for 2 more times. What is the quantity of pure milk on the
resulting solution?
#%
30l 30 l 30 l of 150 l = × 100 = 20%
%
% % %
150 × × × = 76.8
%% %% %%

-20% -20% -20%


150 lt 120 96 76.8

116
10. A cask contains 40 lt milk solution in which 10% is water. How many lts of water
must be added to make 20% water in the resulting solution?
After water is added
Water 20p

40 lt 100 P
36 M 4W milk 80 p
80 p = 36
100 p = 45
Milk = 45, 5 lt water is added.
Milk
90 0 sp = 40
80 ip = 52
Or
!
80 10 =!
#$

16 + 4x = 36
X=5
11. A cask contains 500 ml of milk solution in which 10% is milk. How much water must
be added to make 8% milk in the resulting solution?

8pm
500 ml 100 p

92 p w

450w 50m 8 p = 50 lt
100 p = 625 ml 125 ml
Or

117
%
= = 10 0
! % & #

1150 = 900 +2x 8


2x = 250 ⇒ x = 125 8 2
4 : 1
4p = 500
1p = 125

118
NUMBERS

♠ TYPES OF NUMBERS: - Natural numbers (N) = {1, 2, 3, 4, ---------∞} only + ve.


Whole number(s) (W) = {0, 1, 2, 3, 4, ---------- ∞} + ve with ‘0’.
Intezers z (or), I = {∞ ---- -3, -2, -1, 0, 1, 2 -----∞} + ve1 – ve with ‘0’.
Natural numbers also called counting numbers.

REAL NUMBERS
♠ Rational Number with approximate value
Any fraction from p/q (q ≠ 0) √17o, √31, √2, √3, 2 + √3š, e any symbols.
1/4, 6, -4, 0, √16, 0.3, 0.2727, 22/7
Convers 0.3 into p/q form
0.3 = 3/9
0.29 = 29/99
1.29 = 1 29/99
0.38 = (38 – 3)/90 = 35/90
0.124 = (123 – 12)/900 = 111/900

♠ PRIME NUMBERS: - (Exactly 2 factors – 1 and it self a counting numbers is called


prime number it has exactly two factors.
2 3 5 7 61 67
11 13 17 19 71 73 79
23 29 83 89
31 37 97
41 43 47
53 59
2, 3, 5, 7, 11, 13, 17, 19, 23, 29, 31, 37, 41, 43, 47, 53, 59, 61, 67, 71, 73, 79, 83, 89,
97
There are 25 prime numbers below 100.
All prime numbers except 2 are odd.

119
2 is the only even prime number.
Any prime number starting from 5 divided by 6 gives the remainder 1 (or) 5.

Concept:
221 is prime (or) not?
221 – (15)2 near
2, 3, 5, 7, 11, 13 221 ÷ 13 = 17 13 × 17 = 221(not prime)

161 169 (13) 2


2, 3, 5, 7, 11, 7× 23 = 161
Logic: 221 a2 - b2
= (15) 2 – (b) 2= (15 + 2) (15 – 2) = 17 × 13
(161)2 =152 - 82
= (15 + 8) (15 – 8) = 23 × 7
Any number which can be converted into a2 - b2 will not be a prime number.
♠ COMPOSITE NUMBERS: - A counting number it has more than 2 factors.
4 – 1, 2, 4 6 – 1, 2, 3, 6 8 – 1, 2, 4, 8
1 is neither prime nor composite number.
Model 1: To find numbers of factors factorial L.C.M form
• Find the No.of factors for 40
2 40 23 × 51 = stander form No.of factors
2 20 = (3 + 1)(1 + 1) = 4 × 2 = 8
2 10
5
Without it self = 8 – 1 = 7
Without 7 = 7
Without 1 & itself = 8 – 1 = 6
ap × bq × cr when a, b, c are prime numbers then No.of factors
= (p + 1) (q + 1) (r + 1).
Find No.of factors for 1296
24 × 34 = 5 × 5 = 25

120
Model 2: The No.of ways to represent a number as a product of two factors.
• Note 1: If the No.of factors is even no. then No.of ways
= No.of factors/2.
• Note 2: If the No.of factors is odd the no. then No.of ways
= No.of factors/2
♠ If No.of factors is an odd number. Then the given number is perfect square.
1296 has 25 factors (36) 2
No.of ways = (25 + 1)/2 = 13
No.of different ways = 13 – 1 = 12
♠ No.of ways to represent 784 as a product of two factors.
784 = (28) 2 = (2 × 2 × 7) 2 = 24 × 72 = 5 × 3 = 15
No.of factors = 15
No.of ways = (15 + 1)/2 = 8
No.of different factors = 8 – 1 = 7
What least number should be multiplied with 756 to make it a perfect square?
756 = 22 × 33 × 7= 7 × 3 = 98
Model 3: The product & sum of all factors of a number 6 1, 2, 3, 6.
Sum = 1 + 2 + 3 + 6 =12
Product = 1× 2 × 3 × 6 = 36 = 62
8 1, 2, 4, 8
Product = 1 × 2 × 4 × 8 = 64 = 82
12 1, 2, 3, 4, 6, 12 (12) 3
6 - 6 No.of factor/2
8 - 8 factor/2
12 – 12 factor/2
For 16 No.of factors – 5
Product of factor = (16) 2 = 42 × 5/2 = 55
If No.of factors is ‘d’. The product of all factors = (No) d/2
Find the product of all factors for 180.
180 =22 × 51 × 32 =
No.of factors = 3 × 2 × 3 = 18

121
Product = (180)?
♠ SUM OF FACTORS :- If ap × bq × cr is standard from
Then sum = {(a p +1 – 1)/a - 1 × (b q +1 - 1)/(b – 1) × (c r + 1 – 1)/(c – 1)
Sum means sum of all factors
12 = 2 2 × 3 1

= (2 3 -1)/1 × (3 3 – 1)/2 =

= 7 × 8/2 = 28

Sum of all factors for 140

140 = 7 × 5 × 2 2

= (2 3 – 1)/1 × (7 2 – 1)/6 × (5 2 × 1)/4

= 7 × 48/6 × 24/4

= 7 × 8 × 6 = 336

Model 4:- The No.of ways to represent a number as a product of co primes.

If a standard form is ap × bq × cr with 3 bases a, b, c then the No.of ways as a product


of co-primes = 2n-1 where (n) = No.of bases.

1. No.of way for 120 – as a product of co-primes

120 = 2 3 × 3 1 × 5 1

Bases = 2, 3, 5

No.of ways = 2 3 -1 = 2 2 = 4

Model – 5:- No.of co-primes of a number N which are less than N No.of co-primes
below 10 and co-primes with 10.

1, 2, 3, 4, 5, 6, 7, 8, 9, 10

No.of co-primes = 4

If a standard form ap × bq × cr

No.of co-primes = N × [1 – 1/2] [1 – 1/b] [1 – 1/c]

122
= 10[1 – 1/2] [1 – 1/5]

= 10 × 1/2 × 4/5 = 32

Model 6:- The sum of all co-primes of number ‘N’ sum of all co-primes =

N/2 × No.of co primes

= N/2 × N [1 – 1/a] [1 – 1/b] -------

Find the sum of all co primes for 30 = 30/2 × 30 [1 – 1/2][1 – 1/3][1 – 1/5]

= 15 × 30 × 1/2 × 2/3 ×4/5 = 120

1. Number of zero’s & maximum powers in factorial No.of 5’s = No.of zeroes
In a factorial – when divided by 5 successively it given No.of zeros.

2. Find No.of zero’s at the end of 72!


72! = 72/5 = 14/5 = 2 14 + 2 = 16 16 zero’s
64! = 64/5 = 12/5 = 2 12 + 2 = 14

Maximum powers:-
3. Find the maximum powers of 2 in 25!
25! = 25/2 = 12/2 = 6/2 = 3/2 =1 = 22
Whenever number minimum powers are needed we must divisible the number
with factorial that number.

4. Maximum power of 3 in 100!


100/3 = 33/3 = 11/3 = 3/3 = 1 = 48

5. Find the maximum powers of 6 in 10!


8
6=2×3 101/2 = 5/2 = 2/2 =1
4
101/3 = 3/3 =1
4

6. Find minimum power of 24 in 32!

123
24 = 23 × 3
32/2 = 16/2 = 8/2 = 4/2 = 2/2 = 1 21/3 =10 (23 so that)
10
32/3 = 10/3 = 3/3 =1 14 (24)
7. Find maximum of 12 in 32!
12 = 22 × 3
32/2 = 16/2 = 8/2 = 4/2 = 2/2 =1 31/2 = 15
32/3 = 10/3 = 3/3 =1 14 1214
Maximum power of 12 in 32! = 1214
8. Last digit (or) last two digit.
2 - 21 = 2, 22 = 4, 23 = 8, 24 = 6, 25 =2
2 has reputation cycle with 4 units. Last digit in any power depends on last digit
in the number.
(42)12378= (123789)/4 3094 2
Last digit will be 4.
Like that every number from 1 – 9 will have a reputation cycle. This gives the last
digit in any power of number.

REPITATION CYCLE
2(4) = 2, 4, 8, 6
3(4) = 3, 9, 7, 1
4(2) = 4, 6
7(4) = 7, 9, 3, 1
8(4) = 8, 4, 2, 6
9(2) = 9, 1
0, 1, 5, 6 last digits are 0, 1, 5, 6 respectively
100
(100) = 0
(125)33 = 5
(126)127 = 6

9. The last digit in 256 × 4357 × 1234 × 759 × 643


=6×7×4×9×3
Last digit = 6
10. Last digit in (2)421 × (3)526 × (9)439
=2 × 9 × 9=2
11. Last digit in (57867)192567 – (452)876
3 – 6 = 7 last digit
12. (2)156 × (3)729 ×(5)146 × (6)729 × (9)846
Last digit = 0 because 5 × odd =5 5 × even = 0
Last digit (3)729 ×(5)146 × (6)729 × (9)846
Last digit = 5

LAST TWO DIGITS:-


124
(0)100 0 0 * zero power any no. last two digits.
(0)75 0 0
5 as a last digit even powers 2 5
5 as a last digit odd powers 7 5
Ex:- (635)94 = 2 5
(675)99 = 7 5
• If the number ends with even number (2, 4, 6, 8) then the cycle repeat after
20 steps. The 20th power will ends with 7 6
• If the number ends with odd number ending with (1, 3, 7, 9) then the cycle
repeats after 20 steps the 20th power will end with 0 1
• If the last two digit in a number are 7 6 then what even may be the
power values the last two digits will be 7 6

Ex:-1. (154)42 last two digits


42/20 = 2 (50 + 4) 2 = 1 6

Ex:- 2. (169)673 42/20 = 2 (50 + 14)2 = 1 9 6

TEST OF DIVISIBILITY

(a) Divisibility rules


Basic rules from 2 to 11
Rule for 2 last digit

21 – 2 0/ last digit ÷ by 2
2
22 – 4 00/ last two digit ÷ by 4
4
23 – 8 000/ last three digit ÷ by 8
8
24 – 16 000/ last four digit ÷ by 16
16

125
48794196 ÷ 2

÷4

÷8×

÷ 16 ×

DIVISIBLE RULE FOR 5 LAST DIGIT

51 – 5 0/
5
52 – 25 00/
25
53 – 125 000/
125
46839375 ÷ 5
÷ 25
÷ 125

DIVISIBLE RULE FOR 3


31 – 3 SUM/3 (num > 3)
32 – 9 SUM/9 (num > 9)
33 – 27 SUM/27 (num > 27)

DIVISIBLE RULE ROR 11


18 18
4 1 6 8 2 9 6 4 4 6 3 8
18 7
18 – 18 = 0 18 – 7 = 11/11
NO = 11 No = 11

126
• Difference b/w sum of even no places and odd no places should be
zero (or) divisible by 11.

DIVISIBILITY RULE FOR 7 LAST DIGIT DOUBLE AND SUBTRACT

154 4×2=8 23714 2371


15 ___8
8 236
7 ___6
230 (23)
• Divisibility rule for 7 last digit double and add.
DIVISUBILITY RULE FOR 6
6 2×3 36 4×9
12 4×3 30 10 × 3
15 5×3 5×6

18 2×3×5

(101 – 1) 9 46 53 _4 + 6 + 5 + 3_ = 18
9 9
(102 – 1) 54 45 (45 + 54)/9 = 99/99
01 54 45 45 + 54 + 1 = 100/99 ×
(103 – 1) abcdef (def + abc)/999
(101 – 1)11 4 6 5 3
(102 – 1) 101 64 56 49 (64 + 49 – 56)/101 ×
23 47 61 (61 + 23 – 47)/101 ×
(103 – 1)(1001) three digits
Example:
676 474 237 ÷ 1001
676 + 237 = 913 439 ÷ 1001 ×
-774 Remainder is 439.
43

127
NUMBER SERIES
PRIME NUMBER SERIES
1. 7, 11, 13, 17, 19, ?, 23
Prime number series after 10 23 is the prime number.

2. 3, 7, 17, 31, ?, 53

, # &, #, & #K, ! , !#, !K


53
# !

Prime numbers
Addition Series
3. 2, 8, 14, 20, ?, 26

6 6 6 6

4. 2, 6, 11, 17, 23, ?, 36

4 5 6 7 8

Difference Series
5. 200, 192, 184, 176, ? 168 200 – 8 = 192
192 – 8 = 184
-8 -8 -8 -8 184 - 8 = 176
176 – 8 = 168
Multiple Series
6. 6, 12, 36 144 ? 720 6 × 2 = 12
12 × Q = Q›
×S ×Q ×U ×V 36 × U = TUU
144 × V = œSW

7. 6, 24, 96, 392, ? 1536 6 × U = SU


24 × U = o›
× U × U × U × U 96 × U = Q•U
384 × U = TVQ›
Division Series
8. 4500, 900, 180, 36, ? 7.2 4500 ÷ V = oWW
900 ÷ V = T•W
÷5 ÷V ÷V ÷V 180 ÷ V = Q›
36 ÷ V = œ. S

128
× œ × › × V × U × Q

9. 15120, 2160, 360, 72, 18, ?

÷ œ ÷ › ÷ V ÷ U ÷ Q
6 × Q = T• 15120 ÷ œ = ST›W
18 × U = œS 2160 ÷ › = Q›W
72 × V = Q›W 360 ÷ V = œS
360 × › = ST›W 72 ÷ U = T•
2160 × œ = TVTSW 18 ÷ Q = ›

YS Series
10. 1, 4, 9, 16, 25, 36, 49, ? 64

TS SS QS US VS ›S œS •S

11. 121, 144, 169, 196, ? 225

TTS TSS TQS TUS TVS

(YS + T) Series
7 9 11 13 15
12. 10, 17, 26, 37, 50, ? 65

QS + T QS + T VS + T ›S + T œS + T •S + T

10 – 17 – 7 gap
17 – 26 – 9 gap
26 – 37 – 11 gap
37 – 50 – 13 gap
50 – 65 – 15 gap
Increasing 2

13. 122, 145, 170, ?, 226, 257 196

TTS + T TSS + T TQS + T TUS + T TVS + T T›S + T

YQ Series
14. 8, 27, 64, 125, ?, 216

SQ QQ UQ VQ ›Q

YQ + T Series
15. 65, 126, 217, 344, ? 513

129
QQ + T VQ + T ›Q + T œQ + T •Q + T

YQ − T Series
16. 0, 7, 26, 63, 124, ? 215

TQ − T SQ − T QQ − T UQ − T VQ − T ›Q -1
YQ + Y Series
17. 64040, 27030, 8020, ?, 1010

UWQ + UW QWQ + QW SWQ + SW TWQ + TW

YQ − Y Series
18. 0, 6, 24, 60, 120, ? 210

TQ − T SQ − S QQ − Q UQ − U VQ − V ›S − ›
Alternating Series
+4 +4 +4
19. 15, 14, 19, 11, 23, 8, ? ? 27, 5

- 3 -3 -3
÷S ÷S ÷S

20. 50, 200, 100, 100, 200, 50, 400, ?, 800 25


×S ×S ×S ×S
Previous Year questions
1. 8, 17, 30, 47, 68, ?, 93

9 13 17 21 25

8 + 9 = 17
17 +13 = 30
30 + 17 = 47
47 + 21 = 68
68 + 25 = 93
Increasing by 4

2. 24, 12, 12, 18, ?, 90 36


× × 1 × T. V × S × S. V
24 × = 12

130
12 × 1 = 12
12 × 1.5 = 18
18 × 2 = 36
36 × 2.5 = 90

3. 5, 16 49, 104, ?, 280 77

11 33 55 77 99
Difference 111’ multiple
5 - 16 = 11
6 - 49 = 33
49 – 104 = 55
104 – 181 = 77
280 – 181 = 99

4. 13, 19, 30, 48, 75, ? (75 + 38) =113

6 11 18 27 30

5 7 9+2 11

2 + 2 3 + 2 4 +2 5 + 2 6 + 2
5. 125, 128, 119, 146, 65, ? (243 + 66) = 308
3 -9 27 -81
3 −3 3# - 3! 3

6. 4, 10, 27, 112, 555, ? 3336

4 × S + S = TW
10 × Q − Q = Sœ
27 × U + U = TTS
112 × V − V = VVV
555 × › + › = QQQ›

7. 15 22 36 64 ? 232 120
7 14 28 56 112

8. 12 19 35 59 90 ? (90 + 37) = 127

7 16 24 31 37

131
9 8 7 6

9. 19 23 14 30 5 ? 41

+4 -9 + 16 - 25 + 36

10. 16 7 5 6 15 ? 66
16 × T − o = œ
7 × S − o = V
5 × Q − o = ›
6 × U − o = TV
15 × V − o = ››

11. 22 10 8 12 40 ? 304
22 × − 1 = 10
10 × T − S = •
8 × S − U = TS
12 × U − • = UW
40 × • − T› = QWU

12. 6 14 39 160 796 ? 4776


6 × S + S = TU
14 × Q - 3 = 39
39 × U + U = T›W
160 × V − V = œoV
796 × › − › = UœœW + ›

13. 29 20 36 11 ? 47

-9 16 - 25 36
3 4 5 6

14. 18 23 36 56 82 ? (82 + 31) = 113

5 13 20 26 31

8 7 6 5

15. 13 21 37 69 ? 261 133

8 16 32 64 128

16. 18 8 6 8 24 ? 176
18 × – 1 = 8

132
8 × T - 2 = 6
6 × 2 - 8 = 8
8 × 4 - 8 = 24
24 × 8 – 16 = 176

17. 5 12 33 136 676 ? 4056


5 × 2 + 2 = 12
12 × 3 – 3 = 33
33 × 4 + 4 = 136
136 × 5 – 5 = 675
675 × 6 + 6 = 4056

18. 11 24 44 70 101 ? 136

13 20 26 31 35
7 6 5 4

19. 20 24 15 31 6 ? 42

+4 -9 +16 - 25 +36
2 3 4 5 6

20. 14 6 4 4 8 ? 48
T
14 × S - 1 = 6
6 × 1 – 2 = 4
4 × 2 – 4 = 4
4 × 4 – 8 = 8
8 × 8 – 16 = 48

21. 50 30 40 75 170 ? 450


T
50 × S + 5 = 30
30 × 1 +10 = 40
40 × 1.5 + 15 = 75
75 × 2 + 20 = 170
170 × 2.5 + 25 = 450

22. 6 9 18 45 135 472.5


6 × 1.5 = 9
9 × 2 = 18
18 × 2.5 = 45
45 × 3 = 135
135 × 3.5 = 472.5

133
PARTNERSHIP
When two or more than two persons run a business, jointly they are called partners and the
deal is known as partnership. It is of two kinds
1. Simple partnership:If the capital of the partners are invested for the same period,
the partnership is called simple.

2. Compoundpartnership:If the capital of the partners are invested for different


lengths of time, the partnership is called compound.
In a group of ‘n’ persons invested different amount for different period then the profit ratio
is:
At1: Bt2 : Ct3 :Dt4 : ………..: Xtn
(Here first person invested amount A for t1period, second person invested amount B for t2
period, and so on.)
Again partner can be sleeping partner (or) working partner
Sleeping partner is one who invests the capital in the business but does not actively
participate in the conduct of business.
Working partner besides investing capital, takes part in running the business. For his
work, he is either paid some salary or given a certain percent of profit, in addition.
Note
Investment (I) ᵡ time period (t) = profit (p)
i.e. I1ᵡ t1 : I2ᵡ t2 : I3ᵡ t3 ....... = p1: p2 : p3 ..........

Problems

1. P and Q started a business investing Rs 8500 and Rs 15000 respectively. In what


ratio the profit earned after 2 years be divided among P & Q respectively?
P Q
Investment 8500 15000
Time 2 years 2 years
Profit Ratio 8500 ᵡ 2 : 15000 ᵡ 2
17 : 30

2. A starts a business with Rs.300000 and B joins him after 3 months with
Rs.1200000. Find the ratio of their profits at the end of the year.

134
A B
Investment 300000 1200000
Time 12 months 9 months
Profit Ratio 300000 ᵡ 12 : 1200000 ᵡ 9
3 ᵡ 12 : 12 ᵡ 9
1:3

3. Ram,shyam and kamal together start a business in partnership. The ratio of their
capitals is 3 : 4 : 7. If their annual profits be Rs.21000. what will be kamal’s share
in this profit?

R S K
3 : 4 : 7

K
Kamal’s share !
ᵡ 21000 = 10500

4. A and B invest in the ratio of 3 : 5 respectively. After 6 months, C enters the


business with the investment of the capital equal to that of B. What will be the
ratio of the profits of A, B and C at the end of the year?

A:B=3:5
A = 3x
B = 5x
C = 5x
A : B: C = 3x ᵡ 12 : 5x ᵡ 12 : 5x ᵡ 6
A : B: C= 6 : 10 : 5

5. A,B and C invested capitals in the ratio of 4:6:9. At the end of the business term,
they received the profit in the ratio of 2:3:5. Find the ratio of their time for which
they contributed their capitals.
5 5 5#
T1 : T2 : T3 = : :
#
#
= : :
! $ &
#
= ᵡ 36 : ᵡ 36 : ᵡ 36
! $ &

= 18 : 18 : 20
= 9 : 9 : 10

135
6. A and B invest in a business in the ratio 3 : 2 . if 5% of the total profit goes to
charity and A’s share is 855 . then the total profit is ?
Suppose total profit = 100 %
= -5%
= 95 %

A B
3 : 2
3x 2x

5x = 95
X = 19
A’s profit 3x = 3 ᵡ 19 = 57
57= 855
%%
Total profit 100 = ? ᵡ 855 = 1500
K

7. Sonu invested 10% more than the investment of mona and mona invested 10%
less than the investment of Raghu. If the total investment of all the three person
is Rs. 5780, find the investment of Raghu
R = 100 %
M = 90 %
S = 99 %
R : M : S = 100 : 90 : 99
%%
Raghu investment = ᵡ 5780
&

= 2000

8. A and B entered into a partnership investing Rs.16000 and Rs. 12000


respectively. After 3 months A withdrew Rs. 5000 while B invested Rs. 5000
more. After 3 more months, C joins the business with a capital of Rs. 21000. The
share of B exceeds that C, out of a total profit of Rs. 26400 after one year by?
A = 16000 ᵡ 3 + 11000 ᵡ 9

136
B = 12000 ᵡ 3 + 17000 ᵡ 9
C = 21000 ᵡ 6
A : B : C = (16000 ᵡ 3 + 11000 ᵡ 9) : (12000 ᵡ 3 + 17000 ᵡ 9) : 21000 ᵡ 6
A:B:C=7:9:6
#
ᵡ 26400 = 3600

9. A, B and C enter into a partnership in the ratio 7/2 : 4/3 : 6/5. After 4 months, A
increases his share by 50%.if the total profit at the end of one year be Rs. 21600
then B’s share in the profit is?
K ! $
A:B:C= :#:
K ! $
= ᵡ 30 : # ᵡ 30 : ᵡ 30

= 105 : 40 : 36
%
A = 105x ᵡ 4 + 105x ᵡ %%
ᵡ8

B = 40x ᵡ 12
C = 36x ᵡ 12
A : B : C = 1680x : 40x ᵡ 12 : 36x ᵡ 12
= 35 : 10 : 9
%
B’s share in the profit = !
ᵡ 21600

= 4000

10. A and B entered into partnership with capitals in the ratio 4 : 5. After 3
months, A withdrew ¼ of his capital and B withdrew 1/5 of his capital. The gain at
the end of 10 months was Rs. 760. A’s share in this profit is:

A = 4x ᵡ 3 +3x ᵡ 7
B = 5x ᵡ 3 +4x ᵡ 7
A : B= 33x : 43x
##
A’s share in the profit = K$ ᵡ 760

= 330

137
11. A, B and C entered into partnership in a business. A got 3/5 of the profit. B and C
distributed the remaining profit equally. If C got Rs. 400 less than A, the total
profit was?
Let total profit x
A = 3x/5
B +C = 2x/5 (x – 3x/5)
B = x/5
C = x/5
3x/5 – x/5 = 400
2x/5 = 400
X = 1000

12. A and B invest Rs. 3000 and 4000 respectively in a business. A receives Rs. 10 per
month out of the profit as a remuneration for running the business and the rest
of the profit is divided in proportion to the investment. If in a year A totally
receives Rs. 390, what does B receives ?
A : B = 3000 : 4000
=3:4
A = 390 – 120 = 270 (profit)
3 parts = 270
B receives 4 parts = 360

13. A, B and C rent a pasture in which A put 10 oxen for 7 months, B puts 12 oxen for
5 months and C puts 15 oxen for 3 months for grazing. If the rent of pasture is Rs.
175. How much C pay as his share of rent?
A : B : C = 10 ᵡ 7 : 12 ᵡ 5 : 15 ᵡ 3
= 14 : 12 : 9
&
C’s value = ᵡ 175
#

= 45

138
14. A starts a business with Rs. 85000. He was joined afterwards by B with Rs. 42500.
For how much period does B join if the profit at the end of 1 year are divided in
the ratio 3 :1?
%%% ᵡ #
! %% ᵡ ¢
=

x = 8 months

15. A starts a business with Rs. 3500 and after 5 months B joins him as his partner.
After a year the profit is divided in the ratio 2:3. What is B’s contribution
# %% ᵡ
=#
ᵡ K

X = 9000

16. A, B and C subscribe Rs. 50000 for a business. A subscribe Rs 4000 more than B
and B subscribe Rs 5000 more than C. out of a total profit of Rs 35000. A receives
how much?
C=x
B = x + 5000
A = x + 9000
A +B +C = 3x+14000 = 50000
X = 12000
A : B : C = 21000 : 17000 : 12000
= 21 : 17: 12

= ᵡ 35000
%

= 14700

17. Three partners A, B and C invest Rs. 6000, Rs.8000, Rs 10000 respectively in
business. A receives 15% of the profit as manager and B receives 10 % of the
profit as assistant manager, after which the remaining profit is divided in
proportion to the capital subscribed by each. Find share of C (in Rs) when A
receives Rs. 1080.
A : B : C = 6000 : 8000 : 10000
=3:4:5

139
Let profit = 100x
A = 15x
B = 10x
(100x – 25x) = 75x
3: 4:5
#
A = 1080 = 15x + ᵡ 75x

X = 32

Share of C = ᵡ 75 ᵡ 32

= 1000

18. A and B started a business by investing 250 and 3000 respectively. At the end of
every month A invested 250 more and B withdraw 250 from his investment. Find
in what ratio the profit should be distributed at the end of the year
A B
250 3000
250 + 11 (250) 3000 – 11 (250)
3000 250
Profit = 1 : 1

19. A and B started a business by investing 150 and 2550 respectively. At the end of
every month A invest 150 more and B withdraw 150 after how many months the
profit is distributed equally
A B
150 2550
150 + x (150) = 2550
x (150) = 2400
x = 16

20. Two partners invested Rs 20000 and Rs 30000 respectively. In a business and
agreed that 60% of the profit should be divided equally between them and the

140
remaining profit in the ratio of their investment. If one partner gets Rs 248 more
than the other, find the total profit made in the business?
60%
A B
30% 30%
40%
A B
20000 30000
2 : 3
5 parts = 40
1 part = 8
A’s profit = 16 (2 * 8)
B’s profit = 24 (3 * 8)
Difference 8 = 248
Profit 100 = ? Rs. 3100

141
ANALOGY
Questions on analogy cover almost all types of relationship (or) commonality that
one can think of and which we generally that one can think of and which we generally
observe in our day – to – day life. Some common types of relationship which frequently
appear in the examinations have been listed below.
1. State and capital relationship.
2. Country and currency relationship.
3. Country and Name of the parliament.
4. Country and its National games/sports.
5. Sex relationship (male and female).
6. Parents and child relationship.
7. Animal and its individual trait.
8. Animal and its habitat relationship.
9. Worker and working place relationship.
10. Tool and action relationship.
11. Raw material and product relationship.
12. Worker and tool relationship.
13. Term and its subject matter.
14. Matter and its state relationship.
15. Instruments and measurement relationship.
16. Quantity and unit relationship.
17. Worker and product relationship.
18. Word and its synonym relationship.
19. Word and its antonym relationship.
20. Part and whole relationship.
21. Word and intensity relationship.
22. Country and national emblem relationship.
23. Disease and causative analogy.
24. Numeral analogy.

Type – 1
1. House: Room:: World : ?
a. Land b. Sun c. Air d. Nation (d)
Room is a part of the house similarly, nation is a part of the world.

2. Botany: Plants: : Entomology : ? (c)


a. Birds b. Plants c. Insects d. Snakes
Study of plants is called Botany. Similarly, Study of insects is called entomology.

3. Mother : Child : : Cloud : ?


a. Shine b. Water c. Rain d. Water (c)

142
Mother gives birth to child similarly, cloud causes rain.
4. Education : Child labour : : Employment : ?
a. Poverty b. Union c. Labour d. Concern (a)
Education is a good solution for child labour similarly employment is a good
solution for poverty.

5. India : Mango :: New Zealand : ?


a. Apples b. kiwi c. Grapes d. Bananas
India is famous for mango. Similarly kiwis are found in new Zealand.

6. Chess : Board :: Roulette : ?


a. Ball b. Wheel c. Coins d. Cards (b)
Chess can be played using square board and routatte will be played using wheel.

7. Fear : Panic :: Provocation : ?


a. Simulate b. Temper c. Angry d. Tension (c)
Fear causes panic in most cases. Similarly provocation causes anger.

8. Quartz : Clocks :: Gypsum : ?


a. Glass b. Metal c. Cement d. Powder (c)
Quartz is a mineral used to make clocks. Similarly Gypsum used to make cement

9. Doctor : Hospital : : Teacher : ?


a. Field b. Laboratory c. School d. Industry (c)
Doctor attends patients in hospital (or) clinic white teacher in school.

10. Christian : Muslim : : ? : Quran


a. Geeta b. Ramayan c. Angel d. Bible (d)
Christian and Muslim represent people of two different religions. Bible is the
holy epic of Christians and Quran is the holy epic for Muslims.

143
Type – 2
From among the given alternatives select the one in which the set of
numbers is most like the set of numbers given in the question.
1. Given set : (3, 7, 15)
a. 2, 6, 10b. 4, 8, 18 c. 5, 9, 17 d. 7, 12, 19 (c)
3+4=7 5+4=9 7 + 8 = 15 9 + 8 = 17

2. Given set : (3, 18, 36)


a. 2, 10, 16 b. 12, 72, 96 c. 4, 24, 48 d. 6, 42, 48 (c)
3 × 6 = 18 and 18 × 2 = 36
Similarly
(4, 24, 48)
4 × 6 = 24 and 24 × 2 = 48

3. Given sit : (7 , 15, 31)


a. 7, 13, 28 b. 5, 13, 28 c. 9, 13, 26 d. 5, 13, 29 (d)
7 15 31 5 13 29
× 2 + 1 × 2 + 1 × 2 + 3 × 2 + 3

4. Given set : (63, 49, 35)


a. 81, 63, 45 b. 64, 40, 28 c. 72, 40, 24 d. 72, 48, 24 (a)
63 49 35 81 63 45
7× 9 7× 7 7× 5 9× 9 9× 7 9× 5

5. Given set : (2, 14, 16)


a. 2, 7, 8 b. 3, 21, 24 c. 4, 16, 18 d. 2, 9, 16 (b)
2 × 7 = 14 3 × 7 = 21
2 × 8 = 16 3 × 8 = 24

6. Given set : (4, 10, 15)


a. 3, 6, 12 b. 2, 8, 10 c. 5, 12, 18 d. 7, 10, 18 (c)
4 + 6 = 10 5 + 7 = 12 10 + 5 = 15 12 + 6 = 18

144
7. Given set : (3, 4, 5)
a. (6, 8, 10) b. (9, 12, 15) c. (6, 7, 8) d. (12, 16, 20) (c)
The consecutive numbers are given.
8. Given set : (2, 5, 15)
a. (7, 10, 13) b. (3, 8, 24) c. (3, 6, 18) d. (4, 5, 9) (b)
2× 2 + 1 = 5 3×2+2=8 5 × 3 = 15 8 × 3 = 24

9. Given set : ( 12, 18, 24)


a. (6, 12, 30) b. (5, 15, 10) c. (10, 16, 18) d. (8, 12, 16) (d)
6 × 2 = 12 4×2=8
6 × 3 = 18 4 × 3 = 12
6 × 4 = 24 4 × 4 = 16

10. Given set : (9, 8, 729)


a. (3, 9, 30) b. (7, 49, 343) c. (12, 144, 221) d. (6, 36, 92)
9 × 9 = 81 =£¤
9 × 9 × 9 = 729
Similarly 7 × 7 = 49
7 × 7 × 7 = 343

Type – 3
In each of the following questions, select the related number from the given
alternatives.
1. 3 : 243 :: 5 : ?
a. 405 b. 465 c. 3125 d. 546 (c)

(3) (3) = 243

5 (5) = 3125

2. 5 : 36 : : 6 : ?
a. 48 b. 50 c. 49 d. 56 (c)

5 + 1 = (6) = 36
145
6 + 1 = (7) = 49

3. 169 : 13 : : 289 : ?

√169 = 13 a. 19 b. 17 c. 27 d. 23

√289 = 17 (b)
4. 6 : 222 : : 48 : ?
(6)#+ 6 = 222 1. 350 2. 343 c. 336 d. 210
(7)#+ 7 = 350
(a)

5. 24 : 126 :: 48 : ?
5 – 1 = 24 5# + 1 = 126

7 – 1 = 48 7# + 1 = 344
a. 433 b. 192 c. 240 d. 344 (d)

6. 7584 : 5362 :: 4673 : ?


a. 2367 b. 2451 c. 2531 d. 2485
7 5 8 4 5 3 6 2 4 6 7 3 2 4 5 1
-2 -2
----2 -2
-2 -2
-2 -2

7. 2 : 4 : 8 : 10 : 20 : 40 : : 3 : 6 : 9 : ?
1. 18 : 27 : 542. 5 : 10 : 17 3. 6 : 24 : 56 4. 15 : 30 : 45 (4)
2 × 2 =4 and 2×4=8 3 × 2 = 6 similarly 15 × 2 = 30
10 × 2 = 20 and 20 × 2 = 40 3 × 3 = 9 15 × 3 = 45

8. 47 : 65 : : 59 : ?
a. 95 b. 110 c. 106 d. 118 (a)
4 + 7 = 6 + 5 = 11
5 + 9 = 9 + 5 = 14

146
9. 20 : 30 : : ? : 72
a. 56 b. 59 c. 68 d. 61 4 × 5 = 20 ¥¦§¦¨=©¨ª
5 × 6 = 30
7 × 8 = 56
8 × 9 = 72

10. 6 : 5 : : 8 : ?
a. 2 b. 4 c. 6 d. 10
2×3=6 2+3=5
2×4=8 2+4=6

Type – 4
In each of the following questions, select the related letter from the given
alternatives.
1. TELE : LATE : : ? : CAFÉ
a. FACE b. CAEF c. CEFA d. FEAC
1 2 3 4 TALE = LATE
3 2 1 4 (a)
The first and third letters have been interchanged.
3 2 1 4 1 2 3 4
C A F E = F A C E

2. BOQD : ERTG :: ANPC : ?


a. YUQM b. XURO c. SHFU d. DSQF
2 15 17 4 5 18 20 7 Similarly
B 0 Q D E R T G A N P C D Q S F
+3 +3
+3 +3
+3 +3
+3 +3

3. AFKP : ZUPK : : BGLQ : ?

147
a. YUQM b. XURO c. YXWV d. YTOJ (d)
Series of opposite letters
A B C D E F G H I J K L M

Z Y X W V U T S R Q P O N

NOW A F K P Similarly B G L Q

Z U P K Y T O J

4. E G I K : F I L O :: F H J L : ?
a. G J M P b. G M J P c. J G M P d. J G P M
EGIK F I L O Similarly FHJL GIMP
+1 +1
+2 +2
+3 +3
+4 +4

5. EGIK : WUKSQ :: DFHJ : ?


a. XVTR b. BDFH c. ECGI d. SQON
E G I K W U S Q Similarly D F H T X V T R
+2 +2 +2 +2 +2 +2 +2 +2 +2 +2 +2 +2

6. AZBY : DWEV :: HSIR : ?


a. JQKO b. KPOL c. KPLO d. KOLP
A Z
Pairs of opposite letters
B y
Similarly
H S and K P

148
I R L O

7. NUMBER : UNBMRE :: GHOST : ?


a. HOGST b. HOGTS c. HGSOT d. HGOST
Two adjacent letters are inter changed.
N UM B E R Similarly G H O S T
U N B M R E H G S O T

TYPE – 5
Select the related letter / number from the given alternatives.
1. D E : 4 5 : : B C : ? (b)
a. 34 b. 23 c. 56 d. 43
D E position number in English alphabet similarly
4 5
B C
2 3

2. CAT : 3120 :: MAT : ? (c)


a. 1312 b. 10120 c. 13120 d. 12120
C A T Similarly M A T

3 1 20 13 1 20

3. RUST : 9687 :: MAT : ? (b)


a. 7896 b. 7869 c. 7689 d. 6789
R U S T Similarly T S U R

9 6 8 7 7 8 6 9

4. F : 216 :: L : ? (b)
a. 1723 b. 1728 c. 1700 d. 1600

149
F 6 ; 6# = 216
L 12; 12# = 1728

5. C 3 F 6 : J 9 L 12 :: K 11 N 14 : ? (b)
a. O 15 R 20 b. R 18 U 21 c. Q 17 T 20 d. L 12 O 15
C +6 T K +6 Q
3 +6 9 11 +6 17
F +6 L N +6 T
6 +6 12 14 +6 20
u ! !
6. : :: – : ? (d)
* K
! $ $
=. b. c. d.
% % &$ !
!
C = 3, 3 × 9 = 27 ⇒ %

L = 12, 12 × 2 = 24
$
E = 5, 5 × 9 = 45 ⇒!

N = 14, 14 × 4 = 56

$&
7. M.K: :: JH : ? (b)

$! %% %%
=. b. c. d.
%% % $!

« ⇒ 13 and 13 = 169 similarly J = 10 and 10 = 100


¬ ⇒ 11 and 11 = 121 H = 8 and 8 = 64

$& %%
MK = JH =
$!

150
A

A
B A
A
BA
AA
All +All= No

Some +All= Some Not

Some +Some= No Conclusion

Rule No.4 (Either –OR) & (Neither –Nor)

1. Both Conclusions Must be Wrong


2. Subjects and Objects are same
3. One should be positive another should be negative
4. All + No= No conclusion

Q1. Statements:

All Mangoes are Apples

Conclusions:

I. Some Mangoes are not apples


II. Some Apples are mangoes
III. All Apples are Mangoes
IV. Some Mangoes are Apples
V. No Apple is Mango
VI. No Mango is Apple
VII. Some Apples are not Mangoes

Q2. Statements:

Some Mangoes are Apples

Conclusions:

I. Some Apples are not Mangoes


II. Some Apples are mangoes
III. All Apples are Mangoes
IV. All Mangoes are Apples
V. No Apple is Mango
VI. No Mango is Apple
VII. Some Mangoes are not Apple

153
Q3. Statements:

No Mango is Apple

Conclusions:

I. Some Mangoes are not apples


II. Some Apples are mangoes
III. All Apples are Mangoes
IV. Some Mangoes are Apples
V. No Apple is Mango
VI. All Mangoes are Apple
VII. Some Apples are not Mangoes

Q4. Statements:

Some Mangoes are not Apples

Conclusions:

I. Some Mangoes are not apples


II. Some Apples are mangoes
III. All Apples are Mangoes
IV. Some Mangoes are Apples
V. No Apple is Mango
VI. No Mango is Apple
VII. Some Mangoes are not Apple

Q5. Statements:

All Clouds are Rains

All Rains are Water

Conclusions:

I. All Clouds are Water


II. Some Clouds are Water
III. Some Water are Clouds
IV. All Water are Clouds
V. Some Clouds are not Water
VI. Some Water are not Clouds
VII. No Water is Cloud
VIII. No Cloud is water

Q6. Statements:

All Cars are Buses

All Buses are Trains

154
All Trains are Planes

Conclusions:

I. All Cars are Trains


II. Some Trains are Cars
III. All Buses are Planes
IV. Some Planes are Buses
V. Some Cars are planes
VI. Some Planes are Cars

Q7. Statements:

All Rains are Water

All Clouds are Blue

Some Clouds are Rains

Conclusions:

I. Some Water are Blue


II. Some Blue are Water
III. All water are Blue
IV. No Water is Blue

Q8. Statements:

No Dice is Cube

No Cube is Square

Conclusions:

I. No Dice is Square
II. No Square is Dice
III. All Dices are Squares
IV. All Squares are Dices

Q9. Statements:

All Pens are Books

No Book is copy

Conclusions:

I. No Pen is Copy
II. Some Pens are not Copies
III. No Copy is Pen
IV. Some Copies are not pens
V. All Pens are Copies

155
Q10. Statements:

Some Pens are Books

No Book is copy

Conclusions:

I. No Pen is Copy
II. Some Pens are not Copies
III. No Copy is Pen
IV. Some Copies are not pens
V. All Pens are Copies

Q11. Statements:

All Rains are Drops

No Water is drop

No Rain is Blue

Conclusions:

I. No Rain is Water
II. No Water is Rain
III. Some Rains are not Water
IV. Some Water are not rains
V. No Drop is Blue
VI. No Blue is Drop
VII. Some Drops are not Blue
VIII. Some Blue are not Drops

Q12. Statements:

All Earths are Moons

No Moon is Planet

Some Moons are Sun

Conclusions:

I. Some Moons are Earths


II. Some Planets are not Moons
III. All Suns are Moons
IV. Some Earths are not Planets
V. Some Planets are not Earths
VI. Some Planets are not Suns
VII. Some Suns are not Planets
VIII. All Earths are Suns
IX. Some Suns are Earths

Q13. Statements:

156
All Bulbs are lights

All Tubes are Lights

Conclusions:

I. Some Lights are not Bulbs


II. All Bulbs are Tubes

Q14. Statements:

All Dogs are Animals

Some Cats are Dogs

Conclusions:

I. All Animals are Dogs


II. Some Cats are Animals

Q15. Statements:

No apple is Mango

No Mango is Orange

Conclusions:

I. Some Oranges are not mangoes


II. No Apple is Orange

Q16. Statements:

All Books are Copies

No Book is Pen

Conclusions:

I. Some Books are not Pens


II. Some pens are not Copies

Q17. Statements:

No Train is Bus

All Cars are Buses

Conclusions:

I. Some Trains are not Cars


II. Some Cars Buses

Q17. Statements:

All dogs are Animals

Some Animals are Cats

157
Conclusions:

I. Some Dogs are Cats


II. Some Dogs are not Cats

Q18. Statements:

No Student is Actor

No Actor is Hero

Conclusions:

I. Some Students are Hero’s


II. Some Students are not Hero’s

Conclusions:

I. No Student is Hero
II. Some Students are Hero’s

Conclusions:

I. All Students are Hero’s


II. No Student is Hero

Conclusions:

I. Some Students are Hero’s


II. Some Hero’s are not Students

Conclusions:

I. All Students are Hero’s


II. Some Hero’s are not Students

Conclusions:

I. Some Students are Hero’s


II. Some Hero’s are not Actors
III. No Student is Hero

Conclusions:

I. All Students are Hero’s


II. Some Students are not Hero’s
III. No Student is Hero

Q19. Statements:

All Pens are Books

No Book is Copy

All Tests are Copies

158
Conclusions:

I. No Pen is Test
II. No Test is Pen
III. Some Tests are not Pens
IV. Some Pens are not Tests
V. All Pens are Tests
VI. All Tests are Pens
VII. Some tests are pens
VIII. Some Pens are Tests

Q20. Statements:

All Books are Pens

No Book is Copy

All Tests are Copies

Conclusions:

I. No Pen is Test
II. No Test is Pen
III. Some Tests are not Pens
IV. Some Pens are not Tests
V. All Pens are Tests
VI. All Tests are Pens
VII. Some tests are pens
VIII. Some Pens are Tests

Q21. Statements:

All Books are Pens

No Book is Copy

All Copies are Tests

Conclusions:

I. No Pen is Test
II. No Test is Pen
III. Some Tests are not Pens
IV. Some Pens are not Tests
V. All Pens are Tests
VI. All Tests are Pens
VII. Some tests are pens
VIII. Some Pens are Tests

Q22. Statements:

Only Boys are Actors

Some Actors are Engineers

159
Conclusions:

I. Only Boys are Engineers


II. Some Boys are Engineers
III. All Boys are Engineers

Q23. Statements:

Some Boys are Actors

All Actors are Engineers

Conclusions:

I. 70% Boys are Engineers


II. 50% Boys are Engineers

Q24. Statements:

60% Boys are Actors

All Actors are Engineers

Conclusions:

I. Some Boys are Engineers

160
PUZZLE TEST
TYPE 1
This type consists in which certain items belonging to different groups or possessing
different are given along with some clues with the help of which the candidate is required to
group and analyse the given items and answer the questions accordingly.
1. Study the following information carefully and answer the given questions:
a. B and E are good in Dramatics and Computer Science.
b. A and B are good in Computer Science and Physics.
c. A, D and C are good in Physics and Mathematics.
d. C and A are good in Physics and Mathematics.
e. D and E are good in History and Dramatics.
I. Who is good in Physics, History and Dramatics?
a. A b. B c. D d. E
II. Who is good in physics, history and mathematics, but not in Computer
Science?
a. A b. B c. C d. D
III. Who is good in computer science history and dramatics?
a. A b. B c. C d. E
IV. Who is good in history, physics, computer science and mathematics?
a. A b. B c. C d. D
V. Who is good in physics, Dramatics and computer science?
a. A b. B c. D d. E
Sol: The given information can be analysed as under:

Dramatics Computer sc Physics History Mathematics

A × √ √ √ √
B √ √ √ × ×

C × × √ √ √
D √ × √ √ ×

E √ √ × √ ×

I. D is good in Physics, History and Dramatics. So, the answer is(c).


II. Both A and C are good in Physics, History and Mathematics. But A is good in
Computer Science, while C is not. So, the answer is(c).
III. E is good in Computer Science, History and Dramatics. Hence, the answer is (d).
IV. A is good in History, Physics, Computer Science and Mathematics. Hence, the
answer is(a)
V. B is good in Physics, Dramatics and Computer Science. Hence, the answer is(b).

161
2. Read the following information carefully and answer the questions that follow:
a. Five friends P, Q, R, S and T travelled to five different cities of Chennai,
Kolkata, Delhi, Bangalore and Hyderabad by different modes of transport of
Bus, Train, Aeroplane, Car and Boat from Mumbai.
b. The person who travelled to Delhi did not travel by boat.
c. R went to Bangalore by car and Q went to Kolkata by Aeroplane.
d. S travelled by boat whereas T travelled by train.
e. Mumbai is not connected by bus to Delhi and Chennai.

I. Which of the following combinations of person and mode is not correct?


a. P – Bus b. Q – Aeroplane c. R – car d. S – Boat e. T – Aeroplane

II. Which of the following combinations is true for S?


a. Delhi – Bus b. Chennai – Bus c. Chennai – Boat d. Data inadequate
e.None of these
III. Which of the following combinations of place and mode is not correct?
a. Delhi – Bus b. Kolkata – Aeroplane c. Bangalore - Car
d. Chennai – Boat e. Hyderabad – Bus
IV. The person travelling to Delhi went by which of the following modes?
a. Bus b. Train c. Aeroplane d. Car e. Boat

v. Who among the following travelled to Delhi?


a. R b. S c. T d. data inadequate e. None of these
Sol:- The given information can be analyses as follows:
a. Mode of Transport:- R travels by Car, Q by Aero plane, S by Boat and T by
Train. Now, only P remains. So, P travels by Bus.
b. Place of Travel:- R goes to Bangalore, Q to Kolkata, Now, bus transport is not
available for Delhi or Chennai. So, P who travels by Bus goes to Hyderabad. S
travels by boat and hence, by (ii), did not go to Delhi. So, S goes to Chennai.
Now, only T remains. So, T goes to Delhi.

Person P Q R S T

Place Hyderabad Kolkata Bangalore Chennai Delhi

Mode Bus Aero plane Car Boat Train

I. Clearly, the incorrect combination is T – Aero plane. So, the answer is (e).

162
II. Clearly, the correct combination for S is Chennai – Boat. So, the answer is (c).
III. Clearly, the incorrect combination is Delhi – Bus. So, the answer is (a).
IV. Clearly, T travelled to Delhi by train. So, the answer is (b).
V. Clearly, T travelled to Delhi. So, the answer is (c).
TYPE – 2
In this type of questions, some clues regarding seating or placing sequence (linear or
circular) of some persons or items is given. The candidate is required to from the proper
sequence using these clues and answer the given questions accordingly.
3. Read the following information carefully to answer the questions given below:
Nine cricket fans are watching a match in a stadium. Seated in one row, they are J, K,
L, M, N, O, P, Q and R, L is at the right of M and at third place at the right of N, K is at
one end of the row. Q is seated adjacent to both O and P. O is at the third place at
the left of K, J is right next to left of O.

I. Who is sitting at the Centre of the row?


a. I b. J c. O d. Q
II. Who is at the other end of the row?
a. J b. N c. P d. R
III. Which of the following statements is true?
a. N is two seats away from J. c. R and P are neighbors
b. b. M is at one extreme end. d. There is one person between L and O.
sol:- L is at the right of M and at third place at the right of N, So, the sequence becomes:
NOML. O is at third place at the left of K and J is right next to left of O i.e. JO K. Q is
adjacent to both O and P i.e. JOQPK. Thus, the only black space left must be occupied by R
i.e. NRML.
Since K is at one end of the row, so the entire seating sequence becomes: NRMLJOQPK.
I. J is sitting at the center of the row, so the answer is ( b)
II. N is at the other end of the row. So, the answer is (b)
III. Clearly, there is one person J between L and O.
So, only (d) is true.
Hence, the answer is (d).

4. Eight books are kept one over the other. Counting from the top, the second, fifth and
sixth books are on Plays. Two books on Plays are between two books on
Composition. One book of Plays is between two books on Poetry while the book at
the top of the book of Literature is a book of Composition. Which book is fourth from
the top?
Sol:- We analysis the given information as follows:
Let C denote ‘Composition’, P denote, ‘Plays’, Po denote ‘Poetry’ and L denote “
Literature’

163
1 2 3 4 5 6 7 8
- P - - P P - -
- - - C P P C -
Po P Po - - - - -
- - - - - - C -
So, the arrangement from top to bottom becomes:
1 2 3 4 5 6 7 8
Po P Po C P P C L

TYPE – 3
In such type of questions, clues are given regarding comparisons among a set of
persons or things with respect to one or more qualities. The candidate is required to
analyse the whole information, form a proper ascending/descending sequence and
then answer the given questions accordingly.

5. Read the following information and answer the questions given below it:
Alka is older then Mala. Gopal is older than Mala but younger than Alka. Kapil is
younger than Ram and Mala. Mala is older than Ram.
I. Whose age is between Gopal and Ram?
a. Mala b. Kapil c. Alka d. None of these
II. Whose age is between Mala and Kapil?
a. Gopal b. Ram c. Alka d. None of these
III. Whose age is exactly in the middle of all the five?
a. Mala b. Gopal c. Ram d. Alka
IV. Who is the eldest?
a. Alka b. Mala c. Kapil d. Gopal
V. Who is the youngest?
a. Mala b. Ram c. Alka d. Kapil
Sol: Let us denote the five persons by the first letter of their names, namely A, M, G, K
and R.
Then, A > M, A > G > M, R > K, M > K and M > R.
Combining all the above, we get: A > G > M > R > K
I. Mala’s age is between Gopal and Rom. So, the answer is (a).
II. Ram’s age is between Mala and Kapil. So, the answer is (b).
III. Clearly, Mala lies in the middle when all the five persons are arranged in
ascending or descending order of their ages. So, the answer is (a).
IV. Clearly, Alka is the eldest. So, the answer is (a).
V. Kapil is the youngest. So, the answer is (d).

164
6. Read the information given below and answer the questions that follow:
i. There is a group of five girls.
ii. Kamini is second in height but younger than Reena.
iii. Pooja is taller than Monika but younger in age.
iv. Reena and Monika are of the same age but Reena is taller between them.
v. Neelam is taller than Pooja and elder to Reena.
vi. Kamini is not the youngest in the group.
a). If the girls are arranged in the ascending order of heights, who will be in third
position?
a. Monika b. Reena c. Monika or Reena d. Data inadequate
e. None of these
b). If they are arranged in the descending order of their ages, who will be in fourth
position?
a. Monika or Reena b. Kamini c. Only (vi) d. Data inadequate
e. None of these
c). to answer the question “who is the eldest person in the group”. Which of the
given statements is superfluous?
a. only (i) b. Only (ii) c. Only (vi) d. Either (i) or (iv) e. None of these
Sol:- We first find the sequence of heights.
By (iii), we have: M < P.
By (v), we have: P < N.
Now, we determine the age sequence:
By (ii), we have: K < R.
. By (iii), we have: P < M.
By (iv), we have: R = M.
By (V), we have: R < N.
So, the sequence of ages may be: N > R = M > K > P or N > R = M > P > K.
But Kamini is not the youngest.
So, the sequence of ages is : N > R = M > K > P.
a) Clearly, in the descending order of heights, Neelam is in third position. Hence,
the answer is (e).
b) Clearly, in the descending order of ages, Pooja will be in fourth position (because
Monika and Reena both lie at third position). Hence, the answer is (e).

165
c) Clearly, to find the eldest person in the group, statement (vi) is not necessary.
Hence, the answer is (c).

TYPE – 4
In this type of questions, some clue are given regarding the order of certain events.
The candidate is required to analyse the given information, frame the right sequence
and then answer the questions accordingly.
7. Read the following information carefully to answer the given questions:
Six films – P, Q, R, S, T and U are to be released on consecutive Fridays, the schedule
of the released a week before T.
I. P must be released a week before?
II. R must not be released immediately after the first release.
III. Q must be released on the Friday following the Friday on which U is released.
IV. S must be released on fifth Friday and should not be immediately preceded
by Q.
V. T must not be released in the last.
1. Which of the following films preceded T?
a. P b. Q c. S d. U
2. Which of the following films released immediately after Q?
a. P b. R c. T d. U
3. Film R cannot be released on which of the following Fridays in addition to
second Friday?
a. Q first b. Third c. Fourth d. Fifth
4. In between which of the two films S is to be released?
a. Q and T b. R and T c. P and T d. T and U
5. Which of the following films released first?
a. P b. Q c. R d. U
Sol:- Clearly, S must be released on fifth Friday. P must be released a week
before T i.e., order PT must be followed. But T cannot be released in the last. Also, Q
must be released immediately after U i.e. order UQ must be followed. But Q cannot
precede S. So, U and Q can be released on first and second Fridays respectively and P
and T on third and fourth Fridays respectively. R, which cannot be released on
second Friday, shall be released last.
Thus, the order followed will be: U, Q, P, T, S, R.
1. Clearly, the release of P precedes that of T. So, the answer is (a).
2. P is released immediately after Q. So, the answer is (a).
3. R cannot be released on fifth Friday as well because S has to be staged on that
day. So, the answer is (d).
4. Clearly, S is released between T and R.
So, the answer is (b).
5. Clearly, U is released first. So, the answer is (d).

166
8. Read the following information carefully and answer the questions given below it:
I. Eight doctors P, Q, R, S, T, U, V and W visit a charitable dispensary every day
except on a holiday i.e. Monday.
II. Each doctor visits for one hour from Tuesday to Sunday except Saturday. The
timings are 0 a.m. to 1 p.m. and 2 p.m. to 6 p.m: 1 p.m. to 2 p.m. is lunch
break..
III. On Saturday, it is open only in the morning i.e. 9 a.m. to 1 p.m. and each
doctor visits for only half an hour.
IV. No other doctor visits the dispensary before doctor Q and after doctor U.
V. Doctor W comes immediately after lunch break and is followed by R.
VI. S comes in the same order as P in the afternoon session.
1. Doctor P visits in between which of the following pairs of doctors?
a. S and V b. U and W c. R and W d. R and U e. none of these
2. At what time the visit of doctor R is over on Sunday?
a. 1 p.m. b. 3 p.m. c. 4 p.m. d. 5 p.m. e. none of these
3. At what time the visit of doctor T would be over on Saturday?
a. 10 a.m. b. 11 a.m. c. Either 10 a.m. or 11 a.m. d. data
inadequate e. none of these
4. If the lunch break and subsequent visiting hours are reduced by 15 minutes, at
what time doctor U is expected to attend the dispensary?
a. 3.15 p.m. b. 4 p.m. c. 4.15 p.m. d. 4.45 p.m. e. none of these
Sol:- We first form the sequence of visit using (iv), (v) and (iv).
From (iv), we know that Q visits first and U visits last.
From (v), we know that W visits first after break and is followed by R.
From (vi), we know that P visits after break.
Thus, the sequence of visit after break becomes W, R, P, U. Also S has the
same position in morning sessin as P in afternoon session.
So, sequence of visit before break is Q, T/V, S, V/T.
1. Clearly, P visits between R and U. Hence, the answer is (d).
2. The time of visit of W is 2 p.m. to 3 p.m., that of doctor R is 3 p.m. to 4 p.m. So,
the visit of doctor R is over at 4 p.m. Hence, the answer is (c).
3. Clearly, T visits either second or fourth. So, the time of visit on Saturday will be
either 9.30 a.m. or 10.30 a.m. Thus, T’s visit will be over at either 10 a.m. or 11
a.m. Hence, the answer is (c).
4. Clearly, as mentioned, lunch break will be over and doctor W will visit at 1.45
p.m., doctor R will visit at 2.30 p.m., doctor P will visit at 3.15 p.m. and U will
visit at 4 p.m. Hence the answer is (b).

TYPE – 5
In such type of questions, a few essential criteria for selection of a group of
items are given. The candidate has to keep these conditions in mind and make
the required selection as per the directions given in each question.

167
9. Study the following information carefully and answer the questions given below it:
From amongst six boys A, B, C, D, E and F and five girls P, Q, R, S and T, a team of six
is to be selected under the following conditions:
I. A and D have to be together.
II. C cannot go with S.
III. S and T have to be together.
IV. B cannot be teamed with E.
V. D cannot go with P.
VI. B and R have to be together.
VII. C and Q have to be together.
1. If there be five boys in the team, the lone girl member is.
a. P b. Q c. R d. S
2. If including P, the team has three girls, the members are.
a. B, C, F, Q, R b. A, D, E, S, T c. A, D, B, S, T d. B, F, R, S, T
3. If the team including C consists of four boys, the members of the team other
then C are
a. A, D, E, P, Q b. A, B, D, Q, R c. D, E, F, A, Q d. B, E, F, R, Q
4. If four members including E have to be boys, the members other than E are
a. A, B, C, Q, R b. A, D, F, S, T c. B, C, F, Q, R d. A, C, D, F, Q
5. If four members have to be girls, the members of the team are.
a. B, C, P, Q, R, S b. B, F, P, R, S, T c. B, C, Q, R, S, T
d. B, C, P, Q, R, T.
Sol:-
1. selected. If C is selected, a girl Q will be selected.
From B and E, one has to be selected. So, we select E because B will be
accompanied by a girl. The fifth boy will be F. So, the only girl will be Q in the
team A, C, D, E, F, Q. Hence, the answer is (b).
2. If P is included, then D and hence A cannot be included. If Q is selected, C has to
be selected and so S cannot be selected. T goes with S. So, T is also not selected.
The third girl can be R. With R, B will be selected but with B, E cannot be
selected. So, the sixth member can be F only i.e. the team becomes P, Q, C, R, B,
F. Hence, the answer is (a).
3. If team includes C, Q will be included. If another girl included is R, B will be there
and hence E cannot be there. A and D have to be together. So, they are also In a
team of six, if five boys are to be selected, then both A and D together are
included and only F can be excluded. Thus, the team is C, Q, R, B, A, D. Hence, the
answer is (b).
4. If E is included, B cannot be included. A and D have to be together. So, they are
both included. Without B, R will not be there. With D, P cannot be there. So, two
girls together can be only S and T. If S is there, C cannot be there. So, the fourth
boy can be F alone. Thus the team becomes E, A, D, S, T, F.
Hence, the answer is (b).
5. In four girls, S and T are taken together. With S, C cannot be there. So Q will not
be there. If P is included, D and hence A cannot be there. If R is included, B will be
there and hence E cannot be there. So, only F can be there. Thus, the team is S, T,
P, R, B, F. Hence, the answer is (b).

168
10. TYPE – 6
In this type of questions, some clues are given regarding relationship among
different members of a family, together with their professions, dresses,
preferences etc. The candidate is required to analyse the whole information
and then answer the given questions accordingly.

169
RACES
The place at which a race conduct is called as race course.

. 100
A, B goal
starting A
point 90 10

A beats B by 10 m in a race of 100 mts that is A covers a distance of 100 mts and B
covers 90 mts only.

Race = 20 sec
A B B = 25 sec
100 mts 90 mts
A beats B by 5 sec.

A beat B by 25 m or 5 sec in a race of 100 mts.

100
5 sec A A B
75 100 75
B 25 m
SB = = 5 MPs

A B
200 m 150 m
Race course length = 200 m
A beat B by 50 m

A B
100 m 90 m
100

A
10 B
B is ahead of 10 m to A.

B has a lead of 10 m to A.
(Start)
A can give a start of 10 m to B.

That is a covers 100 m and B covers 90 m only.

170
In a race of 500, A can give a start of 100m to B.
500
A B
A B 500 m 400
100
Draw = dead – heat race (same time).

1. In a km race, a beats B by 40 m per 8 sec. Find the time taken by A to complete


the race. 1000 m
!%
Speed of B = = 5 mps 5
%%%
Time of B = = 200 sec 960 B 40 A
Time of A = 200 – 8 = 192 sec 192
%%%
Speed of A = &

2. In a 200 m race, A beats B by 20 m or 9 sec. Find speed of A in kmph.


%
Speed of B = &
200
&
Time of B ⇒ 200 × %%
= 90 sec 9
Time of A ⇒ 81 sec 20
%% %
∴ speed of A = × ⇒ kmph
&

3. In a km race, A beats B by 200 m and B beats C by 280 m in the same race B eats
C by.
How many meters = 80
A B C
1000 800 720
80 is more in 800 m
In 1000 %
800 m = 80
%%% × %
1000 m = = 100 m
%
∴ B beat C by 100 m.

4. A beat B by 50 m in a race of 500 m and B beats C by 50 m in a race of 300 m. in a


race of 500 m A beat C by bow many meters.
A B C B C
500 450 375 300 250
300 = 450
! % × %
450 = = 375
#%%

171
A B C
500 450 375
A, B eat C in 125 m.

5. In a km race, A can give B a start of 50 m and C a start of 69 m. How many meters


B can give C in the same rae.

1000
900
950 = 19
%%% × &
1000 = = 20
%

B can give C = 20

6. The speeds of A and B are 12 kmph, 18 kmph respectively. In a race of 500 m B


give A a start of 120 m. Find B beat A by how many.
500 A B
B A 380 500
120
%
B beat A = $
= 20

tA = 12 kmph
tB = 18 kmph
%
∴ 12 × = #

tB = 18 × = 5sec
#
⇒ tn = 380 × = 114 sec
%
%%
tB = = 100 sec

B beat A by 14 sec

7. A speed is 1 times of B speed in a race A give B a start of 300 m. Find the length
of the race course so that both A & B reach the goal at the same time.

A B

172
d d – 300
# 2„
SA = SB = =?

T constant =
3#%%
⇒ =
#

⇒ 2¤ = 3¤ − 900
¤ = 900 ∴ d = 900
¨-£®¯ℎ = 900 §
-
8. In a race of 400 m, A gives B a start of 20 m and beats him by 20 m. In the same
race A beat C by 20 m then find ratio of the speed of B & C.
A B C
400 360 380 9m
dB : dC ⇒ 360 : 380 A→
16 m B←
⇒ 18 :19
³∝S
Sb : SC = 18 :19

I $ !
z
=µ ⇒µ& =#

173
RANKING

¶·¸¹º·»¼½¾ ¾¿À¼, ¸¹Á¶¼


¿¸¼¹Ã½¾ ¼·Ä, Å·¼¼·Æ
RANKING

A
L end 10 11 R end

Total = (10 + 11) - 1

Total = L + R - 1 Horizontal

L = (total + 1) –R
R = (total + 1) – L
Total = (Top + Bottom) - 1 Vertical

Top = (Total + 1) – bottom


Bottom = (Total + 1) – top.

Q1. In a row Mohan is 18th from the Left end if he is 15th from Right end, Then
how many persons are there in the row?
Total = L + R – 1
= 18 + 15 – 1 = 32
Q2. In a row of 50 persons Deepak is 19th from Right end. What is the rank of
Deepak from Left end?
L = 51 – 19 - 32
Q3. In a row Ravi is 15th from Left end Amit is 16th from Right end. If Sumit is 4th
to Right of Ravi and is 5th to the Left of Amit, then
R 4 S 5 A
Lend 15 16 Read
1. What is rank of Ravi from Right end?
16 + 5 + 4 = 25 Rank = distance

174
2. What is the rank of Amit from Left end?
15 + 4 + 5 = 24
3. What is the rank of Sumit from Right end?
16 + 5 = 21
4. What is the rank of Sumit from Left end?
15 + 4 = 19
5. How many persons are there between Ravi and Amit?
9–1=8 Persons = D - 1
6. How many persons are there between Amit and Sumit?
5–1=4
7. How many Total persons are there in a row?
15 + 4 + 5 + 16 = 40 – 1 = 39
Q4. In a row Rohan is 12th from left end Kamal is 16th from Right end. Kamal is
right to Rohan if there are 7 persons between them, then
a. What is the rank of Rohan from Right end?
16 + 8 24
b. How many persons are there in the row?
(12 + 8 + 16) – 1 = 36 – 1 = 35
R Kamal persons = 7
Lend 12 8 16 distance = 8
Q5. In a row Ishu is 13th from left end Abhay is 16th from Right end Abhay is left
to Ishu if there are 5 persons between them, then how many persons are
there in the row? A I
13 6 Rend
7 16
7 16
23 – 1 = 22 persons in a row
Q6. In a row Gopal is 12th from left end prince is 10th from Right end. It there
are 3 person’s between them then, we don’t know
a. How many are there in the row?
26 – 1 = 25, (8 + 10 ) = 17
b. What could be the minimum no.of person’s in the row?
17
c. What could be the maximum no.of person’s in the row 25?
G P (maximum - R
12 4 10 minimum - L

175
P G
4

12 10
Q7. In a row Neeraj is 10th from left end mausam is 6th from Right end if there
are 7 persons b/w then then how many persons are there in the row?
N M 24 – 1 = 23
10 8 6
Q8. In a row of girls Neha is 15th from left end, Preeti is 16th from Right end
Preeti is right to Neha. When neha is shifted buy 5 places towards her left
then there are 8 girls b/w then then how many girls are?
N 5 N
10 15 16 Rend
8+1=9
10 + 9 + 16 – 1 = 34
Q9. In a row of 50 girls when Priyanka was shifted to her left by 7 places then
her number from left end becomes 10. What is the rank of Sonam from
the right end if the Sonam was 6th places to the right of Priyanka’s original
position.
P1 P S 10 + 7 + 6 = 23
10 7 S(L) = 23
S(R) = (50 + 1) - L
Rank S(R) = 28
= 51 - 28
= 28
Q10. In a row Charu is 10th from left end Shivani is 12th from the right end
when they inter change their positions then Charu becomes 19th place
from the left end then?
a. How many persons are there?
19 + 12 – 1 = 31 – 1 = 30
b. What is new position of Shivani from right end?
Rank = 12 + 9 = 21
S1 charu S C1

176
10 9 12 Rend

19th
19 + 12 = 31 – 1 = 30
Q11. In a row girls Shivani is 12th from left end Rashmi is 10th from right when
they interchange their positions then Shikha becomes 7th from left end
then how many girls are there in the row?
S1 R S R1
7 12
10
7 = 10 = 17 – 1 = 16
Q12. In a column of boys Deeponsu is 15th from the top. If there were thrice as
many behind him as there were in front then how many boy are there
b/w Deeponsu and Vishal, who is 7th from end of the column?
Top
14
D 15
36 43
7 v 42 persons 36 – 1 = 35

Q13. In a class of 45 where girls are twice to that of boys. Shwetha a girls is 10th
from the top. If there are 5 girls ahead of her. Then how many boys are
after her?
5Ç 30
4~ 15
9 45

11 B

After her
24 G
Q14. In a class of 50 Bharti is 6th rank ahead of Renu who is 19th then what is
the rank of Bharti from the bottom?

177
Renu (top) = 19th
Bharti (top) = 13th
Bharati (bottom) = 50 + 1 – 13
= 51 – 13 = 38
Q15. In a column Sumit is 10th from the top Amit is 15th from the bottom,
Deepak is 18th from the top. If Deepak is exactly b/w the two, then how
many persons are there in the column?
top
10 8 S 18 D
A 15 8 10 + 8 + 8 + 15 – 1
41 – 1 = 40

178
AGES
1. Raju age after 15 years will be 5 times his age 5 years back, what is the present age of Raju

X+15 = 5(x-5)

4x = 40

x = 10

2. Sachin is younger than Rahul by 7 years. If the ratio of their ages is 7:9, find the age of Sachin

If Rahul age is x, then Sachin age is x-7,


so (x-7)/x = 7/9
9x-63 = 7x
2x = 63
x = 31.5

3. The ratio between the present ages of P and Q is 6:7. If Q is 4 years old than P, what will be the
ratio of the ages of P and Q after 4 years

Let P age and Q age is 6x years and 7x years.


Then 7x - 6x = 4

x=4
so required ratio will be (6x+4): (7x+4) = 28:32

= 7:8

4. Ages of two persons differ by 16 years. If 6 year ago, the elder one be 3 times as old the younger
one, find their present age

Let the age of younger person is x,


Then elder person age is (x+16)
3(x-6) = (x+16-6) [6 years before]
3x-18 = x+10
x = 14.
So other person age is x + 16 = 30

5. The sum of the ages of a father and son is 45 years. Five years ago, the product of their ages was
four times the father’s age at that time. The present age of father and son

179
Let sons age = x years.

Then fathers age = (45 - x) years.


(X—5)(45—x—5) = 4(45- x - 5)

Hence (x—5) = 4 so x = 9
their ages are 36 years and 9 years.

6. Ten years ago, P was half of Q in age. If the ratio of their present ages is 3:4, what will be the
total of their present ages

Let the present age of P and Q be 3x and 4x respectively.


Ten years ago, P was half of Q in age
2(3x – 10) = (4x – 10)
6x – 20 = 4x – 10

2x = 10
x=5

7. The total age of A and B is 12 years more than the total age of B and C. C is how many year
younger than A

Given that A+B = 12 + B + C


A – C = 12 + B – B = 12
C is younger than A by 12 years

8. The ages of two persons differ by 16 years. 6 years ago, the elder one was 3 times as old as the
younger one. What are their present ages of the elder person

Let's take the present age of the elder person = x


and the present age of the younger person = x – 16
(x – 6) = 3 (x-16-6)
x – 6 = 3x – 66
2x = 60
x = 60/2 = 30

9. The ages of two persons differ by 20 years. If 5 years ago, the elder one be 5 times as old as the
younger one, their present ages (in years) are respectively

Let their ages be x and (x + 20) years.


5 (x - 5) = (x + 20 - 5)

4x = 40 or x = 10.
Their present ages are 30 years and 10 years

180
10. Ratio between Rahul and Deepak is 4:3, After 6 Years Rahul age will be 26 years. What is
Deepak present age?

Present age is 4x and 3x,


4x + 6 = 26 => x = 5
so Deepak age is = 3(5) = 15

11. Sachin was twice as old as Ajay 10 years back. How old is Ajay today if Sachin will be 40 years
old 10 years hence

Sachin's age today = 30 years.


Sachin's age 10 years back = 20 years.
Ajay's age 10 years back = 10 years.
Ajay's age today =20 years

12. In 10 years, A will be twice as old5as B was 10 years ago. If A is now 9 years older than B, the
present age of B is

Let B's age = x years.


Then, As age = (x+ 9) years.
(x+9+10)=2(x—10)
hence x=39.
Present age of B = 39 years

13. Ten years ago A was half of B in age. If the ratio of their present ages is 3 : 4, what will be the
total of their present ages

Let A's age 10 years ago = x years.


Then, B's age 10 years ago = 2x years.
(x + 10) / (2x+ lO) = 3/4
x = 5.
So, the total of their present ages =(x + 10 + 2x + 10)
= (3x + 20) = 35 years.

14. Sushil was thrice as old as Snehal 6 years back. Sushil will be times as old as Snehal 6 years
hence. How old is Snehal today

181
Let Snehals age 6 years back = x.
Then, Sushils age 6 years back = 3x.
(5/3) * (X + 6 + 6) = (3X + 6 + 6)
So 5(x+ 12) = 3(3x+ 12),
so x=6.
Snehal Age = (x+ 6) years = 12 years

15. Six years ago, the ratio of the ages of Kunal and Sagar was 6:5, Four years hence, the ratio of
their ages will be 11:10. What is Sagar age at present

Let six years ago the age of Kunal and Sagar are 6x and 5x resp.
then,

((6x+6)+4)/((5x+6)+4)=11/10
10(6x+10)=11(5x+10)
5x=10
x=2
So Sagar age is (5x+6) = 16

17. Total of the ages of A, B ahd C at present is 90 years. Ten years ago, the ratio of their ages was
1: 2: 3. What is the age of B at present

Let their ages 10 years ago is x, 2x and 3x years.


10 + 2x + 10 + 3x + 10 = 90

Hence x= 10
B’s present age = (2x + 10) =30 years

18. A man is 24 years older than his son. In two years, his age will be twice the age of his son. The
present age of his son is

Let the son's present age be x years.

Then, man's present age = (x + 24) years


(x + 24) + 2 = 2(x + 2)
x + 26 = 2x + 4
so, x = 22

182
19. Ratio of ages of three persons is 4:7:9, Eight years ago, the sum of their ages was 56. Find their
present ages.

Let the present ages are 4x, 7x, 9x.


(4x-8) + (7x-8) + (9x-8) = 56
20x = 80
x=4
So their present ages are: 16,28,36

20. A man is 24 years older than his son. In two years, his age will be twice the age of his son. The
present age of this son is

Let's Son age is x, then Father age is x+24.


2(x+2) = (x+24+2)
2x+4 = x+26
x = 22 years

21. Tanya's grandfather was 8 times older to her 16 years ago. He would be 3 times of her age 8
years from now. What was ratio of ages of Tanya and her grandfather 8 years ago.

Let, Tanya age 16 years ago = x,


Grandfather's age 16 years ago = 8x.
8 years from now, 3(x+16+8) = (8x+16+8)
x = 48/5
8 years ago ratio was:
(x+8)/(8x+8)=(48/5+8)/(8×48/5+8)

88/424=11/53

22. A person's present age is two-fifth of the age of his mother. After 8 years, he will be one-half of
the age of his mother. How old is the mother at present

Let the mother's present age be x years.


Then, the person's present age = (2/5 x) years.

(2/5 x + 8/2) = 1 (x + 8)
2(2x + 40) = 5(x + 8)

x = 40

183
SIMPLE INTEREST & COMPOUND INTEREST
PRINCIPAL (P):- The money borrowed (or) lent out for a certain period is called the
principal (or) the sum.

Interest:- lender is money paid to the lender by the borrow for using the lenders
money for using the lenders money for a specified period of time.
Lender (borrower)

SIMPLE INTEREST (SI):- If the interest on a sum borrowed for a certain period is
reckoned uniformly. Then it is called simple interest.
Every year interest is calculated on principal only.
Simple interest is same for every year.

COMPOUND INTEREST:- Sometimes it so happens that the borrow and lender agrees
to fix up a certain period of time, say yearly, half-yearly, quarterly to settle the
previous account. In such cases, the amount after first period of time becomes the
principal for the second period of the time. The amount after second period of time.
It will become principal for the third period of the time. The difference b/w amount
and the money borrowed.

RATE OF INTEREST (R):- Rate of which interest is calculated on original sum.

AMOUNT (A):- Sum of principal and interest. What is the difference b/w simple
interest and compound interest.
SI CI
P I T.I A P I T.I A
1000 100 100 1100 1000 100 100 1100
1000 100 200 1200 1100 110 210 1210
1000 100 300 1300 1210 121 331 1331

1. In SI interest is same for every year.


Example:- If rate of interest is 10%.
After 3 year = 3 × 10 = 30%

2. In CI interest is increased by R% as every period of year.


If rate of interest is 20%.
CI interest for 2 year CI c.a is 44%.

3. In SI amounts are in geometric progression. A2 – A1 = A1 –P

184
4. In CI amounts are in geometric progression. A2/A1 = A1 – P

5. In CI amounts also increased by R% per every year. (CI, C.A)

PROBLEMS ON SIMPLE INTEREST:- In this chaper the sum (or) principal is taken as
100.

6. Find the simple interest on Rs.68,000 at 12% per annum for 4 years.
1 year 12y 100 = 68000
4 year 48% 48 = ? 32,640

7. Find the simple interest on Rs. 24,000 at 16 2/3 per annum for a months.
1 year 50/3 % 12 months
12 50/3
9 (9/12 × 50/3) = 12.5
100 = 24000
12.5 = ? 3000/-

8. Find the simple interest on Rs. 3000 at 6 1/4 % per annum for the period from 4th
Feb 2005 to 18th April 2005?
(24 + 31 + 18) = 73 days
365 25/4
73 73 × 25/4 × 1/365 = 1.25%
100 = 3000
1.25 = ? 37.5/-

9. Adam borrowed some money at the rate of 6% p.a. for the first two years, at the
rate of 9% p.a. for the period beyond five years. If he pays a total interest of Rs.
11,400 at the end of nine years, how much money did he borrow?
2 × 6 = 12
3 × 9 = 27 95 = 11,400
4 × 14 = 56 100 =? 12,000/-
95
10. What sum will become an amount of 5800/- at the rate of 15% p.a. for a period of 3
years simple interest.
145 = 5800
100 = 4000/-

11. A certain sum becomes an amount of 17,700/- of 3 years and amount to 19,500/-
after 5 years find the respective value of sum & the rate of interest p.a. under simple
interest.
3 2y
p 17, 700
185
2 years = 1800
1 years = 900
3 years = 2700
P = 17700 – 2700 = 15,000 p = 15,000
Rate of interest (R ) = 900/15,000 × 100 = 6% p.a

12. In how many years a sum becomes 6 times it self rate of interest is 20% pq under
simple interest .
100 × 500/20 × 600 = 25 years

13. At what rate percent of simple interest will a sum of money double itself in 12 years.
100
100 7 200
100 – 7 years
400 - ? 28 years
14. A man lent out an amount 10,000/- into 2 parts the first part at 8% p.a. and second
part at 10% pa bath under S.I. at the end of the year received 890/- as a total
interest. Find the amount he lent out at 8% p.a.

Lender:- Logic:-
100x + 100y = 10,000 10,000/- 890/-
8x + 10y = 890 890/10,000 × 100 = 8.9%
10x + 10y =1000 8% 10%
2x = 110 8.9% 20p = 10,000
X = 55 1.1% 0.9 11p = 5500
100x = 100 × 55 = 5500 11 : 9 9p = 4500
15. Peter invested an amount of Rs. 12,000/- at the rate of 10 p.c p.a simple interest and
another amount at the rate of 20p.c p.a simple interest. The total interest earned at
the end of one year on the total amount invested?
Logic 10% 20%
14% 3p = 12,000/-
6:4 2p = 8,000/-
3:2 total = 20,000/-

16. A sum of 3700 Rs becomes an amount of 6470 Rs after 4 years at a certain rate of
interest under simple interest. If the rate of interest is 6% p.a. more, then the same
sum will become how much amount in the same time period?
6% of 3700 = 222

186
4 years = 4 × 222 = 888/-
Amount = 6470 + 888 = 7358/-
COMPOUND INTEREST:-
17. Find the interest & amount on 8000/- at the rate of 10% p.a. for period of 3 years
compounding annually.
800 10% 8800 10% 9680 10% 10648
800

CI = 2648/- A = 10648
Logic:- 10 × 10 × 10 = 1000 100 = 80000
11 × 11 × 11= 1331 1331 = ? 10648

18. Find the compound interest Rs 10,000/- in one and half year at 20% per annum. The
interest being compounded half yearly.
10 × 10 × 10 = 1000 = 10000 100 = 10000
11 × 11 × 11= 1331 =? 331 = ? 3310

19. What sum will become an amount of 7200/- after 2 years at the rate of 20% p.a.
compounding annually?
10 × 10 = 100 124 = 7200
12 × 12= 124 100 = ? 5000/-

20. The compound interest on a certain sum at certain rate of interest for second year is
1440/- and for 3rd year the compound interest is 1656. Find the rate of interest per
annum?
Second year = 1440 216/1440 × 100 = 15%
Third year = 1656

21. A certain sum amounts to Rs. 7350 in 2 years and to Rs. 8575 n 3 years. And rate of
interest per annum?
2 year = 7350 8575
3 year = 8575 7350
1225/7350 × 16 2/3 % 1225

22. A sum of money amount to after 2 years and to Rs. 3600 after 4 years on compound
interest find the rate of interest?
P{1 + R/100}2 2500, P(l + R/100) 4 = 3600
{1 + R/100}2 = 3600/2500 = 36/25
1 + R/100 = 6/5 R/100 = 6/5 – 1 = 1/5 R = 20% p.a.
23. A sum of money amount to 16,000 after 2 years and to Rs. 25,000 after 4 years on
compound interest compounding annually. Find the rate of interest and sum?
2 years A2 = P(1 + R/100)2 = 16,000/-
4 years A4 = P(l + R/100) 4 = 25,000/-
(1 + R/100)2 = 25000/16000 1 + R/100 = 5/4

187
R/100 = 5/4 -1 = 1/4 R = 25%
P(1 + R/100}2 = 16,000
P × 26/16 = 1600 P = (1600 × 16)/25 = 10,240

24. Find the compound interest Rs. 16,000/- at 24/4 % p.a for 9 months compounded
quarterly?
10 × 10 × 10 = 1000 1000 = 16,000
11.8 × 11.8 × 11.8 = 1643 643 = ? 10,288/

25. A sum of 4800/- interested at compound interest for 3 years, the rate of interest is
10% p.a, 20% p.a., 25% p.a. for the first, second, third years respectively. Find the
total interest received after 3 years?
10 × 10 × 10 = 1000 1000 = 4800
11 × 12 × 12.5 = 1650 716 = ? 650/-
1000 = 4800
650 = ? 3120/-

26. A certain sum becomes double after 9 years at compound interest compound in
annually in how many years the same sum will become 16 times it self?
P/100 A/200 A/400 800 1600 = 21 = 9 years
9y 9 9 9 = 24 = 36 years.

27. A certain sum becomes an amount of 12,150/- at the end of 4 years and amount to
14,580 at the end of 8 years at compound interest. Find the sum

28. A certain sum becomes double at a certain rate of interest for 2 years at compound
interest compound annually. Find the rate of interest.
P /100 2Y A/200 P(1 + R/100)2/P = 200/100
1 + R/100 =√2 = 1.414 R/100 = 0.414 R = 41.4

COMBINATION OF COMPOUND INTEREST AND SIMPLE INTEREST:-


29. The difference b/w C.I and S.I on certain sum for 2 years is 405/- if the rate of
interest is 25% per annum then find the sum.

S.I C.I 6.25 = 405


25 25 100 = ? 6480
25 25 + 6.25
50 56.25

188
Difference = P R2/1002 for 2 years
Difference = P R2/1002 [r/100 + 3] for 3 years.

30. The C.I & S.I on a certain sum at certain rate of interest for 2 years are 11,730/- and
10,200/- respective find the rate of interest and sum.
S.I C.I 11730
5100 5100 1530/5100 × 100 = 30% 5100
5100 6630 6630
Sum: - 30% = 5100 5100
100 % = ? 17,000/- 1530
INSTALLMENTS:-
31. What annual instalment will discharge a debt of Rs. 1092 due in 3 years at 12%
simple interest?
1 year 100 100 336 = 1092
2 year 100 112 100 = ? 325/-
3 year 100 124

32. What is the annual instalment will discharge a dest of Rs. 18,000 due in 2 year at the
rate of 25% p.a.
1 year 100 100 225 = 18,000
2 year 100 125 100 = ? 8,000/-

33. The sum of 2912/- is due to be repaid at the end of 3 years. If it has to be repaid in 3
equal instalments at 20% p.a. at C.I find the value of instalment.
1 year 100 100 364 = 2912
2 year 100 120 100 = ? 800/-
3 year 100 144

34. A man borrows Rs. 2550 to be paid back with compound interest at the rate of 4%
per annuam by the end of 2 years in two equal yearly instalments how much will
each instalment be?
10 × 10 = 100 (2550 × 108.16)/100 = 2758.08
10.4 × 10.4 = 108.16
100 = 2550
108.16 = ?
1 year 100 100 204 = 2758.08
2 year 100 104 100 = ? 1352/-
35. A sum of money is borrowed and paid back in two annual instalments of Rs. 882
each allowing 5% compound interest. The sum borrowed was?
1 year 882 882
2 year 882 882 + 44.1 = 926.1

189
Amount = 1808.1
110.25 = (1808.1 × 100)/110.25 = 1640/-

36. With a given rate of simple interest, the rate of principal and amount for a certain
period of time is 4 : 5 after 3 years, with the same rate of interest the ratio of the
principal and amount becomes 5 : 7. The rate of interest per annum is?
P A P : A
5
(4 : 5) 20 : 25 3/20 × 100 = 15%
4
(5 : 4) 20 : 28 P.A = 15%/3 = 5%

37. A man distributed Rs. 25,300 among three son’s A, B, C in such a way that the
amount of their parts at 10% simple interest in 2 years and 4 years will be equal A1
share is.
120A = 130B = 140C
A : B : C = (13 × 14) : (12 × 14) (12 × 13)
= 182 : 168 : 156 if interest one equal note: 20A : 30B :
40C
= 91 : 84 : 78
253P = 25,300
91P = 9100/-
38. A loan was repaid in two annual instalments of 1210 each. If the rate of interest of
1210 each. If the rate of interest be 10% per annum compounded annually. The sum
borrowed was?
1210 1210
1210 1210 + 121 1331
121 = 2541
100 ? 2100/-

39. A sum of Rs.5300 was taken as a loan. This is to be repaid in two equal instalments. If
the rate of interest be 12% compounded annually the value of each instalment is?
10 × 10 = 100 = 5300
11.2 × 11.2 = 125.44 = ? 6648.32
100 100 212 = 6648.32
100 120 100 = 664832/212 = 3136/-

190
SIMPLE EQUATIONS

TYPES OF PROBLEMS

• Problems based on ages


• Problems based on numbers
• General based problems
• Fraction based problems
• Remaining based problems

SOLVING OF EQUATION

• Single variable equation

X/3 + x/2 = 12
(2x+3x)/6 = 12
5x = 72
X = 72/5

• Two variable equation

2x + y = 10
X–y= 5
3x = 15
X=5
Y=0

Note
Twice of A is 20 less than three times of B
2A = 3B – 20
3B – 2A = 20

Problems based on ages


1. 10 years ago age of father was 4 times of age of son. 10 years hence age of father
was twice the age of son. Find the present age of father and son?
Method 1
Let present age of father F
Let present age of son S

191
10 years ago F- 10 = 4 (S- 10)
F- 10 = 4S – 40
F+30 = 4S (1)
10 years hence F+10 = 2(S+10)
F+10 = 2S + 20
F-10 = 2S (2)
Solving (1) and (2)
S = 20
F = 50
Method 2
Let the age of the son 10 years ago = x
Let the age of the father 10 years ago = 4x
10 years hence
2(x+20) = 4x+20
2x = 20
X = 10
Therefore present age of the son = 20
Present age of the father = 50

2. 10 years ago age of father was 35 years more than twice of age of son. After how
many years from now he will be twice age of son?

Let the present age of father = x


Let the present age of son = y

X-10 = 2(y-10) +25


X-10 = 2y+15
x= 2y+25 (1)
After n years he will be twice the age of the son
(x+n) = 2(y+n)
X = 2y+n
From (1)
2y+25 = 2y+n

N = 25 years

192
Problems on numbers

Consecutive integers = x, x+1, x+2, x+3…….


Consecutive even number = x, x+2, x+4, x+6……
Consecutive odd number = x, x+2, x+4, x+6……

Take equations when three consecutive numbers are given → x-1,x,x+1


Take equations when three consecutive even no. are given→ x-2, x, x+2
Take equations when three consecutive odd no. are given → x-2, x, x+2
.

3. Sum of 4 consecutive integers is 46. What is the smallest integer?

X+x+1+x+2+x+3 = 46
4x =40
X =10
So numbers are 10 11, 12, 13
So smallest integer is 10

4. Sum of squares of 3 consecutive even numbers is 200 then find the numbers?

(x-2)2 + x2 + (x+2)2 = 200


3x2 = 192
X2 = 64
X=8
So numbers are 6, 8, and 10

5. There are five consecutive integers thrice of middle integer exceeds the sum of
greatest and least integer by 12. Find the middle integer?

X,x+1,x+2,x+3,x+4
3(x+2) – (x+x+4) = 12
3x+6 –x-x-4 = 12
X+2 = 12
X = 10
So middle integer is 12

193
Two digit number

Suppose 72

2 → unit place
7 → Tens place

7* 10 + 2 = 72

ab = 10a + b

sum of digits = a+b

number formed after reversing the digits = ba

= 10b+a
6. Sum of the digits of a 2 digit number is 9. Number formed after reversing the digit is
27 more than original number then find the numbers?

ab → a+b =9 (1)

ab ba

ab = ba – 27
10a+b = 10b+a-27
9a-9b = -27
a-b = - 3 (2)
Solve (1) and (2) we get a= 3
b= 6
7. A two digit number product of digits is 12. If 36 is added to it the digits interchange
their places. Then find the number?

ab = 12
ab+36 = ba

2 * 6 = 12
26+36 = 62
So number is 26

Three digit number

abc

c → unit place

194
b → Tens place
a → Hundreds place

abc = 100a+10b+c

8. Sum of the 3 digit number is 6. The number formed after reversing the digit is 198
more than original number. If the middle digit is zero. What is the number?
a0b
a+0+b = 6
a+b = 6
a0b = b0a – 198
100a+b = 100b+a-198
99a – 99b = -198
a – b = -2 (1)
a+b=6 (2)

from (1) and (2)


a=2 b = 4 so number is 204
Problems on fractions
x/y
Where x = Numerator
y = Denominator

9. Sum of the Nr. And Dr. of a fraction is 22. If 2 is added in Nr. And 3 is added in Dr. the
fraction becomes ½. What is the fraction?
(X+2) /(22-x+3) = ½
2x+4 = 25-x
X=7
So, fraction is 7/15
10. Few cows and hens are there in certain place. If we count their heads the sum is 100.
If we count their legs the sum is 300. Find the number of hens and cows ?

Let number of cows be C

195
Then Hens 100-C
4c+2(100-c) = 300
2c = 100
C = 50
H = 50

1. A teacher has 120 sweets with him and distributes it equally. If there were 4 students
less, then each of the students will get one more .Find the number of students (TCS)
Let number of students X
Each student get Y sweets
Then XY = 120
(X – 4)(Y +1) = 120
XY+X-4Y-4 = 120
X-4-4Y = 0
X-4-4(120/X) = 0
X2-4X-480 = 0
(X-24)(x+20) = 0
X = 24
2. 40 shots are taken for a target. For each hit you get 50 paisa and pay 10 paisa for every
miss. After 40 shots you get Rs 5. How many shots hit the target? (CTS)
H = hit
M = miss
H + M = 40
50H – 10M = 500
5H – M = 50
H = 15
3. A can copy 50 papers in 10 hours, while A and B can copy 70 papers in 10 hours. How
many hours are required for B to copy 26 papers? (TCS)
A +B = 70 papers 10hours
A = 50 papers 10 hours
Then B = 20 papers 10 hours
So 26 papers can be copy (26/20) * 10 = 13 hours

196
4. The lowest temperature in a city is 1/3rd more than half the highest. The sum of the two
temperatures is 100. What is the lowest temperature? (TCS)
L+ H = 100
L = 1/2H+ (1/3) (1/2) H
L =2/3 H
H = 3/2 L
L + 3/2L = 100
L = 40

5. In a club there are male and female members. If 15 females quit than the number of
females is double the males. If 45 males quit than the number of females is five times the
number of males. Find the number of females and males? (CTS CAT)
Let number of males initially M
Number of females F
F – 15 = 2M
F = 5 (M- 45)
2M+15 = 5M – 225
240 = 3M
M = 80
F = 175

6. If 1/5th of a hive of bees flew to the Badamba flower, 1/3rd flew to the slandbara, 3 times
the difference of these two numbers flew to an arbour, and one bee continued to fly about,
attracted on each side by the fragrant ketaki and malati. What was the total number of
bees? (Infosys Wipro)
Let number of bees x
(X/5)+(x+3) +3((x/3)-(x/5)) +1 = x
X = 15

7. Candidates P and Q Contested in an election in a country where the voters pre-decide the
candidate they will vote for by giving a promise before the polling day. As per the promises

197
received from the voters, P was to win the election by a margin of 1 vote. However, on the
day of election, 1/7th of the candidates who had promised to vote for P, voted for Q.
similarly 1/9th of the candidates who had promised to vote for Q, shifted their votes to P. Q
finally won the election by 1 vote. The total number of voters in the election must be. (CTS
HCL)
P Q
Promised votes x y x – y =1
Actual votes x – (x/7)+ (y/9) y + (x/7) – (y/9)
Y+(x/7)-(y/9) – x+(x/7)-(y/9) = 1
9x -7y = 63
X= 28 y = 27
Total number of voters = 55

8. A group of friends go for dinner and gets bill of Rs 2400. Two of them say that they have
forgotten their purse so remaining make an extra contribution of Rs 100 to pay up the bill.
Tell the no. of person in that group. (CTS)
Let number of persons be X
(2400/(x -2)) - (2400/X) = 100
24x – 24x +48 = x2 – 2x
X2 – 2x – 48 = 0
(X-8)(X-6) = 0
X =8

9. Three friends divided some bullets equally. After they shot 4 bullets each, the total
number of bullets remaining is equal to the bullets, each one had after division. Find the
original number of bullets. (Accenture GATE Infosys Mahindra Satyam HCL)
Let x be number bullets initially
3((x/3) – 4) = x/3
X-12 = x/3
x =18

198
10. The total expense of a boarding house is partly fixed and partly variable with the number
of boarders. The charge is Rs 70 per head when there are 25 boarders and Rs 60 when there
are 50 boarders. Find the charge per head when there are 100 boarders? (TCS)
F+25V = 70*25 = 1750
F+50V = 60*50 = 3000
25V =1250
V =50
F =500
500 + 100*50 = 5500
5500/100 = 55

11. A tank is one fifth full and after one liter is added it becomes one fourth full. Find the
capacity.
Let tank capacity be X liters
(X/5)+1 = x/4
X =20 liters
12. 600 are to be seated, 10 benches were found to be less. Hence 2 more persons are
seated in each bench. How many benches should have been there? (Accenture Infosys)

Let x be the number of people seated in one bench


Xb = 600
(x+2)(b-10) = 600
Xb+2b-10x-20=600
b-5x-10 = 0
b-5(600/x)-10 = 0
b2-10b-3000 = 0
b =60
13. Two candles are of same length but of different thickness. One candle takes 4 hours to
burn completely whereas the other candle takes 5 hours. The light goes off and both the
candles are lit at the same time. When the light comes back it was observed that the length
of one candle is four times the length of the other. For what period the light was off.
Let each candle length be L

199
In 1 hour it burns L/4 and L/5 respectively
After T hours L – L/5T L - L/4T
L – L/5T = 4(L - L/4T)
T = 3.75
14. Due to increase of 30% in the price of eggs, 3 eggs less are available for Rs7.80. the
present rate of eggs per dozen is (IBPS)
Let E be the eggs
EP = 7.80
(E -3)1.3P = 7.80
EP/(E -3)1.3P= 7.80/7.80
E =1.3E -3.9
E = 13
13P = 7.80
P= 0.60
.60 *(30/100) = .18
.60+0.18 = .78
.78 * 12 = 9.36
15. Sidhu and Vicky are good friends. Each has some money. If Sidhu gives Rs 50/- to Vicky
then Vicky will have twice the money left with Sidhu but if Vicky gives Rs 10/- to Sidhu then
Sidhu will have thrice the money left with Vicky . How much money does each have?
2(S -50) = (V+50)
3(V -10) = S+10
By solving S= 98
V= 46

200
CODING – DECODING

A CODE is a system of singals. Therefore, Coding is a method of transmitting a message


between the sender and the receiver without a third person knowing it.
The Coding and Decoding Test is set up to judge the candidate’s ability to decipher the rule
that codes a particular word / message and break the code to decipher the message.
TYPE 1 : LETTER CODING
In these questions, the letters in a word are replaced by certain other letters
according to a specific rule to form its code. The candidate is required to detect the coding
pattern / rule and answer the questions accordingly.
Case I. To form the code for another word (CODING)

Ex. 1. In a certain code, TEACHER is written as VGCEJGT. How is CHILDREN written in that
code?
a. EJKNEGTP b. EGKNEITP c. EJKNFGTO d. EJKNFTGP
Sol:- Clearly, each letter in the word TEACHER is moved two steps forward to obtain the
corresponding letter of the code.
T E A C H E R
+2 +2 +2 +2 +2 +2 +2
V G C E J G T
Similarly, we have:
C H I L D R E N
+2 +2 +2 +2 +2 +2 +2 +2
E J K N F T G P
So, the desired code is EJKNFTGP. Hence, the answer is (d).

Ex. 2. In a certain code language, RUSTICATE is written as QTTUIDBSD. How would STATISTIC
be written in that code?
a. RSBUJTUHB b. RSBUITUHB c. RSBUIRSJD d. TUBUITUMB
Sol:- Clearly, the middle letter of the word remains unchanged in the code. Each of the first
two and the last two letters of the word is moved one step backward, while each of the

201
remaining letters is moved one step forward to obtain the corresponding letters of the
code.
R U S T I C A T E
-1 -1 +1 +1 +1 +1 1 -1
Q T T U I D B S D
Similarly, we have:
S T A T I S T I C
-1 -1 +1 +1 +1 +1 -1 -1
R S B U I T U H B
So, the required code is RSBUITUHB. Hence, the answer is (b).

Ex. 3. If ROAST is coded as PQYUR in a certain language, then how will SLOPPY be coded in
that language?
a. MRNAQN b. NRMNQA c. QNMRNA d. RANNMQ
Sol:- Clearly, the letters in the word ROAST are moved alternately two steps backward and
two steps forward to obtain the letters of the code. Thus, we have:
R O A S T S L O P P Y
-2 +2 -2 +2 -2 -2 +2 -2 +2 -2 +2
P Q Y U R Q N M R N A
So, the required code is QNMRNA. Hence, the answer is (c).

Ex. 4. In a certain system of coding, the word STATEMENT is written as TNEMETATS. In the
same system of coding, what should be the code for the word POLITICAL?
a. LACITILOP b. LCATILIOP c. OPILITACL d. None of these
Sol:- Clearly, the letters of the given word are written in a reverse order to obtain the code.
Reversing the order of letters in POLITICAL, we get LACITILOP, which is the required code.
Hence, the answer is (a).

Ex. 5. If HEALTH is written as GSKZDG, then how will NORTH be written in that code?
a. OPSUI b. GSQNM c. FRPML d/ IUSPO

202
Sol:- Clearly, the letters of the given word are written in a reverse order and then each letter
is moved one step back ward to obtain the code.
Reversing the order of letters in NORTH, we get HTRON. Thus, we have:
H T R O N
-1 -1 -1 -1 -1
G S Q N M
So, the required code is CSQNM, Hence, the answer is (b).

Ex. 6. In a certain code, BREAKTHROUGH is written as EAOUHRBRGHKT. How is


DISTRIBUTION written in that code?
a. TISTBUOBDIRI b. STTIBUONRIDI c. STTIBUDIONRI d.
RISTTIBUDION e. None of these
Sol:- let us divide the letters of the given word into pairs and label these pairs from 1 to 6.
z€ J€ È p€ É6 Êp
# ! $

Clearly, the code contains these pairs arranged in the order 2, 5, 4, 1, 6, 3.


Dividing the letters of the word DISTRIBUTION in pairs, we have:
€ z6 É–

Arranging these in the order 2, 5, 4, 1, 6, 3, we get the required code i.e.


STTIBUDIONRI. Hence, the answer is (c).

Ex. 7. In a certain code language, BEAT is written as YVZG, then what will be the code of
MILD?
a. ONRW b. NOWR c. ONWR d. NROW
Sol:- B, E, A, T are respectively the 2nd , 5th, 1st, 20th letters from the beginning of the English
alphabet. The letters of the code Y, V, Z, G are respectively the 2nd, 5th, 1st, and 20th letters
from the end of the English.
Similarly, M, I, L, D are respectively 13th, 9th, 12th, 4th,letters from the beginning of the
English alphabet. And, the 13th, 9th, 12th, 4th, letters from the end of the English alphabet are
N, R, O, W respectively. So, the required code is NROW.
Hence, the answer is (d).

203
TYPE 2 : DIRECT LETTER CODING
What we have studied till now was ‘rule-coding’ in which letters were assigned codes
according to a set pattern or rule concerning the movement or reordering of letters and one
needs to detect this hidden rule to decode a message. But, sometimes, particular letters are
made codes for particular letters without there being any set pattern. For example, let us
consider a language in which A is coded as W, C as P, E as T, L as Z, S as B and T as K. Then,
the code for CASTLE in that language is PWBKZT.
Such type of coding is called direct-coding.
In direct-coding, the code letters occur in the same sequence as the corresponding letters
occur in the words.
In questions on direct-coding, either the particular codes of letters are given or the codes or
two or more words are given and one is asked to find the codes of given words involving
only those letters for which the codes have already been mentioned.

Ex.1. If in a certain code, O is written as E, A as C, M as I, S as O, N as P, E as M, I as A, P as N


and C as S, then how will COMPANIES be written in that code?
a. SMINCPAMO b. SEIACPAMO c. SEINCPAMO d. SEINCPMIO e. None of these
Sol:- Substituting the letters of the given word with their respective codes, we have:
C O M P A N I E S

S E I N C P A M O
Hence, the answer is (c).

Ex. 2. If in a code language, PARENT is written as BDFGJK and CHILDREN is written as


MOXQUFGJ, how is REPRINT written in that code?
a. FGBFXJK b. FGBUXJK c. FGBFXGD d. BGBFXJK
Sol:- Observing the given words and codes, we notice that:
I. There is no apparent rule governing the coding;
II. Both the given words gave common codes corresponding to common letters
i.e. PARENT and CHILDREN have common in the word and ‘FGJ’ in common in
the code. This indicates that the code letters are in the same sequence as the
corresponding letters in the words;
III. REPRINT is formed by a combination of letters of PARENT and CHILDREN. All
the above three indicate that this is a question on direct-coding.

204
Thus, from the given words we have:

Letter P A R E N T C H I L D

Code B D F G J K M O X Q U

Thus, the code for REPRINT becomes FGBFXJK, Hence, the answer is (a).

Ex. 3. If the word EARTH be written as QPMZS in coded form, how can HEART be written
following the same coding?
a. SQPZM b. SQMPZ c. SPQZM d. SQPMZ
Sol:- Observing the above question, we may notice that HEART consists of the same letters
as EARTH and the four possible codes given as alternatives also consist of the same letter
codes as those in the code for EARTH. This indicates that this is a question on direct-coding.
Thus, we have:

Letter E A R T H

Code Q P M Z S

So, the code for HEART becomes SQPMZ.


Hence, the answer is (d).

Ex. 4. In a defence message, GET AWAY, FIRE BACK-WARDS, MOVE SLOW is coded as BEN
CDCI, QHOE PCTL DCOXU, ZMWE VFMD.
Based on the coding scheme, spot the codes for the following words:
1. OVER a. MWED b. MWEO c. MWOE d. MWZO
2. DEADLY a. XECXEI b. XEEXCI c. XECXFI d. XENXPI
3. REWARD a. OEDCOU b. OEDCOX c. OEDNXE d. OTDCOX
4. GREAT a. BOECN b. BOENC c. BOEHC d. BOEQN
Sol:- Observing the given message, we find that as such, no definite rule of coding seems to
follow. Also, whenever A occurs in the message C comes at the corresponding place in the
code. Similarly, E corresponds to E, D corresponds to W and so on. Thus, every letter in the
message has a particular code. This is direct-coding.
Thus, from the given message, we have:

Letter G E T A W Y F I R B C K D S M O V L

Code B E N C D I Q H O P T L X U Z M W F

205
1. The code for OVER is MWEO, So, the answer is (b).
2. The code for DEADLY is XECXFI. So, the answer is (c).
3. The code for REWARD is OEDCOX. So, the answer is (b).
4. The code for GREAT is BOECN. So, the answer is (a).

TYPE 3: NUMBER/SYMBOL CODING


In these questions, either numerical code values are assigned to a word or
alphabetical code letters are assigned to the numbers. The candidate is required to analyse
the code as per the questions.
Clearly, letters and numbers are correlated to each other in no other way except, in
relation to the position of the English alphabet. So, either this relation holos or the coding
has to be done as per a set of given rules. In all other cases, the question is one of direct-
coding.
Case 1. When numerical / Symbol codes are assigned to words.
Ex. 1. If MACHINE is coded as 19 – 7 – 9 – 14 – 15 – 20 – 11, how will you code DANGER?
a. 11 – 7 – 20 – 16 – 11 – 24 c. 10 – 7 – 20 – 13 – 11 - 24
b. 13 – 7 – 20 – 9 – 11 – 25 d. 13 – 7 – 20 – 10 – 11 – 25
Sol:- Clearly, every letter is assigned a numerical code obtained by adding 6 to the numeral
denoting the position of that letter in the English alphabet.
Thus, A is coded as (1 + 6) i.e. 7, B as (2 + 6) i.e. 8, C as (3 + 6) i.e. 9, ….., M as (13 +6) i.e.
19, ….., z as (26 + 6) i.e. 32.
Since, D, A, N, G, E, R are (4 + 6), (1 + 6), (14 + 6), (7 + 6), (5 + 6), (18 + 6) i.e. 10, 7, 20, 13, 11,
24. So, the code for DANGER is 10 – 7 – 20 – 13 – 11 – 24.
Hence, the answer is (c).

Ex. 2. If E = 5, PEN = 35, Then PAGE = ?


a. 27 b. 28 c. 29 d. 36
Sol:- Clearly, putting A = 1, B = 2, C = 3, D = 4, E = 5, ……., M = 13, ……, X = 24, Y = 25, Z =
26, we have:
PEN = P + E + N = 16 + 5 + 14 = 35.
So, PAGE = P + A + G + E = 16 + 1 + 7 + 5 = 29.
Hence, the answer is (c).

Ex. 3. If RED is coded as 6720, then how would GREEN be coded?


a. 1677199 b. 1677209 c. 16717209 d. 9207716

206
Sol:- Clearly, the order of letters in the word is reversed and then each letter is replaced by
the numeral denoting its position in the English alphabet. Next, 2 is added to each numeral
and the numerals so obtained are joined together physically to get the code. Thus, we have:
RED DER 4/5/18 6/7/20 6720.
GREEN NEERG 14/5/5/18/7 16/7/7/20/9 1677209.
Hence, the answer is (b).

Ex. 4. Study the following letters and their corresponding digit codes followed by certain
conditions of coding and answer the questions given below them by finding out which of the
digit combinations given in (a), (b), (c) and (d) is the coded form of the letter-groups given in
each question and mark your answer accordingly.

Letter P N A J I R E B U K

Digit codes 5 3 9 1 4 6 2 7 0 8

Conditions: (1) If both the first and the last letters in the group are vowels, both should be
coded as $.
(2) If both the first and the last letters in the group are consonants, both should be coded as
#.
1. RBUKAE
a. #70892 b. 670892 c. 670982 d. 607892 e. None of these
2. KUNAJB
a. 803917 b. $0391$ c. #0391# d. #0392$ e. None of these
3. EBNAPI
a. 273954 b. $7395$ c. 7395# d. $7395# e. None of these
Sol:- 1. Clearly, the given letter-group begins with a consonant and ends with a vowel. So,
each letter must by replaced by individual digit code. Thus, the desired code is 670892.
Hence, the answer is (b).
2. Clearly, the given letter-group begins with a consonant and also ends with a consonant.
So, each of the first and last and last letters must be coded as # while the middle four letters
must be replaced by individual digit codes. Thus, the desired code is #0391#. Hence, the
answer is (c).
3. Clearly, the given letter-group begins with and also ends with a vowel. So, each of the first
and last letters must be coded as $ while the middle four letters must be replaced by
individual digit codes. Thus, the desired code is $7395$.
Hence, the answer is (b).
207
Ex. 5. In a certain code, RAIN is written as 8$%6 and MORE is written as 7#8@. How is
REMAIN written in that code?
a. #@7$%6 b. #a&$%6 c. 7@#$%6 d. 8@7$%6 e. None of these
Sol:- Clearly, no apparent rule of coding is visible in the given codes. Also, the given words
have ‘R’ in common and ‘8’ in common at the corresponding places in the code. This implies
that letters have been assigned set numeral/symbol; codes i.e. this is a question of direct-
coding. Thus, we may sort out the individual codes as under:

Letter R A I N M O E

Code 8 $ % 6 7 # @

Thus, the code for REMAIN is 8@7$%6.


Hence, the answer is (d).

Ex. 6. If in a certain language, CHARCOAL is coded as 45164913 and MORALE is coded as


296137, how is the word ALLOCHRE coded in that language?
Sol:- Clearly, this is a question of direct-coding. Thus, the individual codes are as under:

Letter C H A R O L M E

Code 4 5 1 6 9 3 2 7

Thus, the code for ALLOCHRE is 13394567.


Hence, the answer is (c).

TYPE 5 : SUBSTITUTION
In this type of questions, some particular words are assigned certain substituted
names. Then a question is asked that is to be answered in the substituted code language.
Ex. 1. If ‘cook’ is called ‘butler’, is called ‘manager’, ‘manager’ is called ‘teacher’, ‘teacher’ is
called ‘clerk’ and ‘clerk’ is called ‘principal’, who will teach in a class?
a. Cook b. Butler c. Manager d. Teacher e. Clerk
Sol:- Clearly, a ‘teacher’ teaches in a class and as given, ‘teacher’ is called ‘clerk’. So, a ‘clerk’
will teach in the class. Hence, the answer is (e).

Ex. 2. If ‘diamond’ is called ‘gold’, ‘gold’ is called ‘silver’, ‘silver’ is called ‘ruby’ and ‘ruby’ is
called ‘emerald’, which is the cheapest jewel?

208
a. Diamond b. Silver c. Gold d. Ruby e. Emerald
Sol:- We know that ‘silver’ is cheapest. But as given, ‘silver’ is called ‘ruby’. So, ‘ruby’ is the
cheapest. Hence, the answer is (d).

Ex. 3. If ‘eye’ is called ‘hand’, ‘hand’ is called ‘mouth’, ‘mouth’ is called ‘ear’, ‘ear’ is called
‘nose’ and ‘nose’ is called ‘tongue’, with which of the following would a person hear?
a. Eye b. Mouth c. Nose d. Ear e. Tongue
Sol:- A person hears with his ‘ear’. But as per the given information, ‘ear’ is called ‘nose’. So,
a person will hear with the ‘nose’. Hence, the answer is (c).

TYPE 6 : DECIPHERING MESSAGE WORD CODES


In this type of questions, some messages are given in the coded language and the
code for a particular word or message is asked. To analyse such codes, any two messages
bearing a common word are picked up. The common code-word will thus represent that
word. Proceeding similarly by picking up all possible combinations of two, the entire
message can be decoded and the codes for individual words found.
Ex. 1. In a certain language, ‘sun shines brightly’ is written as ‘ba lo sul’, houses are brightly
lit’ as ‘kado ula ari ba’ and ‘light comes from sun’ as ‘dopi kup lo nro’. What code-words are
written for ‘sun’ and ‘brightly’?
a. ba, sul b. sul, lo c. lo, ba d. ba, lo
Sol:- In the first and third statements, the common word is ‘sun’ and the common code-
word is ‘lo’. So, ‘lo’ is the code for ‘sun’.
In the first and second statements, the common word is ‘brightly’ and the common
code-word is ‘ba’. So, ‘ba’ is the code for ‘brightly’.
Hence, the answer is (c).

Ex. 2. If in a certain language, ‘oka peru’ means ‘fine cloth’; ‘meta lisa’ means ‘clear water’
and ‘dona lisa peru’ means ‘fine clear weather’. Which word in that language means
‘weather’?
a. peru b. oka c. meta d. done
Sol:- In the first and third statements, the common code-word is ‘peru’ and the common
word is ‘fine’. So, ‘peru’ means ‘fine’.
In the second and third statements, the common code-word is ‘lisa’ and the common word
is ‘clear’. So, ‘lisa’ means ‘clear’.

209
Thus, in the third statement, ‘lisa’ means ‘clear’ and ‘peru’ means ‘fine’. So, ‘dona’ means
‘weather’.
Hence, the answer is (d).

Ex. 3. Read the information given below to answer the questions that follow:
In a certain code language,
i. ‘pit na sa’ means ‘you are welcome’;
ii. ‘na ho pa la’ means ‘they are very good’;
iii. ‘ka da la’ means ‘who is good’;
iv. ‘od ho pit la’ means ‘they welcome good people’.
1. Which of the following means ‘people’ in that code language?
a. od b. la c. ho d. pit e. Data inadequate
2. Which of the following means ‘very’ in that code language?
a. pa b. na c. da d. Data inadequate e. None of these
3. Which of the following statements(s) is/are redundant to answer the above two
questions?
a. None b. (i) and (ii) c. (ii) or (iii) d. (i) or (iv) e. None of these

Sol:- 1. In statements (i) and (iv), the common code-word is ‘pit’ and the code-word is
‘welcome’. So, ‘pit’ means ‘welcome’.
In statements (ii) and (iv), the common code-word are ‘ho’ and ‘la’ and the common words
are ‘they’ and ‘good’. So, ‘ho’ and ‘la’ mean ‘they’ and ‘good’. Thus, in (iv), the remaining
code-word. i.e. ‘od’ means ‘people’.
Hence, the answer is (a).
2. From 1, we know that ‘ho’ and ‘la’ are codes for ‘they’ and ‘good’.
Now, in statements (i) and (ii), the common code-word is ‘na’ and the common word is ‘are’.
So, ‘na’ means ‘aer’.
Thus, in (ii), the remaining code-word i.e. ‘pa’ means ‘very’.
Hence, the answer is (a).
3, Clearly, to answer the above two questions, we used statements (i), (ii) and (iv), and
didn’t require (iii).
So, (iii) is redundant.
Hence, the answer is (c).

210
Ex. 4. Study the following information to answer the given questions:
In a certain code language,
i. ‘ka bi pu ya’ means ‘you are very intelligent’;
ii. ‘ya lo ka wo’ means they seem very intelligent’;
iii. ‘la pu le’ means ‘you can see’;
iv. ‘sun pun yun ya’ means how intelligent she is’.
In each of the following questions, find which of the above statements is/are required to
find the exact code of the given word. If the code cannot be ascertained even by using
all the given statements, then your answer is (e) i.e. Data inadequate.
1. You
a. I and II b. II and III c. I and III d. None of these
2. How
a. I and II b. I and IV c. I, II and IV d. All the four
3. are
a. I, II and III b. I, II and IV c. I, III and IV d. All the four
4. Very
a. I, II and III b. I, II and IV c. I, III and IV d. All the four
5. They
a. I and II b. I, II and IV c. II, III and IV d. All the four
6. Intelligent
a. I and II b. I and IV c. II and IV d. Either (b) or (c)
7. Can
a. I and III b. I, II and III c. III and IV d. Either (a) or (c)
Sol:- 1. Clearly, ‘you’ is the only common word in statements I and III. So, the code for ‘you’
is the common code-word in I and III, which is ‘pu’.
Hence, the answer is (c).
2. Statement IV has only one word ‘intelligent’ which is common in two or more of the given
statements. So, only the code for ‘intelligent’ can be found; while it is not possible to
determine the exact codes for ‘how’, ‘she’ and ‘is’. Thus, the given data are inadequate.
Hence, the answer is (e).
3.Clearly, in I, to find the code for ‘are’, we need to first determine the codes for ‘you’, ‘very’
and ‘intelligent’.
From I and III, the code for ‘you’ is ‘pu’.
From I and II, the codes for ‘very’ and ‘intelligent’ are ‘ya’ and ‘ka’.
So, in I, the remaining code-word i.e. ‘bi’ stands for ‘are’.
Thus, the statements I, II and III are required. Hence, the answer is (a).

211
4.Clearly, there are no two statements in which ‘very’ is the only common word. So, the
code for ‘very’ can be4 determined by finding the codes of the other three words in I.
Now, from 1 and 3, we know that I, II and III are required to find the codes for ‘you’ and
‘are’.
Since ‘intelligent’ is the only word common to (I and IV) or (II and IV), so the code-word
common to any of these pairs of statements stands for ‘intelligent’.
After getting the codes for ‘you’, ‘are’ and ‘intelligent’, the remaining code-word in I stands
for ‘very’.
Thus, all the four statements are required.
Hence, the answer is (d).
5.Statement II has two uncommon words-‘they’ and ‘seem’. So, it is not possible to
determine the exact code for any of these words.
Hence, the answer is (e).
6.As discussed in 4, we need (I and IV) or (II and IV) to determine the code for ‘inttelligent’.
Hence, the answer is (d).
7.statement III has two uncommon words-‘can’ and ‘see’. So, it is not possible to determine
the exact code for any of these words.
Hence, the answer is (e).

Ex. 5. In a certain code language, ‘si po re’ means ‘book is thick’, ‘ti na re’ means ‘bag is
heavy’. ‘ka si’ means ‘interesting book’ and ‘de ti’ means ‘that bag’. What should stand for
‘that is interesting’ in that code language?
a. Ka de re b. ti po ka c. ka re na d. de si re e. None of these
Sol:- In the first and second statements, the common code-word is ‘re’ and the common
word is ‘is’. So, ‘re’ means ‘is’.
In the first and third statements, the common code-word is ‘si’ and the common word is
‘book’. So, ‘si’ means ‘book’. Thus, the other code-word in the third statement i.e. ‘ka’
means ‘interesting’.
In the second and fourth statements, the common code-word is ‘ti’ and the common word
is ‘bag’. So, ‘ti’ means ‘bag’. Thus, the other code-word in the fourth statement i.e. ‘de’
means ‘that’.
Thus, the codes for ‘that’, ‘is’ and ‘interesting’ are ‘de’, ‘re’ and ‘ka’ respectively. So, any
combination of these three codes shall be the required code.
Hence, the answer is (a).

212
TYPE 7 : DECIPHERING NUMBER AND SYMBOL CODES FOR MESSAGES
In this type of questions, a few groups of numbers/symbols, each coding a certain
message, are given. Through a comparison of the given coded messages, taking two at a
time, the candidate is required to find the number/symbol code for each word and then
formulate the code for the given message.
Ex. 1. In a certain code language, ‘123’ means ‘bright little boy’, ‘145’ means ‘tall big boy’
and ‘637’ means ‘beautiful little flower’. Which digit in that language means ‘bright’?
a. 1 b. 2 c. 3 d. 4
Sol:- In the first and second statements, the common code digit is ‘I’ and the common word
is ‘boy’. So, ‘I’ means ‘boy’. In the first and third statements, the common code digit is ‘3’
and the common word is ‘tittle’. So, ‘3’ means ‘tittle’. Thus, in the first statement, ‘2’ means
‘bright’.
Hence, the answer is (b).

Ex. 2. In a certain code language, ‘go for morning walk’ is written as ‘$ *?#’, ‘good for health’
is written as ‘ℒ?@’ and ‘good to walk fast’ is written as ‘+@↑#’, then what is the code for
‘health’ in that code language?
a. + b. # c. ‘ℒ d. ? e. None of these
Sol:- In the first and second statements, the common code symbol is ‘?’ and the common
word is ‘for’. So, ‘?’ means ‘for’.
In the second and third statements, the common code symbol is ‘@’ and the common word
is ‘good’. So, ‘@’ means ‘good’. Thus, in the second statement, ‘‘ℒ’ means ‘health’.
Hence, the answer is (c).
Ex. 3. In a certain code language, ‘617’ means ‘sweet and hot’, ‘735’ means ‘coffee is sweet’
and ‘263’ means ‘tea is hot’. Which of the following would mean ‘coffee is hot’?
a. 731 b. 536 c. 367 d. 753 e. None of these
Sol:- In the first and third statements, the common code digit is ‘6’ and the common word is
‘hot’. So, ‘6’ means ‘hot’.
In the second and third statements, the common code digit is ‘3’ and the common word is
‘is’. So, ‘3’ means ‘is’.
In the first and second statements, the common code digit is ‘7’ and the common word is
‘sweet’. So, in the second statement, ‘5’ means ‘coffee’.
Clearly, ‘536’ would mean ‘coffee is hot’.
Hence, the answer is (b).

213
Data interpretation
TABULATION

1. The following table gives the sales of batteries manufactured by a company over the
years. Study the table and answer the questions that follow.

NUMBER OF DIFFERENT TYPES OF BATTERIES SOLD BY A COMPANY OVER THE


YEARS (NUMBERS IN THOUSANDS)
TYPES OF BATTERIES
Year
4AT 7AH 32AH 35AH 55AH Total

1992 75 144 114 102 108 533

1993 90 126 102 84 126 528

1994 96 114 75 105 135 525

1995 105 90 150 90 75 510

1996 90 75 135 75 90 465

1997 105 60 165 45 120 495

1998 115 85 160 100 145 605

1. The total sales of all seven years is the maximum for which battery?
a. 4AH b. 7AH c. 32AH d. 35AH e. 55AH
2. What is the difference in the number of 35AH batteries sold in 1993 and 1997?
a. 24000 b. 28000 c. 35000 d. 39000 e. 42000
3. The percentage of 4AH batteries sold to the total number of batteries sold was
maximum in the year:
a. 1994 b. 1995 c. 1996 d. 1997 e. 1998
4. In the case of which battery there was a continuous decrease in sales from 1992
to 1997?
a. 4AH b. 7AH c. 32AH d. 35AH e. 55AH
5. What was the approximate percentage increase in the sales of 55AH batteries in
1998 compared to that in 1992?
a. 28% b. 31% c. 33% d. 34% e. 37%
Sol:- 1. (c): The total sales (in thousands) of all the seven years for various
batteries are:
For 4AH = 75 + 90 + 96 + 105 + 90 + 105 + 115 = 676
For 7AH = 144 + 126 + 114 + 90 + 75 + 60 + 85 = 694

214
For 32AH = 114 + 102 + 75 + 150 + 135 + 165 + 160 = 901
For 35AH = 102 + 84 + 105 + 90 + 75 + 45 + 100 = 601
For 55AH = 108 + 126 + 135 + 75 + 90 + 120 + 145 = 799
Clearly, sales are maximum in case of 32AH batteries.

Sol:- 2. (d): Required difference = [(84 – 45) × 1000] = 39000.

Sol:- 3. (d) : The percentages of sales of 4AH batteries to the total sales in
different years are:
K &%
For 1992 =[ × 100 ]%= 13.81%; For 1993 = [ × 100 ]%= 17.05%
!#
&$ %
For 1994 =[ × 100 ]%= 18.29%; For 1995 =[ × 100 ]%= 20.59%
%
&$ %
For 1996 =[ !$ × 100 ]%= 19.35%; For 1997 =[ !& × 100 ]%=21.21%

For 1998 = =[ $% × 100 ]%= 19.01%

Clearly, the percentage is maximum in 1997.

Sol:- 4. (b): From the table it is clear that the sales of 7AH batteries have been
decreasing continuously from 1992 to 1997.

( ! 3 % )
Sol:- 5. (d): Required Percentage =[ × 100 ]%=34.26% = 34%
%

2. Study the following table carefully and answer these questions :

NUMBER OF CANDIDATES APPEARED AND QUALIFIED IN A


COMPETITIVE EXAMINATION FROM DIFFERENT STATES OVER THE YEARS
Year/ 1997 1998 1999 2000 2001
state
App. Qual. App. Qual. App. Qual. App. Qual. App. Qua.

M 5200 720 8500 980 7400 850 6800 775 9500 1125

N 7500 840 9200 1050 8450 920 9200 980 8800 1020

P 6400 780 8800 1020 7800 890 8750 1010 9750 1250

215
Q 8100 950 9500 1240 8700 980 9700 1200 8950 995

R 7800 870 7600 940 9800 1350 7600 945 7990 885

1. Combining the states P and Q together in 1998, what is the percentage of the
candidates qualified to that of the candidates appeared?
a. 10.87%b. 11.49% c. 12.35% d. 12.54% e. 13/05%
2. The percentage of the total number of qualified candidates to the total number
of appeared candidates among all the five states in 1999 is :
a. 11.49%b. 11.84% c. 12.21% d. 12.57% e. 12.73%
3. What is the percentage of candidates qualified from state N for all the years
together, over the candidates appeared from State N during all the years
together?
a. 12.36%b. 12.16% c. 11.47% d. 11.15% e. None of these
4. What is the average of candidates who appeared from State Q during the given
years?
a. 8700 b. 8760 c. 8810 d. 8920 e. 8900
5. In which of the given years the number of candidates appeared from State P has
approximately what percentage of the total number of candidates qualified from
all the states together in 1998?
a. 1997 b. 1998 c. 1999 d. 2000 e. 2001
6. Total number of candidates qualified from all the states together in 1997 is
approximately what percentage of the total number of candidates qualified from
all the states together in 1998?
a. 72% b. 77% c. 80% d. 83% e. 86%
( % % !%) $%
Sol:- 1. (c): Required Percentage = Í( × 100Î % = Í × 100Î %
%% & %%) #%%

= 12.35%.

( % & % &% & % # %)


Sol:- 2.(b):Required Percentage= Í(K!%% × 100 Î%
! % K %% K%% & %%)

!&&%
= Í! %
× 100Î% = 11.84%.

( !% % % & % & % % %)
Sol:- 3. (d):Required Percentage = Í(K × 100Î%
%% & %% ! % & %% %%)

! %
= Í!# × 100Î% = 11.15%.
%

( %% & %% K%% &K%% & % !!& %


Sol:- 4. (e):Required average =Í Î% = Í Î = 8990

216
Sol:- 5.(e): The percentages of candidates qualified to candidates appeared from State
P during different years are:
K % % %
For 1997 = Í × 100Î% = 12.19%; For 1998 = Í × 100Î% = 11.59%
$!%% %%

&% % %
For 1999 =ÍK %%
× 100Î% = 11.41%; For 2000 = Í %%
× 100Î% = 11.54%
%
For 2001 = Í × 100Î% = 12.82%
&K %

Maximum percentage is for the year 2001.

(K % !% K % & % K%)
Sol:- 6. (c):Required percentage = Í(& × 100Î%
% % % % % !% &!%)

! $%
=Í × 100Î% = 79.54% = 80%.
#%

BAR GRAPHS
3. The bar graph given below shows the foreign exchange reserves of a coutry (in
million US$) from 1991 – 92 to 1998 – 89. Answer the questions based on this graph.

FOREIGN EXCHANGE RESERVES OF A COUNTRY


(in million US $)

6000

5000
5040
4000 4320
3720
3000 3360
3120 3120
2640 2520
2000

1000

0
1991-92 1992-93 1993-94 1994-95 1995-96 1996-97 1997-98 1998-99

1. The foreign exchange reserves in 1997-98 was how many times that in 1994-95?
a. 0.7 b. 1.2 c. 1.4 d. 1.5 e. 1.8
2. What was the percentage increase in the foreign exchange reserves in 1997-98
over 1993-94?
a. 100 b. 150 c. 200 d. 620 e. 2520

217
3. For which year, the percent increase of foreign exchange reserves over the
previous year, is the highest?
a. 1992-93 b. 1993-94 c. 1994-95 d. 1996-97 e. 1997-98
4. The foreign exchange reserves in 1996-97 were approximately what percent of
the average foreign exchange reserves over the period under review?
a. 95% b. 110% c. 115% d. 125% e. 140%
5. The ratio of the number of year, in which the foreign exchange reserves are
above the average reserves, to those in which the reserves are below the
average reserves, is:
a. 2 : 6 b. 3 : 4 c. 3 : 5 d. 4 : 4 e. 5 : 3
%!%
Sol:- 1.(d): Required ratio = = 1.5
##$%

Sol:- 2.(a): Foreign exchange reserves in 1997-98 = 5040 million US$


Foreign exchange reserves in 1993-94 = 2520 million US$.
∴ Increase = (5040 – 2520) = 2520 million US$.
%
∴ Percentage increase = Í × 100Î% = 100%
%

Sol:- 3. (a): There is an increase in foreign exchange reserves during the years
1992-93, 1994-95, 1996-97 and 1997-98 as compared to previous year (as show
n by bar-graph)
The percentage increase in reserves during these years compared to previous
year are:
(#K %3 $!%)
I. For 1992-93 = Í × 100Î% = 40.91%
$#%

(##$%3 %)
II. For 1994-95 = %
× 100% = 33.33%

(!# %3# %)
III. For 1996-97 = Í × 100Î% = 38.46%
# %

( %!%3!# %)
IV. For 1997-98 = Í × 100Î% = 16.67%
!# %

Clearly, the percentage increase over previous year is highest for 1992-93.

Sol:- 4. (d): Average foreign exchange reserves over the given period.
= Í × (2640 + 3720 + 2520 + 3360 + 3120 + 4320 + 5040 +
3120)Î million US$
= 3480 million US$
Foreign exchange reserves in 1996-97 = 4320 million US$
!# %
∴ Required Percentage = Í#! % × 100Î% = 124.14% = 125%

218
Sol:- 5. (C): Average foreign exchange reserves in 1996-97 = 4320 million US$. The
country had reserves above 3480 million US $ during the years 1992-93, 1996-97 and
1997-98 i.e. for 3 years and below 3480 million US $ during the years 1991-92, 1993-
94, 1994-95, 1995-96, and 1998-99 i.e. for 5 years.
Hence, required ratio = 3 : 5.

4. The bar-graph provided on next page gives the sales of books (in thousand numbers)
from six branches of a publishing company during two consecutive years 2000 and
2001. Answer the questions based on this bar- graph.
Sales of books (in thousand numbers ) from six Branches- B1,B2, B3, B4 ,B5 and B6 of
a publishing Company in 2000 and 2001.

140

120 110
105
Sales (in thousand numbers)

100 95 95 95
85
80 80
80 75 75
70
65
60

40

20

0
B1 B2 B3 B4 B5 B6
Bronches

2000 Branches 2001 branches

1. Total sales of branches B1, B3 and B5 together for both the years(in thousand
numbers) is:
a. 250 b. 310 c. 435 d. 560 e. 585
2. Total sales of branch B6 for both the years is what percent of the total sales of
branch B3 for both the years?
a. 68.54% b. 71.11% c. 73.17% d. 75.55% e. 77.26%
3. What is the average sale of all the branches (in thousand numbers) for the year
2000?
a. 73 b. 80 c. 83 d. 88 e. 96

219
4. What is the ratio of the total sales of branch B2 for both years to the total sales of
branch B4 for both years?
a. 2 : 3 b. 3 : 5 c. 4 : 5 d. 5 : 7 e. 7 : 9
5. What percent of the average sales of branches B1, B2 and B3 in 2001 is the
average sales of branches B1, B3 and B6 in 2000?
a. 75% b. 77.5% c. 82.5% d. 85% e. 87.5%
Sol:- 1. (d): Total sales of branches B1, B3 and B5 for both the years (in thousand
numbers) = (80 + 105) + (95 + 110) ( 75 + 95) = 560.
(K% %) %
Sol:- 2. (c): Required Percentage=Í(& × 100Î% =Í × 100Î% = 7317%
%) %

Sol:- 3. (b): Average sales of all the six branches (in thousand numbers) for the year
2000 = × [80 + 75 + 95 + 85 + 75 + 70] = 80.
(K $ ) !% K
Sol:- 4.(e): Required ratio = ( & )
= = .
% &

Sol:- 5. (e): Average sales (in thousand numbers) of branches B1, B3 and B6 in 2000
!
= # × (80 + 95 + 70) = Í Î.
#

Average sales (in thousand numbers) of branches B1, B2 and B3 in 2001


%
= × (105 + 65 + 110) = .
# #
,w-
Í Î !
∴ Required Percentage = Ï ,/Ð
v
× 100Ñ% = Í × 100Î% = 87.5%.
Í Î %
v

220
PIE-CHARTS
5. The following pie-chart shows the sources of funds to be collected by the
National Highways Authority of India (NHAI) for its Phase II projects. Study the pie-
chart and answer the questions that follow.
SOURCES OF FUNDS TO BE ARRANGED BY NHAI
FOR PHASE II PROJECTS (IN CRORES RS)

Crores RS

External Assistance
[VALUE]

Market Borrowing SPVS [VALUE]


[VALUE]

Toll [VALUE]

annuity [VALUE]

Total funds to arranged for Projects (Phase II) = Rs. 57,600 crores.
1. Near about 20% of the funds are to be arranged through:
a. SPVS b. External Assistance c. Annuity d. Market Borrowing
2. The central angle corresponding to Market Borrowing is:
a. 52° b. 137.8% c. 187.2° d. 192.4°
3. The approximate ratio of the funds to be arranged through Toll and that
through Market Borrowing is:
a. 2 : 9 b. 1 : 6 c. 3 : 11 d. 2 : 5
4. If NHAI could receive a total of Rs. 9695 crores as External Assistance, by what
percent (approximately) should it increase the Market Borrowing to arrange for
the shortage of funds?
a. 4.5% b. 7.5% c. 6% d. 8%
5. If the toll is to be collected through an outsourced agency by allowing a
maximum 10% commission, how amount should be permitted to be collected
by the outsourced agency, so that the project is supported with Rs. 4910
crores?
a. Rs. 6213 b. Rs. 5827 c. Rs. 5401 d. Rs. 5216

Sol:- 1. (b): 20% of the total funds to be arranged = Rs. (20% of 57600) crores

221
= Rs. 11520 crores = Rs, 11486 crores
Rs. 11486 crores is the amount of funds to be arranged through
External Assistance.
Sol:- 2. (c): Central angle corresponding to Market Borrowing
&&
=Í × 360°Î = 187.2°
K$%%
!& %
Sol:- 3. (b): Required ratio = = =
&& $. $

Sol:- 4. (c): Shortage of funds arranged through External Assistance


= Rs. (11486 – 9695) crores = Rs. 1791 crores.
∴ Increase required in Market Borrowings = Rs. 1791 crores
K&
Percentage increase required = Í × 100Î% = 5.98% = 6%
&&

Sol:- 5. (c): Amount permitted = (Funds required from Toll for projects of
Phase II) + (10% of these funds)
= Rs. 4910 crores + Rs. (10% of of 4910) crores
= Rs. (4910 + 491) crores = Rs. 5401 crores

6. The pie-chart provided below gives the distribution of land (in a village) under
various food crops. Study the pie-chart carefully and answer the questions that
follow.
DISTRIBUTION OF AREAS (IN ACRES) UNDER VARIOUS FOOD CROPS

in acres
Wheat [VALUE] ° °
Rice[VALUE]

Barley [VALUE]°

Jowar [VALUE] ° °
Others [VALUE]

Bajra [VALUE]°
Maize [VALUE] °

222
1. Which combination of there crops contribute to 50% of the total area under the
food crops?
a. Wheat, Barley and Jowar c. Rice, Wheat and Barley
b. Rice, Wheat and Jowar d. Bajra, Maize and Rice
2. If the total area under Jowar was 1.5 million acres, then what was the area (in
million acres) under rice?
a. 6 b. 7.5 c. 9 d. 4.5
3. It the production of wheat is 6 times that of barley, then what is the ratio
between the yield per acre of wheat and barley?
a. 3 : 2 b. 3 : 1 c. 12 : 1 d. 2 : 3
4. If the yield per acre of rice was 50% more than that of barley, then the
production of barley is what percent of that of rice?
a. 30% b. 33#% c. 35% d. 36%
5. If the total area goes up by 5%, and the area under wheat production goes up
by 12%, then what will be the angle for wheat in the new pie-chart?
a. 62.4° b. 76.8° c. 80.6° d. 84.2°
Sol:-1. (c): The total of the central angles corresponding to the three crops
which cover 50% of the total area, should be 180°. Now, the total of the central
angles for the given combinations are:
(i) Wheat, Barley and Jowar = (72° + 36° + 18°) = 126°
(ii) Rice, Wheat and Jowar = (72° + 72° + 18°) = 162°
(iii) Rice, Wheat and Barley = (72° + 72° + 36°) = 180°
(iv) Bajra, Maize and Rice = (18° + 45° + 72°) = 135°
Clearly, (iii) is the required combination.
Sol:- 2. (a): The area under any of the food crops is proportional to the central
angle corresponding to that crop.
Let, the area under rice production be x million acres.
(K × . )
Then, 18 : 72 = 1.5 : x ⇒ x = = 6.

Thus, the area under rice production = 6 million acres.


Sol:-3. (b): Let the total production of barley be T tonnes and z acres of land be
put under barley production.
Then, the total production of wheat = (6T) tonnes.
Also, area under wheat production = (2z)
acres.
„‘•“ ŠNÔ•‘ ÕÖ•“” בŒÔŠØ”•ŒN K °
∵ „‘•“ ŠNÔ•‘ ˆ“‘Ù•Ú ×‘ŒÔŠØ”•ŒN = #$° = 2
Ò í
and therefore, Area under wheat = 2 × Area under barley = (2z) acres

223
$ #
Now, yield per acre for wheat = Í î Î tonnes/acre = Í î Î tonnes/ acre

and yield per acre for barley = Í Î tonnes/ acre.


ï
# ð
∴ Required Ratio = Í ðî
î
Î = 3 : 1.

Sol:- 4. (b): Let z acres of land be put under barley production.


I( 7 ( ( ( 7 K °
Then, = = 2.
I( 7 ( — ( ( 7 #$°

∴ Area under Rice production = 2 × area under barley production = (2z) acres
Now, if tonnes be the yield per acre of barley then, yield per acre of rice
#
= (p + 50% of p) tonnes = Í ñÎ tonnes.

∴ Total production of rice = (yield per acre) × (area under production)


#
= Í ñÎ × 2z = (3pz) tonnes.

And, Total production of barley = (pz) tonnes.


î
∴ Percentage production of barley to that of rice = Í# î × 100Î% = 33 #%.

Sol:- 5. (b): Initially, let t acres be the total area under consideration.
K
Then, area under wheat production initially was = Í#$% ¯Î acres = Í Î acres.

Now, if the total area under consideration be increased by 5%, then the new
%
value of the total area = Í %%
¯Î acres.
Also, if the area under wheat production be increased by 12%, then the new
value of the under wheat = ò Í 12% ôõ Îö acres = Í %%
Î acres.
∴ Central angle corresponding to wheat in the new pie-chart
..,ø
I( 7 ( ÷ ( ÷) Í Î
=Í × 360°Î = Ï -ÐÐ
.Ð-ø × 360Ñ = 76.8°
( ( ÷) Í Î
.ÐÐ

224
LINE-GRAPHS

7. In a school the periodical examinations are held every second month. In a


session during Apr. 2001 – Mar. 2002, a student of Class IX appeared for each
of the periodical exams. The aggregate marks obtained by him in each
periodical exam are represented in the line-graph given below. Study the graph
and answer the questions based on it.

MARKS OBTAINED BY A STUDENT IN SIX PERIODICAL EXAMS HELD IN EVERY


TWO MONTHS DURING THE YEAR IN THE SESSION 2001-02
Maximum Total Marks in each Periodical Exam = 500

420
410 410
0
400 400
0
390 390
0
Marks

380 380
0
370 370
0
360 360
0
350
340
330
Apr-01 Jun-01 Aug-01 Oct-01 Dec-01 Feb-o2

Periodical Exams

1. The total number of marks obtained in Feb. 02 is what percent of the total
marks obtained in Apr. 01?
a. 110% b. 112.5% c. 115% d. 116.5% e. 117.5%
2. What are the average marks obtained by the student in all the periodical
exams during the session?
a. 373 b. 379 c. 381 d. 385 e. 389
3. What is the percentage of marks obtained by the student in the periodical
exams of Aug. 01 and Oct. 01 taken together?
a. 73.25% b. 75.5% c. 77% d. 78.75% e. 79.5%
4. In which periodical exams there is a fall in percentage of marks as compared
to the previous periodical exams?
a. None b. Jun. 01 c. Oct. 01 d. Feb. 02 e. None of these
5. In which periodical exams did the student obtain the highest percentage
increase in marks over the previous periodical exams?
a. Jun. 01 b. Aug. 01 c. Oct. 01 d. Dec. 01 d. Feb. 02

225
Sol:- Here it is clear from the graph that the student obtained 360, 365, 370,
385, 400 and 405 marks in periodical exams held in Apr. 01, Jun. 01, Aug. 01,
Oct. 01. Dec. 01 and Feb. 02 respectively.
!%
Sol:- 1. (a): Required percentage = Í × 100Î% = 112.5%.
#$%

Sol:- 2. (c): Average marks obtained in all the periodical exams

= × ù360 + 365 + 370 + 385 + 400 + 405] = 380.83 = 381.


$

(#K% # )
Sol:- 3. (b): Required percentage = Í × 100Î%
( %% %%)

K
=Í × 100Î% = 75.5%.
%%%

Sol:- 4.(a): As is clear from the graph, the total marks obtained in periodical
exams, go on increasing. Since, the maximum marks for all the periodical
exams are same, it implies that the percentage of marks also goes on
increasing. Thus, in none of the periodical exams, there is a fall in
percentage of marks compared to the previous exam.
Sol:-5. (c): Percentage increase in marks in various periodical exams
compared to the previous exams are:
(#$ 3#$%)
For Jun. 01 = Í #$%
× 100Î% = 1.39%
(#K%3#$ )
For Aug. 01 = Í × 100Î% = 1.37%
#$

(# 3#K%)
For Oct. 01 = Í #K%
× 100Î% = 4.05%
(!%%3# )
For Dec. 01 = Í #
× 100Î% = 3.90%
(!% 3!%%)
For Feb. 02 = Í × 100Î% = 1.25%.
!%%

Clearly, highest percentage increase in marks is in Oct. 01.


8. The following line-graph gives the ratio of the amounts of imports by a
company to the amount of exports from that company over the period from
1995 to 2001. The questions given below are based on this graph.

226
Ratio of Value of Imports to Exports by a Company over the Year

1.8
1.55
1.6 1.4
1.4 1.25
1.2
0.95
1 0.85
0.8 0.65
0.6
0.35
0.4
0.2
0
1995 1996 1997 1998 1999 2000 2001

1. In how many of the given years were the exports more than the imports?
a. 1 b. 2 c. 3 d. 4 e. None of these
2. The imports were minimum proportionate to the exports of the Company
in the year:
a. 1995 b. 1996 c. 1997 d. 2000 e. 2001
3. If the imports of the Company in 1996 was Rs. 272 crores, the exports
from the Company in 1996 was:
a. Rs. 370 crores b. Rs. 320 crores c. Rs. 280 crores d.
Rs. 275 crores e. Rs. 264 crores
4. What was the percentage increase in imports from 1997 to 1998?
a. 72 b. 56 c. 28 d. None of these e.
data inadequate
5. If the imports in 1998 was Rs. 250 crores and the total exports in the
years 1998 and 1999 together was Rs. 500 crores, then the imports in
1999 was:
a. Rs, 250 crores b. Rs. 300 crores c. Rs. 357 crores d.
Rs. 420 crores e. None of these
Sol:- 1. (d): The exports are more than the imports implies that the ratio
of value of imports to exports is less than 1.
Now, this ratio is less than 1 in the years 1995, 1996, 1997 and 2000.
Thus, there are four such years.
Sol:- 2. (c): The imports are minimum proportionate to the exports
implies that the ratio of the value of imports to exports has the minimum
value.
Now, this ratio has a minimum value of 0.35 in 1997, i.e. the imports are
minimum proportionate to the exports in 1997.
Sol:-3. (b): Ratio of imports to exports in the year 1996 = 0.85.

227
Let the exports in 1996 = Rs. X crores.
K K
Then, = 0.85 ⇒ x = = 320.
%.

∴ Exports in 1996 = Rs. 320 crores.


Sol:- 4. (e): The graph gives only the ratio of imports to exports for
different years. To find the percentage increase in imports from 1997 to
1998, we require more details such as the value of imports or exports
during these years. Hence, the data is inadequate to answer this
question.
Sol:- 5. (d): The ratio of imports to exports for the years 1998 and 1999
are 1.25 and 1.40 respectively.
Let the exports in the year 1998 = Rs, x crores.
Then, the exports in the year 1999 = Rs. (500 – x) crores.
% %
∴ 1.25 = ⇒x=
.
= 200 [Using ratio for 1998]

Thus, the exports in the year 1999 = Rs. (500 – 200) crores
= Rs. 300 crores.
Let the imports in the year 1999 = Rs. Y crores.

Then, 1.40 = ⇒ y = (300 × 1.40 ) = 420.


#%%

∴ Imports in the year 1999 = Rs. 420 crores.

228
LCM & HCF
FACTORS: - factors of a number are the n umbers which will divide the given
number without leaving any remainders.
MULTIPLES: - Multiples of a number are generated by multiplying other integers
with the given number.
HIGHEST COMMON FACTOR (HCF): - The highest common factor of two numbers
is the largest number that will divide both the numbers without leaving a remainder. It
is also called as the greatest common divisor of GCD.
Example: - Consider the numbers 12 and 15.
Factors of 12: 1, 2, 3, 4, 6 and 12
Factors of 15: 1, 3, 5 and 15
Common factors: 1, 3
Highest common factor: 3
LEAST COMMON MULTIPLE (LCM):- The least common multiple of two numbers
is the lowest number that is a multiple of both the numbers.
EXAMPLE: - Consider the numbers 12 and 15.
Multiples of 12: 12, 24, 36, 48, 60, 72, 84, 96, 108, 120, 132…….
Multiples of 15: 15, 30, 45, 60, 75, 90, 105, 120, 135……
Common multiples: 60, 120……..
Least common multiple: 60
NOTE: - The same concepts of HCF and LCM can be extended for more than two
numbers.
FINDING HCF & LCM WITH PRIME FACTORIZATION: - Suppose we want to find
the HCF and LCM of the numbers 60 and 72, we start by writing each number as a
product of its prime factors.
60 = 2 x 2 x 3 x 5
72 = 2 x 2 x 2 x 3 x 3
To make the next stage easier, we need to write these so that each new
prime factor begins in the same place:
60 = 2 x 2 x 3 x 5
72 = 2 x 2 x 2 x 3 x 3
All the “2”s are now above each other, as are the “3”s etc. This allows us to
match up the prime factors.

229
The highest common factor is found by multiplying all the factors which
appear in both lists.
60 = 2 x 2 x3x5
72 = 2 x 2 x 2 x 3 x 3
HCF= 2 x 2 x3
The lowest common multiple is found by multiplying all the unique factors
which appear in either list.
So the LCM of 60 and 72 is 2 x 2 x 2 x 3 x 3 x 5 which is 360.

1. The largest possible length of a tape which can measure 525 cm, 1050 cm and 1155
cm length of clothes in minimum number of attempts without measuring the length of
a cloth in a fraction of the tape’s length is:

Solution: HCF of 525, 1050, 1155


525= 105 x 5, 1050= 105 x10, 1155= 105 x11
HCF=105 , So largest possible length= 105 cm
2. A milkman has three different qualities of milk, 403 litres of first quality, 465 litres of
second quality 496 litres of third quality. Find the least possible number of bottles of
equal size in which milk of different qualities can be filled without mixing.
HCF OF 403, 465, 496
403= 31 x 13, 465= 31 x15, 496=31 x16, so HCF=31
Maximum capacity of bottle= 31
Least possible number of bottles= (403/31)+(465/31)+(496/31)=13+15+16=44
3. The LCM and HCF of 3/5, 4/7 and 2/9 are
LCM of 3/5,4/7,2/9 is LCM of numerators/HCF of denominator
LCM of 3,4,2 = 12, HCF of 5,7,9 =1
LCM/HCF = 12
HCF of 3/5,4/7,2/9 is HCF of numerators/LCM of denominator
HCF of 3,4,2 = 1
LCM of 5,7,9 =315
HCF/LCM= 1/315

230
APPLICATIONS OF HCF & LCM
NOTE:- Find the greatest number that will exactly divide a, b & c.
APPROACH:- HCF (a, b, c)
Q. Find the greatest number that will exactly divide 24, 36 and 42.
HCF of 24, 36, 42 = 6
NOTE:- Find the greatest number that will divide a, b and c leaving remainders of x,
y and z respectively.
APPROACH:-HCF (a – x, b – y, c – z)
Q. Find the greatest number that will divide 27, 38 and 47 leaving remainders of 3, 2
and 5 respectively.
HCF of 27-3, 38-2, 47-5= HCF of 24,36,42=6
NOTE:- Find the greatest number which when it divides a, b and c will leave the
same remainder in each case.
APPROACH:- HCF (a – b, b – c, c – a) positive values only(take difference)
Q. Find the greatest number which when it divides 74, 110 and 182 will leave the
same remainder in each case.
HCF of 110-74,182-110, 182-74= HCF of 36,72,108=36

NOTE:- Find the least number which is exactly divisible by a, b and c.


APPROACH:- LCM (a, b, c)
Q. Find the least number which is exactly divisible by 12, 8 and 10.
LCM of 12,8, 10 = 60
NOTE:- Find the least number which when divided by a, b and c leaves the same
remainder ‘r’ in each case.
APPROACH:- LCM (a, b, c) + r
Q. Find the least number which when divided by 12, 8 and 10 leaves the same
remainder ‘7’ in each case.
LCM of (12,8, 10)+7 = 60+7=67

NOTE:- Find the least number which when divided by a, b and c leaves the
remainders x, y and z respectively.
APPROACH:- APPROACH:- Check if a – x = b – y = c – z = k. If this is the case,
then LCM (a, b, c)-k

231
Q. Find the least number which when divided by12, 8 and 10 leaves the remainders
10, 6 and 8 respectively.
Solution : 12-10=8-6=10-8=2 , k=2
LCM of (12,8, 10)-2= 60-2=58
Placement problems

1. Two lighthouses can be seen from the top of the hill. The first flashes once every 3
seconds and the other flashes once every 5 seconds. If they flash at the same time,
how long will it be before they flash at the same time again
Solution :
LCM of 3,5 = 15 sec
2. In a seminar, the number of participants, Chemistry and Mathematics are 96, 36
and 180 respectively. Find the minimum number of rooms required if in each room
the same number of participants are to be seated and all of them being in the same
subject.
Solution: HCF of 36,96,180= 12
Minimum number of rooms= (36/12)+(96/12)+(180/12)= 3+8+15=26
3. Three bells chime at an interval of 18, 24 and 32 minutes respectively. At a certain
time they begin to chime together. What length of time will elapse before they
chime together?
Solution :LCM of 18,24,32 = 288 min so Required answer is 4 hours 48 minutes
4. The ratio of two numbers is 15 : 11 and their HCF is 13. What is the sum of the
two numbers?

HCF of 15,11 is 1 part


1 part = 13
Sum of the numbers = 16 parts =26 x13=338
5. The ratio of two numbers is 5 : 6 and their LCF is 300. What is the HCF two
numbers?
Solution: 5,6 LCM =30 parts
30 parts=300, 1 part=10, HCF of 5,6 = 1 part= 10
6. Four bells are heard at intervals of 2,3,5 and 10 minutes respectively. If all bells
toll together exactly 9 a.m, they will again be heard together at ?

232
Solution: 9a.m + LCM of (2,3,5,10)=9a.m + 30 minutes= at 9:30 a. m
7. Four bells are heard at intervals of 2,3,5 and 10 minutes respectively. How many
times all bells toll together exactly in one hour ?
Solution: LCM of (2,3,5,10)=30 minutes
In one hour they will toll together = (60/30) +1=3 times

233

Potrebbero piacerti anche